Biochem II Mid Sem 2 MCQs Answered Sami

Download as pdf or txt
Download as pdf or txt
You are on page 1of 339

LEVEL 2 BIOCHEMISTRY

MID SEMESTER 2 MCQs


Compiled by Jitu

> 2" Update


Answered by Haron Sami

Compiled by Jitu 2023


TABLE OF CONTENTS

This Collection of Level 2 Biochemistry MCQs for Revision on Mid Semester 2 Revision was Extracted
from the following past University of Nairobi Biochemistry examination papers

PAPERS PAGE

END OF YEAR 2021/2022 EXTRACTED QUESTIONS 3

MID SEMESTER 2 CAT 2021/2022 30

MID SEMESTER 2 MAKE UP CAT 2021/2022 55

MID SEMESTER 2 CAT 2020/2021 96

MID SEMESTER 2 MAKE UP CAT 2020/2021 146

END OF YEAR 2020/2021 EXTRACTED QUESTIONS 179

BACTERIA BIOCHEMISTRY MOCK CAT 2019/2020 214


(With Answers)

MID SEMESTER 2 CAT 2019/2020 EXTRACTED QUESTIONS 224

END OF YEAR 2019/2020 EXTRACTED QUESTIONS 244

SPECIAL/SUPPLEMENTARY 2019/2020 EXTRACTED QUESTIONS 282

END COVER 339

Compiled by Jitu 2023


END OF YEAR
2021/2022
EXTRACTED
QUESTIONS

When birds build their nests, they use other


birds' feathers as mattress

~Pokomo Proverb
Mycourses Biochemistry Topic 1 HBC 200: BIOCHEMISTRY EXAMINATION

Started on Monday, 13 June 2022, 9:10 AM


State Finished
Completed on Monday, 13 June 2022, 11:10 AM
Time taken 1 hour 59 mins

Question 2 Regarding oncogenes and tumor suppressor genes, select the one FALSE statement:
Complete
Marked out of 1 Select one:
a. Mutation of one tumor suppressor gene or one oncogene is thought to be sufficient to
cause cancer.

b. The product of an oncogene shows a gain of function that signals cell division.
c. Mutation of an oncogene occurs in somatic cells and is not inherited.
d. RB and P53 are tumor suppressor genes; MYC and RAS are oncogenes.
e. Both copies of a tumor suppressor gene must be mutated for its product to lose its
activity.
Question 9 A bacterium is infected with an experimentally constructed bacteriophage composed of the
Complete T2 phage protein coat and T4 phage DNA. The new phages produced would have
Marked out of 1
Select one:
a. T4 protein and T2 DNA.

b. T4 protein and T4 DNA.

c. T2 protein and T2 DNA.


d. a mixture of the DNA and proteins of both phages.

e. T2 protein and T4 DNA.


Question 1 3 Which of the following is an example of post-transcriptional control of gene expression?
Complete
Marked out of 1
Select one:
a. the binding of transcription factors to a promoter

b. gene amplification contributing to cancer

c. the folding of DNA to form heterochromatin


d. the removal of introns and alternative splicing of exons

e. the addition of methyl groups to cytosine bases of DNA

co

Question 14 The binding of prokaryotic DNA dependent RNA polymerase to promoter sites of genes is
Complete inhibited by the antibiotic:
Marked out of 1
Select one:
a. Puromycin

b. Aureomycin

c. Tetramycin

d. Streptomycin
e. Rifampicin
Question 1 5 Which of the following characteristics, structures, or processes is common to both bacteria
Complete
and viruses?
Marked out of 1
Select one:
a. metabolism

b. genetic material composed of nucleic acid

c. ribosomes

d. independent existence

e. cell division
cr

Question 1 7 Steroid and peptide hormones typically have in common


Complete
Marked out of 1
Select one:
a. the location of their receptors.

b. the building blocks from which they are synthesized.

c. their solubility in cell membranes.


d. their requirement for travel through the bloodstream.

e. their reliance on signal transduction in the cell.


Question 21 What would occur if the repressor of an inducible operon were mutated so it could NOT bind
Complete the operator?
Marked out of 1
Select one:
a. irreversible binding of the repressor to the promoter
b. continuous transcription of the operon’s genes

c. buildup of a substrate for the pathway controlled by the operon

d. overproduction of catabolite activator protein (CAP)


e. reduced transcription of the operon’s genes
Question 23 The following statement is FALSE about Rifampicin
Complete
Marked out of 1
Select one:
a. It can bind specifically to the B subunit of RNA-polymerase

b. It inhibits RNA synthesis in eukaryotes without great toxicity


c. It inhibits RNA synthesis in prokaryotes

d. It interferes with tetrahydrofolate metabolism


e. Itis used for treatment of tuberculosis

Question 24 The normal function of restriction endonucleases is to:


Complete
Marked out of 1
Select one:
a. Remove primer from Okazaki fragments
b. Protect bacteria from foreign DNA
c. Polymerise nucleotides to form RNA

d. Excise introns from hn-RNA

e. Repair damaged DNA

Question 25 Which of the following is NOT a chemical mutagen?


Complete
Marked out of 1 Select one:
a. Nitrous acid

b. X-rays
PHYSICAL

c. Nitrosoguanidine

d. Bromouracil
e. Ethidium bromide
r

Question 26 Growth factors are local regulators that


Complete
Marked out of 1
Select one:
a. bind to cell-surface receptors and stimulate growth and development of target cells.
b. convey messages between nerve cells.
c. are found on the surface of cancer cells and stimulate abnormal cell division.
d. are produced by the anterior pituitary.

e. are modified fatty acids that stimulate bone and cartilage growth.
rc

Question 29 Which of the following is NOT an accurate statement?


Complete
Marked out of 1
Select one:
a. Hormones are chemical messengers that travel to target cells through the circulatory
system.
b. Hormones of the same chemical class usually have the same function.

c. Hormones often regulate homeostasis through antagonistic functions.

d. Hormones are often regulated through feedback loops.


e. Hormones are secreted by specialized cells usually located in endocrine glands.
Question 37 Which of the following is the most likely explanation for hypothyroidism in a patient whose
Complete iodine level is normal?

Marked out of 1
Select one:
a. greater production of T3 than of T4
b. hypersecretion of MSH

c. hypersecretion of TSH

d. a decrease in the thyroid secretion of calcitonin


e. hyposecretion of TSH
Question 47 To cause a human pandemic, the H5N1 avian flu virus would have to:
Complete
Marked out of 1 Select one:
a. become capable of human-to-human transmission.

b. arise independently in chickens in North and South America.

c. spread to primates such as chimpanzees.


d. develop into a virus with a different host range.

e. become much more pathogenic.


Question 59 In the “Lac operon” concept, which of the following is a protein?
Complete
Marked out of 1
Select one:
a. Promoter

b. Vector

c. Operator

d. Repressor

e. Inducer
Question 64 What must happen in order to prevent overstimulation by a hormone?
Complete
Marked out of 1
Select one:
a. G-proteins must be recycled and then degraded.
b. Receptors must dimerize.

c. Receptors must be blocked from continuing to activate G-proteins.

d. Hormones must be degraded.


e. None of the above
Question 66 Consider this pathway: epinephrine — G protein-coupled receptor — G protein ~ adenylyl
Complete cyclase — cAMP. Identify the second messenger.
Marked out of 1
Select one:
a. CAMP

b. GTP

c. adenylyl cyclase
d. G protein-coupled receptor

e. G protein

Question 67 Which hormone is INCORRECTLY paired with its action?


Complete
Marked out of 1 Select one:
a. melatonin—affects biological rhythms, seasonal reproduction

b. insulin—stimulates glycogen breakdown in the liver

c. oxytocin—stimulates uterine contractions during childbirth


d. thyroxine—stimulates metabolic processes

e. ACTH—stimulates the release of glucocorticoids by the adrenal cortex


Question 75 Which hormone is secreted by a cells in the pancreas in response to low blood glucose
Complete
levels?
Marked out of 1
Select one:
a. Insulin

b. Glucagon

c. Somatostatin

d. Epinephrine

e. Estradiol
c

Question 77 Regarding viral carcinogenesis, select the one FALSE statement:


Complete
Marked out of 1
Select one:
a. Approximately 15% of human cancers may be caused by viruses.

b. Tumor viruses act by deregulating the cell cycle, inhibiting apoptosis, and interfering
with normal cell signaling processes.

c. RNA viruses causing or associated with tumors include hepatitis C virus.


d. Retroviruses possess reverse transcriptase, which copies RNA to DNA.

e. Only RNA viruses are known to be carcinogens.


Question 74 Place the events of signaling listed below in the CORRECT order: (1) G-protein binds to
Complete activated receptor forming a receptor—G-protein complex; (2) Release of GDP by the G-
protein; (3) Change in conformation of the cytoplasmic loops of the receptor; (4) Binding of
Marked out of 1
GTP by the G-protein; (5) Increase in the affinity of the receptor for a G-protein on the
cytoplasmic surface of the membrane; (6) Binding of a hormone or neurotransmitter to a G-
protein-coupled receptor; (7) Conformational shift in the a subunit of the G-protein.

Select one:
a.6-§-4-1-7-2-3

b.6-3-5-4-7-2-1

6. 6< 3=9=
1787 =2 =4
qG. 6=3=9=
1 =2=4=7

2 6=-7=3-=6=1-2=4
.
Question 83 An example of antagonistic hormones controlling homeostasis is
Complete
Marked out of 1 Select one:
a. thyroxine and parathyroid hormone in calcium balance.

b. oxytocin and prolactin in milk production.


c. epinephrine and norepinephrine in fight-or-flight responses.

d. progestins and estrogens in sexual differentiation.


e. insulin and glucagon in glucose metabolism.

;
Question 84 What happens to protein kinase A (PKA) following the binding of cAMP?
Complete
Marked out of 1 Select one:
a. The stimulatory regulatory subunits dissociate from the catalytic subunits, inhibiting
the enzyme.
b. The regulatory subunits of PKA dissociate, thereby activating the catalytic subunits.
c. PKA catalytic subunits then bind to two regulatory subunits, thereby activating the
catalytic subunits.
d. Phosphodiesterase binds to the catalytic subunits, which results in enzyme
inactivation.

e. The inhibitory regulatory subunits dissociate from the catalytic subunits, completely
inactivating the enzyme.
c

Question 86 The region of the Lac operon which must be free (unbound) for structural gene transcription
Complete
to occur is:
Marked out of 1
Select one:
a. The ‘z’ gene

b. The operator locus

c. The promoter site


d. The ‘a’ gene

e. The ‘i’ gene


Question 94 RNA viruses require their own supply of certain enzymes because
Complete
Marked out of 1
Select one:
a. these enzymes penetrate host cell membranes.
b. host cells lack enzymes that can replicate the viral genome.

c. host cells rapidly destroy the viruses.


d. these enzymes cannot be made in host cells.
e. these enzymes translate viral mRNA into proteins.

Question 95 Which observation suggested to Sutherland the involvement of a second messenger in


Complete epinephrine's effect on liver cells?
Marked out of 1
Select one:
a. Enzymatic activity was proportional to the amount of calcium added to a cell-free
extract.
b. Epinephrine was known to have different effects on different types of cells.

c. Glycogen breakdown was observed when epinephrine and glycogen phosphorylase


were combined.

d. Glycogen breakdown was observed only when epinephrine was administered to intact
cells.
e. Receptor studies indicated that epinephrine was a ligand.
Question 99 Proto-oncogenes can change into oncogenes that cause cancer. Which of the following best
Complete
explains the presence of these potential time bombs in eukaryotic cells?
Marked out of 1
Select one:
a. Proto-oncogenes first arose from viral infections.
b. Cells produce proto-oncogenes as they age.

c. Proto-oncogenes are mutant versions of normal genes.

d. Proto-oncogenes normally help regulate cell division.


e. Proto-oncogenes are genetic “junk.”
Question 105 \f a particular operon encodes enzymes for making an essential amino acid and is regulated
Complete like the trp operon, then
Marked out of 1
Select one:
a. the enzymes produced are called inducible enzymes.

b. the amino acid inactivates the repressor.

c. the amino acid turns on transcription of the operon.


d. the amino acid acts as a corepressor.

e. the repressor is active in the absence of the amino acid.


Question 1 08 Emerging viruses arise by
Complete
Marked out of 1
Select one:
a. the spread of existing viruses to new host species.

b. all of the above

c. the spread of existing viruses more widely within their host species.

d. mutation of existing viruses.

e. none of the above


cr

Question 111 Which of the following hormones termed the “flight-or-fight” hormone is secreted by the
Complete adrenal medulla?
Marked out of 1
Select one:
a. Oxytocin

b. Epinephrine

c. Somatostatin

d. Glucagon
e. Insulin
Question 112 Shortly after ingesting a big plate of carbohydrate-rich pasta, you measure your bload's
Complete hormone levels. What results would you expect, compared to before the meal?
Marked out of 1
Select one:
a. high insulin, high glucagon
b. low insulin, low glucagon
c. low insulin, no change in glucagon
d. low insulin, high glucagon

e. high insulin, low glucagon


Question 1 1 8 Protein phosphorylation is commonly involved with all of the following except
Complete
Marked out of 1
Select one:
a. enzyme activation.

b. regulation of transcription by extracellular signaling molecules.

c. activation of protein kinase molecules.

d. activation of G protein-coupled receptors.


e. activation of receptor tyrosine kinases.
MID SEMESTER 2
CAT 2022

If the lizard of the homestead should neglect to do


the things for which its kind is known,
it will be mistaken for the lizard of the farmland.

Compiled by Jitu 2022

Answered by Haron Sami


Search Courses Q (Search Courses)

& > Mycourses > Biochemistry > Topic1 >» HBC200: MEDICAL BIOCHEMISTRY CAT II

Started on Tuesday, 12 April 2022, 2:07 PM


State Finished
Completed on Tuesday, 12 April 2022, 3:10 PM
Time taken 1 hour 2 mins

]
Question 1 The rho (p) factor is involved:
Complete
Marked out of 1 Select one:
a. To increase the rate of RNA synthesis
b. To allow proper initiation of transcription
c. In binding catabolite represssor to the promoter region
d. In proper termination of transcription
e. To eliminate binding of RNA polymerase to the promoter

Question 2 What is the mechanism of action of the aminoglycosides?


Complete
Marked out of 1 Select one:
a. Inhibit the DNA gyrase
b. Inhibit mycolic acid synthesis
c. Damage to the membrane
d. Block initiation complex
e. Inhibit peptide chain elongation
Question 3 Recombination is required for stabilization of genetic material newly transferred by
Complete all of the following processes EXCEPT
Marked out of 1
Select one:
a. Integration of a temperate bacteriophage

b. Movement of a transposon

c. Conjugal transfer of an R factor

d. Transduction of a chromosomal gene


e. Transformation of a chromosomal gene

Question 4 Within a cell, the amount of protein made using a given MRNA molecule depends
Complete partly on
Marked out of 1
Select one:
a, the degree of DNA methylation.
b. the types of ribosomes present in the cytoplasm.
c. the rate at which the mRNA is degraded.
d. the presence of certain transcription factors.

e. the number of introns present in the MRNA.

Question 5 Which of the following enzyme catalyses reactions in the biosynthesis of both
Complete catecholamines and serotonin?
Marked out of1 Aromatic L-amino acid decarboxylase
Select one: (AAAD) is the second enzyme in the
a. Dopamine-B-hydroxylase sequence leading to the synthesis of the
catecholamines and serotonin, and it is the
b. Tryptophan hydroxylase
rate-limiting enzyme for the synthesis of the
c. Tyrosine hydroxylase trace amines.
d. Aromatic amino acid decarboxylase
e. Phenyl ethanolamine N-methy! transferase

Question 6 A viral disease in which antiviral antibodies are thought to have an “enhancing” effect
Complete so that reinfection with the virus causes a more serious disease is
Marked out of1
Select one:
ADE has been observed with dengue virus 2, Zika virus 3,
a. Adenovirus E1A
Ebola virus 4 and, importantly in the context of COVID
b. West Nile virus

c. Dengue virus

d. Coxsackie A virus

e. Hanta virus
Question 7 The attachment of HIV to a TH cell is initiated by the binding of the HIV gp |20
Complete molecule to the CD4 receptor. Then gp | 20 undergoes a conformational change and
Marked out of 1
binds to a second molecule. What is the second molecule on the surface of the CD4
lymphocyte that acts as a coreceptor and binds with the lymphotropic HIV?

Select one:
a. The chemokine receptor CCRS expressed on dendritic cells
b. The complement receptor CR2 (CD2 1 ) on B cells

c. The chemokine receptor CXCRS expressed on T cells

d. The chemokine receptor CXCR4 expressed on T cells

e. The cytokine receptor CCRS expressed on T cells

Question 8 Cholera toxin produces diarrhoea due to:


Complete
Marked out of 1 Select one:
a. By increasing synthesis of C-AMP

b. Inhibits the Na+ - K+ stimulated ATPase


c. Prevents axonal ion transport

d. By decreasing C-AMP level

e. Stimulate cholinergic receptors

Question 9 Phosphorylation cascades involving a series of protein kinases are useful for cellular
Complete signal transduction because
Marked out of 1
Select one:
a. the number of molecules used is small and fixed.

b. they amplify the original signal manyfold.

c, they counter the harmful effects of phosphatases.


d. they are species specific.

e. they always lead to the same cellular response.

Question 10 The first viral-induced defense mechanism in a non-immune individual is the:


Complete
Marked out of 1 Select one:
a. Synthesis of neutralizing antibodies
b. Production of interferon

c. Generation of cytotoxic T lymphocytes

d. Synthesis of lymphokines
e. Production of complement system
Question 11 All of the following hormones use cyclic-AMP as a second messenger except:
Complete
Marked out of 1 Select one:
a. LH

b. Epinephrine

c. FSH

d. Estrogen
e. Glucagon

Question 12 Linear, single-stranded DNA is the genetic material of


Complete
Marked out of 1 Select one:
a. Flaviviruses

b. Parvoviruses
c. Calciviruses

d. Papillomaviruses

e. Retroviruses

Question 13 b-lactamases confer antibiotic resistance by


Complete
Marked out of 1 Select one:
a. Altering antibiotic permeability
b. Modifying antibiotic structure
c. Altering 70S ribosome structure
d. Modifying cellular RNA polymerase

e. Altering penicillin-binding proteins

Question 14 The first drug to be effective against AIDS, including the reduction of maternal-to-
Complete child AIDS transmission by 30%, was AIDS drug azidothymidine (AZT). Which of the
Kiaked oukur following describes its mechanism of action?

Select one:
a. It blocks viral budding
b. It stimulates DNA provirus production
c. It inhibits viral reverse transcriptase

d. It inhibits RNA synthesis

e. It inhibits viral DNA polymerase


Question 15 Which of the following is an example of post-transcriptional control of gene
Complete expression?
Marked out of 1
Select one:
a. the removal of introns and alternative splicing of exons

b. the binding of transcription factors to a promoter

c. the folding of DNA to form heterochromatin

d. gene amplification contributing to cancer


e. the addition of methyl! groups to cytosine bases of DNA

Question 16 A pharmaceutical firm is interested in the bacterial production of thymidylate


Complete synthase in large quantities for drug-targeting studies. An important step in the
Marked out of 1
overall cloning strategy involves the ligation of synthase cDNA into a plasmid vector
containing a replication origin, an antibiotic resistance gene, and a promoter
sequence. Which additional nucleotide sequence should be included in this vector to
ensure optimal production of the thymidylate synthase?

Select one:
a. Operator sequence

b. Attenuator sequence
c. Shine-Dalgarno sequence

d. 3’-splice acceptor sequence

e. PolyA sequence

Question 17 50S ribosomal subunits are found in


Complete
Marked out of 1 Select one:
a. Viruses

b. Prions

c. Protozoa

d. Fungi
e. Bacteria
Question 18 The microbiome of the large intestine consists mainly of:
Complete
Marked out of 1 Select one:
a. Fungi

b. Protozoa

c. none of the above

d. Bacteria

e. Viruses

Question 19 What compound is only found in Gram-positive bacteria?


Complete
Marked out of 1 Select one:
a. Capsule

b. Lipopolysaccharide

c. Teichoic acid

d. Peptidoglycan

e. Outer membrane

Question 20 Aminoglycoside antibiotics are


Complete
Marked out of 1 Select one:
a, Require bacterial growth for the effect
b. Bactericidal for Gram-positive bacteria
c. Mycolic acid synthesis inhibitors

d. Inactivated by R-factor phosphotransferases

e. Peptidoglycan synthesis inhibitors

Question 21 A 27-year-old man is seen by his physician for a week-long cough, sore throat, and
Complete difficulty swallowing. He is diagnosed with diphtheria, which has reactivated because
Kiaked oukur of waning immunity. One way in which diphtheria toxin leads to cell death is through
the inhibition of eEF-2. Which statement best explains the function of eEF-2?

Select one:
a. Itis the agent that binds to, and is inactivated by, chloramphenicol.
b. It functions as a peptidyl transferase.

c. It is analogous to the prokaryotic factor elF-1.

d. It is required for the translocation of peptidyl-tRNA during translation.


e. It is required for the initiation of protein synthesis.
Question 22 A 32-year-old woman is referred to an endocrinologist for weight gain, especially
Complete around the waist. She also has striae over the abdomen and a rounded appearance
Marked out of 1
to her face. She is found to have Cushing disease. Which of the following would most
likely be found in this patient, compared with someone who does not have Cushing
disease?

Select one:
a. Increased synthesis of immunoglobulins
b. Reduced liver glycogen stores
c. Increased gluconeogenesis

d. Increased protein synthesis

e. Inhibition of lipolysis

Question 23 Which of the following stimulates the production of progesterone by the corpus
Complete luteum?
Marked out of 1
Select one:
a. GH

b. LH

c. TSH

d. PRL

e. FSH

Question 24 What is transferred when an F+ cell is crossed with an F- cell?


Complete
Marked out of 1 Select one:
a. Generally the whole bacterial chromosome
b. No genes
c. Only some bacterial chromosomal genes

d. Both the plasmid and chromosomal genes


e. Only the fertility factor DNA
Question 25 A 30-year-old woman presents to an infertility clinic complaining that she has been
Complete unsuccessfully trying to have a baby for 2 years. Her physician runs some tests,
Marked out of 1
which show that she may not be ovulating. Which of the following is responsible for
ovulation?

Select one:
a. LH surge

b. Increased FSH

c. Increased progesterone
d. Increased progesterone and estradiol

e. Increased estradiol

Question 26 If the percentage concentration of thymine in DNA is 40%, the concentration of


Complete cytosine will be:
Marked out of 1
Select one:
a. 30%

b. 10%

c. 20%
d. 50%

e. 40%

Question 27 It is now believed that a substantial proportion of the single nucleotide substitutions
Complete causing human genetic disease are due to misincorporation of bases during DNA
Marked out of 1
replication. Which proofreading activity is critical in determining the accuracy of
nuclear DNA replication and thus the base substitution mutation rate in human
chromosomes?

Select one:
a. 5’ to 3’ polymerase activity of DNA polymerase III

b. 3’ to 5’ exonuclease activity of DNA polymerase y

c. 3’ to 5’ polymerase activity of DNA polymerase &


d. Primase activity of DNA polymerase a

e. 3’ to 5’ exonuclease activity of DNA polymerase 6

The 3' to 5' exonuclease activity of DNA pol.delta represents the proofreading
activity of an enzyme required for the replication of human chromosomal DNA.
DNA pol y (mitochondrial) and DNA pol III (prokaryotic) do not participate in
this process, short RNA primers are replaced with DNA during replication, and
new DNA strands are always synthesized in the 5' to 3' direction
Question 28 HTLVs Human T-lymphotropic virus type 1 (HTLV-1) is associated with adult
Complete T-cell leukemia; HTLV-2 is implicated in human hairy cell leukemia.
Marked out of 1 Select one:
a. Carry tyrosine protein kinase oncogenes

b. Are DNA viruses

c. Are defective RNA tumor viruses


d. Synthesize early proteins that interact with p53Rb
e. Are associated with leukemias

Question 29 Enteroviruses differ from rhinoviruses mainly in their:


Complete
Marked out of 1 Select one:
a. Capsid shape

b. Tropism
c. Ability to survive acidic conditions

d. Strandedness
e. Type of nucleic acid

Question 30 An immigrant family from rural Mexico brings their 3-month-old child to the
Not answered emergency room because of whistling inspiration (stridor) and high fever. The child’s
Marked Gueat't physician is perplexed because the throat examination shows a gray membrane
almost occluding the larynx. A physician recognizes diphtheria, now rare in
immunized populations. The child is intubated, antitoxin is administered, and
antibiotic therapy is initiated. Diphtheria toxin is often lethal in unimmunized
persons because it does which of the following?

Select one:
a. Binds to the signal recognition particle receptor on the cytoplasmic face of the
endoplasmic reticulum receptor
b. Shuts off signal peptidase

c. Causes deletions of amino acid by speeding up the movement of peptidyl-tRNA


from the A site to the P site

d. Blocks elongation of proteins by inactivating elongation factor 2 (EF-2, or


translocase)

e. Inhibits initiation of protein synthesis by preventing the binding of GTP to the


40S ribosomal subunit
Question 31 A 13-year-old boy has developed polydipsia, polyuria, polyphagia, and weight loss
Complete over the past few weeks. He was brought to the emergency room by his parents
Marked out of 1
because he woke up this morning very lethargic. His blood glucose was found to be
600 mg/dL, and he was immediately placed on an insulin drip. Insulin works primarily
by which one of the following mechanisms? Insulin binds to extracellular receptors, promoting
dimerization and subsequent phosphorylation of tyrosine
residues on the intracellular portion of the receptor. This
Select one: leads to increased tyrosine phosphorylation of other
proteins, which mediate the intracellular
a. Causing phosphorylation of tyrosine residues effects of insulin action, including glucose uptake and
storage. Hormones like epinephrine bind
b. Producing cGMP to G protein-coupled receptors with activation of
adenylate cyclase. Insulin's primary effect is
c. Activating adenylate cyclase not achieved via activation of adenylate cyclase. Steroid
hormones are lipid soluble and diffuse
d. Activating caspases through the cell membrane, binding intracellular receptors
that activate gene transcription. Caspases
e. Binding to an intracellular receptor are involved in apoptosis, a process that is antagonized
by insulin. ANP activates guanylate
cyclase, which produces cGMP.

Question 32 Apoptosis involves all but which of the following?


Complete
Marked out of 1 Select one:
a. fragmentation of the DNA

b. activation of cellular enzymes

c. cell-signaling pathways

d. digestion of cellular contents by scavenger cells

e. lysis of the cell

Question 33 The formation of a peptide bond during the elongation step of protein synthesis
Complete results in the splitting of how many high energy bonds?
Marked out of 1
Select one:
a. 1

b.2
Gs

d.5
e.4
Question 34 Which of these factors influences the types and amounts of bacteria found in your
Complete gut?
Marked out of 1
Select one:
a. The medications you take

b. Your overall health

c. The foods you eat

d. All of the answers are correct

e. Your age

Question 35 RNA viruses require their own supply of certain enzymes because
Complete
Marked out of 1 Select one:
a. these enzymes cannot be made in host cells.

b. these enzymes penetrate host cell membranes.

c. host cells rapidly destroy the viruses.


d. host cells lack enzymes that can replicate the viral genome.

e. these enzymes translate viral mRNA into proteins.

Question 36 The only correct statement about oncoviruses is


Complete
Marked out of1 Select one:
a. All the oncoviruses are DNA viruses

b. Viral oncogenes are identical to human protooncogenes

c. All the oncoviruses are RNA viruses

d. Both DNA and RNA viruses can be oncoviruses

e. Reverse transcriptase is present in all oncoviruses

Question 37 The exchange of allelic forms of genes is involved in which of the following
Complete processes?
Marked out of 1 DNA or genetic recombination is the general term
Select one: used to describe the exchange of allelic forms of
a. Recombination genes in bacteria or eukaryotic cells.
b. Transduction

c. Transcription

d. Transformation

e. Conjugation
Question 38 In t-RNA molecules, there is a ‘loop’ which contains a minor base:
Complete
Marked out of 1 Select one:
a. Dihydrouracil

b. Thymine

c. Uracil

d. Dihydrocytosine
e. Cytosine

Question 39 The binding of prokaryotic DNA dependent RNA polymerase to promoter sites of
Complete genes is inhibited by the antibiotic:
Marked out of 1
Select one:
a. Aureomycin

b. Streptomycin

c. Tetramycin

d. Puromycin

e. Rifampicin

Question 40 In formation of cyclic AMP the stimulation of adenyl cyclase by the hormone-receptor
Complete complex requires the presence of:
Marked out of1 In the formation of cyclic AMP (cAMP), the stimulation of
Select one:
a, UMP
adenylate cyclase by the hormone-receptor complex requires
the presence of a G protein. G proteins are activated by binding
b. GTP
to guanosine triphosphate (GTP) . When a hormone binds to its
c,; CTP receptor, it activates a G protein by causing it to exchange
d. CDP guanosine diphosphate (GDP) for GTP. The activated G protein
e. UTP
then activates adenylate cyclase, which converts ATP into
cAMP.

Question 41 Transcription factors bound at enhancers stimulate the initiation of transcription at


Complete the cis-linked core promoter through the action of intermediaries are termed as
Marked eater which of the following?
Coactivators are proteins that do not bind
Select one:
DNA directly but are recruited to the promoter
a. Corepressors
by DNA-bound transcription factors. They help
b. Cotranscription proteins to bridge the interaction between transcription
c. Coordinators factors bound at enhancers and the basal
d. Receptors
transcription machinery at the core promoter,
facilitating the assembly of the pre-initiation
e. Coactivators
complex and stimulating transcription
Question 42 In a DNA molecule, guanosine nucleotide is held by the cytosine nucleotide by the
Complete number of hydrogen bonds:
Marked out of 1
Select one:
a3

b.4
Gl

d:i2
e.5

Question 43 The type of RNA that characteristically contain methylated purines and pyrimidines
Complete is:
Marked out of 1
Select one:
a. r-RNA

b. None of the above


c. hn-RNA

d. m-RNA

e. t-RNA

Question 44 The digestive tract and the gut microbiome have a mutualistic relationship. What
Complete does this mean?

Marked out of1


Select one:
a. They do not have any known relationship.

b. They fight against each other.

c. They have the ability to cause mutations in each other.

d. They work together to survive and thrive .


e. They are antagonistic to one another.

Question 45 A 15-year-old member of the high school swim team notices painless, umbilicated
Complete cutaneous lesions on the toes. Large eosinophilic cytoplasmic inclusions are present
Marked out of1
in the affected epithelia.
Pp
What is the most likelyy: causal agent?
&

Select one:
a. B19 virus

b. Molluscum contagiosum virus

c. Adenovirus

d. Herpes simplex virus

e. Human papilloma virus


Question 46 A new infectious agent that lacks a cell wall but has 70S ribosomes. This agent is
Complete most likely a
Marked out of1
Select one:
a. Virus

b. Mycoplasma species

c. Bacterium

d. Rickettsia species
e. Chlamydia species

Question 47 The ability ofa cell to bind DNA to its surface and import it is required for which
Complete genetic process?
Marked out of 1
Select one:
a. Conjugation
b. Homologous recombination
c. Specialized transduction

d. Site-specific recombination

e. Transformation

Question 48 Which hormone is secreted by a cells in the pancreas in response to low blood
Complete glucose levels?
Marked out of 1
Select one:
a. Somatostatin

b. Glucagon

c. Epinephrine

d. Insulin

e. Estradiol

Question 49 N-acetylmuramic acid is located in


Complete
Marked out of 1 Select one:
a. Teichoic acid
b. Outer membrane
c. Lipoprotein

d. Peptidoglycan
e. Lipopolysaccharide
Question 50 Which hormone is INCORRECTLY paired with its action?
Complete
Marked out of 1 Select one:
a. oxytocin—stimulates uterine contractions during childbirth

b. insulin—stimulates glycogen breakdown in the liver

c. thyroxine—stimulates metabolic processes

d. ACTH—stimulates the release of glucocorticoids by the adrenal cortex


e. melatonin—affects biological rhythms, seasonal reproduction

Question 51 PG-Es lower cyclic AMP level in:


Complete
Marked out of 1 Select one:
a. Thyroids

b. Platelets

c. Spleen

d. Lungs
e. Adipose tissue

Question 52 Which of the following is NOT a natural ligand that binds to G protein-coupled
Complete receptors?
Marked out of1
Select one:
a. Chemoattractants
b. Neurotransmitters
c. Hormones

d. Opium derivatives

e. Steroid hormones

Question 53 Which of the following is the most likely explanation for hypothyroidism in a patient
Complete whose iodine level is normal?
Marked out of 1
Select one:
a. hyposecretion of TSH

b. hypersecretion of TSH
c. greater production of T3 than of T4
d. a decrease in the thyroid secretion of calcitonin

e. hypersecretion of MSH
Question 54 A middle-aged man presents to his physician because of anxiety attacks
Not answered accompanied by profuse sweating and heart palpitations. His physician documents a
iachadion bath high blood pressure of 175/110 (hypertension) and orders ultrasound studies that
show an adrenal tumor called pheochromocytoma. The man has also noted weight
loss and fatigue over the past month when the attacks began. Knowing that
pheochromocytoma releases epinephrine from the adrenal medulla, alteration of
which of the following metabolic processes best explains symptoms of decreased
energy and weight loss?

Select one:
a. Ketogenesis

b. Glycolysis

c. Lipolysis
d. Glycogenolysis

e. Gluconeogenesis

Question 55 A nonnucleoside analog that inhibits herpesvirus DNA replication is


Complete
Marked out of 1 Select one:
a. Foscarnet

b. Cytarabine

c. Amantadine
d. Acyclovir

e. Interferon

Question 56 2,5 Asynthetase is induced by:


Complete
Marked out of 1 Select one:
a. Acyclovir

b. Amantadine

c. Interferon

d. Ribavirin

e. Cytarabine
Question 57 Degeneracy of the genetic code denotes the existence of:
Complete
Marked out of 1 Select one:
a. Different protein synthesising systems in which a given triplet codes for
different amino acids
b. Multiple codons for a single amino acid
c. Codons that include one or more unusual bases

d. Base triplets that do not code for any amino acids

e. Codons consisting of only two bases

Question 58 Growth factors are local regulators that


Complete
Marked out of 1 Select one:
a. are modified fatty acids that stimulate bone and cartilage growth.

b. are produced by the anterior pituitary.

c. bind to cell-surface receptors and stimulate growth and development of target


cells.

d. convey messages between nerve cells.


e. are found on the surface of cancer cells and stimulate abnormal cell division.

Question 59 A teenage girl is evaluated in the emergency room and noted to have increased
Not answered respiration (tachypnea), vomiting, and confusion. She bleeds from puncture sites
Marked out of 1 during withdrawal of blood for testing, and a urine toxicology screen reveals
increased amounts of acetosalicylic acid, ibuprofen, and acetaminophen. She has a
past history of depression and suicide attempts. Which of the following enzymes are
likely to be inhibited?

Select one:
a. Cyclooxygenase

b. Phospholipase D

c. Phospholipase A2

d. Lipoprotein lipase

e, Lipoxygenase

Acetosalicylic acid (aspirin), ibuprofen, and acetaminophen are all nonsteroidal


anti-inflammatory drugs (NSAIDs) that work by inhibiting the cyclooxygenase (COX)
enzymes. These enzymes are responsible for the production of prostaglandins and
thromboxanes, which play a role in inflammation, pain, and fever. By inhibiting COX
enzymes, NSAIDs reduce the production of these mediators and alleviate symptoms
Question 60 The anticodon region is an important part of the structure of:
Complete
Marked out of 1 Select one:
a.r-RNA

b. hn-RNA

c. m-RNA

d. Z-DNA

e. t-RNA

Question 61 In March, a nursery school reports an outbreak among the children of a disease
Complete characterized by a high grade fever that starts suddenly and rapidly falls. The
Make oordkt children develop a short-lived (1 to 2 days) maculopapular rash that begins on the
trunk and spreads to extremities; the face is spared. The children have mild malaise,
but otherwise do not appear ill. The most likely causative agent is

Select one:
a. Parvovirus B19

b. Rubella virus

c. HHV-6

d. VZV

e. Coxsackie A16 virus

Question 62 PG Synthesis is increased by activating phospholipase by:


Complete
Marked out of 1 Select one:
a. Aspirin

b. Mepacrine

c. Glucocorticoids
d. Angiotensin II

e. d Indomethacin

Question 63 Bacteriophage containing host-cell DNA is involved in which of the following


Complete processes?
Marked out of 1
Select one: Bacteriophages containing portions of
a. Recombination host-cell DNA can introduce this genetic
b. Conjugation material into new host cells via the process
c. Transcription of transduction
d. Transformation
e. Transduction
Question 64 Of the following, which diagnostic test has been more commonly used in the
Complete diagnosis of COVID-19?
Marked out of 1
Select one:
a. Viral antigen detection test

b. Virus isolation in cell culture

c. Urinalysis

d. Real-time reverse transcription polymerase chain reaction (rRT-PCR) assay

e. Immunofluorescent assay (IFA)

Question 65 Which of the following amino acids produce a vasodilator on decarboxylation:


Complete
Marked out of 1 Select one:
a. Ornithine

b. Arginine

c. Cysteine

d. Glutamic acid

e. Histidine

Question 66 In hyperparathyroidism, which of the following is correct?


Complete
Marked out of1 Select one:
a. High serum phosphorus

b. High serum calcium and low serum phosphorus

c. None of the above


d. Low serum calcium

e. Low serum calcium and high serum phosphorus

Question 67 Passive immunization is available for protection from


Complete
Marked out of1 Select one:
a, Parainfluenza type 2 virus

b. Rubella virus
c. Hepatitis A virus

d. Yellow Virus

e. Influenza A virus
Question 68 A patient stung by a bee is rushed into the emergency room with a variety of
Complete symptoms including increasing difficulty in breathing due to nasal and bronchial
Marked out of 1
constriction. Although your subsequent treatment is to block the effects of histamine
and other acute-phase reactants released by most cells, you must also block the
slow-reacting substance of anaphylaxis (SRS-A), which is the most potent constrictor
of the muscles enveloping the bronchial passages. An SRS-A is composed of which of
the following?

Select one:
a. Complement

b. Prostaglandins
SRS-A is a mixture of LT-C4, LT-D4 and LT-E4
c. Interleukins

d. Leukotrienes

e. Thromboxanes

Question 69 Some individuals with diabetes mellitus are susceptible to rapid drops in blood sugar
Complete (hypoglycemia) with lethargy and potential seizures or coma. Such diabetics are
Marked out of 1
called “brittle” and require careful monitoring of glucose levels as proper insulin
doses are titrated. Of the many actions of insulin, decrease in which of the following
cellular activities best accounts for “brittleness” of certain diabetics?

Select one:
a. Glucose oxidation

b. Gluconeogenesis

c. Formation of ATP, DNA, and RNA

d. Lipogenesis

e. Plasma membrane transfer of glucose

Question 70 Indicate the term generally applied to the extracellular messenger molecules that
Complete bind to transmembrane receptor proteins.
Marked out of 1
Select one:
a. Substrate

b. Scatchard curve
c. Ligand
d. Competitive inhibitor

e. Key
Many viral infections are asymptomatic or subclinical.
Clinical disease, however, is often associated with viral
Question 71 A clinical viral disease
replication in target organs during disseminated viral
Complete
infections.
Marked out of 1 Select one:
a. Usually follows virus infection

b. Is a major feature of congenital viral infections

c. Can result without infection of host cells

d. Is most frequently due to toxin production

e. Is associated with target organs in most disseminated viral infections

Question 72 What effect does degradation of receptor-ligand complexes after internalization have
Not answered upon the ability of a cell to respond if immediately re-exposed to the same hormone?
Marked out of 1
Select one:
a. The cellular response is unchanged to subsequent stimuli.
b. The cellular response is attenuated due to a decrease in cellular receptor
number.

c. All the above

d. Cellular response is enhanced due to reduced receptor-ligand competition.

e. Cell hormone response is now bimodal; enhanced for a short time and
thereafter inactivated.

Question 73 The best prospects for treatment and cure of microbial diseases are always those
Complete unique factors of a pathogen's life cycle that can be altered without affecting the
Marked out of 1
survival of the host's own cells. In HIV, one such therapeutic target would be the
products of the pol gene, which codes for the reverse transcriptase unique to the
retroviral life cycle. If it were possible to ablate expression of the HIV pol gene, what
other aspect of the virus's life cycle would be directly altered?

Select one:
a. Nucleocapsid

b. Integration of proviral DNA

c. Production of viral mRNA


d. Transcription from proviral DNA

e. Viral maturation

The pol gene encodes several proteins, including the


virion “reverse transcriptase,” which synthesizes DNA by
using the genome RNA as a template, an integrase that
integrates the viral DNA into the cellular DNA, and a protease that
cleaves the various viral precursor proteins
Question 74 The absorption of glucose from the gut into intestinal mucosal cells is coupled to
Not answered Nat, K+-ATPase. In contrast, the movement of glucose from the intestinal epithelial
Marked out of 1
cells into the submucosal bloodstream occurs through passive transport. Given these
facts, which of the following statements can be TRUE at one time or another?

Select one:
a. Free glucose levels in the lumen of the intestine can never be higher than
levels in intestinal cells
b. Levels of plasma glucose are approximately equal to levels in the cytosol of
intestinal epithelial cells

c. Cytosolic levels of glucose in intestinal mucosal cells are regulated by levels of


glucose in skeletal muscle cells
d. Levels of glucose in the intestinal lumen are always higher than those in the
cytosol of intestinal epithelial cells

e. Plasma glucose levels are much higher than intestinal cell cytosolic levels of
glucose

Question 75 Enhancers are transcriptional regulatory sequences that function by enhancing the
Complete activity of Specific transcription factors bind to a specific DNA sequence, such as an
enhancer, and to RNA polymerase at a single promoter sequence. They enable
Marked out of1
Select one: the RNA polymerase to transcribe the specific gene for that enhancer more
efficiently.
a. transcription factors that bind to the promoter but not to RNA polymerase
b. RNA polymerase at a single promoter site
c. general transcriptional factors

d. spliceosomes
e. RNA polymerase to enable the enzyme to transcribe through the terminating
region ofa gene

Question 76 A segmented, ambisense genome is found in


Complete
Marked out of1 Select one:
a. Arenavirus

b. Reovirus

c. Parvovirus

d. Parainfluenza virus

e. Bunyavirus
Question 77 Tumor cells from a person with leukemia have been analyzed to determine which
Complete oncogene is involved in the transformation. After partial sequencing of the gene, the
Marked out of 1
predicted gene product is identified as a tyrosine kinase. Which of the following
proteins would most likely be encoded by an oncogene and exhibit tyrosine kinase
activity?

Select one:
a. Nuclear transcriptional activator

b. Growth factor receptor


c. Epidermal growth factor

d. Platelet-derived growth factor

e. Membrane-associated G protein

Question 78 Following statements regarding insulin are all correct except:


Complete
Marked out of 1 Select one:
a. Increases activity of glycogen synthase

b. Converts glycogen phosphorylase to inactive form

c. Induces glucokinase activity


d. Stimulates FA synthase activity

e. Inhibition of acetyl-CoA carboxylase

Question 79 In patients with primary hypothyroidism, which of the following laboratory test will
Complete be useful:
Marked out of 1
Select one:
a. Estimation of iodine

b. Estimation of T4

c. Estimation of T3

d. Estimation of TBG

e. Detection of auto-antibodies

The commonest cause of primary hypothyroidism is autoimmune


thyroid disease, which is associated with the presence of circulating
thyroid antibodies. Therefore, tests for thyroid autoantibodies,
including Thyroid peroxidase antibody (TPO-Ab) and Thyroglobulin
antibody (TG-Ab), may be useful to confirm autoimmunity as the
cause in a patient with primary hypothyroidism in which there is an
abnormality in the thyroid gland.
Question 80 What type of genetic material is created by repeated transpositional recombination
Complete events?
Marked out of 1
Select one:
a. Hfr chromosome

b. Insertion sequences

c. Chromosomal drug resistance genes


d. Multiple drug resistance plasmids
e. Genetic operon
MID SEMESTER 2
MAKE UP CAT 2022

Answered by Haron Sami

Compiled by Jitu 2022


& > Mycourses > Biochemistry > Topic 1 >» HBC200: MEDICAL BIOCHEMISTRY MAKE-UP MIDTERM

Started on Thursday, 14 April 2022, 2:00 PM


State Finished
Completed on Thursday, 14 Apri] 2022, 4:07 PM
Time taken 2 hours 7 mins

Question 1 Beta-lactam drugs bind to penicillin binding proteins to halt synthesis of


Complete peptidoglycan. Where are these proteins located?
Marked out of 1
Select one:
a. In the periplasmic space
b. In the peptidoglycan itself
c. In the capsule
d. In the cytoplasmic membrane
e. In the cytoplasm

Question 2 In a DNA molecule, guanosine nucleotide is held by the cytosine nucleotide by the
Complete number of hydrogen bonds:
Marked out of1
Select one:
a.5

b.4
3
d.1
e.2
Question 3 A patient stung by a bee is rushed into the emergency room with a variety of
Complete symptoms including increasing difficulty in breathing due to nasal and bronchial
Marked out of 1
constriction. Although your subsequent treatment is to block the effects of histamine
and other acute-phase reactants released by most cells, you must also block the
slow-reacting substance of anaphylaxis (SRS-A), which is the most potent constrictor
of the muscles enveloping the bronchial passages. An SRS-A is composed of which of
the following?

Select one: SRS-A is a mixture of LT-C4, LT-D4 and LT-E4


a. Thromboxanes

b. Complement

c. Interleukins

d. Prostaglandins

e. Leukotrienes

Question 4 Which of the following is transcribed during repression?


Complete
Marked out of 1 Select one:
a. Operator gene

b. Regulator gene

c. Structural gene

d. Promoter gene

e. None of the above

Question 5 The following statement is NOT true about DNA replication in prokaryotes.
Complete
Marked out of 1 Select one:
a, requires a primer
b. Adenine base pairs with uracil
c. Utilizes DNA polymerase

d. Both DNA strands serve as template

e. produces a double stranded DNA


Question 6 All prostaglandins have in common in addition to cyclopentane ring one double bond
Complete between positions:
Marked out of 1
Select one:
a.C13 and C14

b. C5 and C6
c. C9 and C10

d.C11 and C12


e. C7 and C8

Question 7 To design a vaccine against HIV infection, a logical goal would be to alter some native
Complete molecule or product of the virion in order to make it highly immunogenic. If you
Marked out of 1
wished to prevent the attachment of the virus to helper T lymphocytes, which
molecule or family of molecules might best be targeted?

Select one:
a. nucleocapsid protein

b. gp 120

eG pl7

d. p24

e. gp41

Question 8 The formation of a peptide bond during the elongation step of protein synthesis
Complete results in the splitting of how many high energy bonds?
Marked out of 1
Select one:
a.2

b.4
el

d.5
e.3

Question 9 Which of the following is not associated with Phase 2 reactions?


Complete
Marked out of 1 Select one:
a. Sulfation

b. Methylation

c. Conjugation with amino acids


d. Glucuronidation
e. Conjugation with oxidized glutathione
Question 10 Mobile genetic elements that code for antibiotic resistance genes in bacteria but are
Complete incapable of self-replication are
Marked out of 1 The spread of mobile genetic elements such as plasmids,
Select one: transposons, and integrons has greatly contributed to the
a. Transposons rapid dissemination of antimicrobial resistance among several
bacterial genera of human and veterinary importance.
b. Temperate RNA phages Antimicrobial resistance genes have been shown to
c. R factor accumulate on mobile elements, leading to a situation where
multidrug resistance phenotypes can be transferred to a
d. Virulent DNA phages susceptible recipient via a single genetic event.
e. Mesosomes

Question 11 Complete hydrolysis of nucleic acids will NOT yield:


Complete
Marked out of 1 Select one: omplete hydrolysis of nucleic acids by chemical or
a. Ribose
enzymatic means liberates the nucleic acid “building
blocks”: phosphate, sugar (ribose or deoxyribose), and
b. Phosphoric acid bases, in a 1:1:1 ratio
c. All of the above
d. Deoxyribose

e. Guanosine

Question 12 Glutamine involvement in nucleotide biosynthesis


Complete
Marked out of 1 Select one:
a. Donates1 Carbon and1 Nitrogen

b. Donates one N atom from its amide group

c. None of the other choices is true


d. Donates one N atom from its alpha-amino group

e. Donates one C atom from its alpha-carboxylic acid group

Question 13 Cyclic AMP shows decrease level in a cell-free cytoplasmic suspension due to addition
Complete of:
Marked out of 1
Select one: Cyclic AMP phosphodiesterase is an enzyme that
a. Aminophylline breaks down cyclic AMP, which can lead to a
b. Dibutyryl cyclic AMP
decrease in its levels in a cell-free cytoplasmic
suspension.
c. Caffeine
d. Adenylate cyclase
e. Cyclic AMP phosphodiesterase
Question 14 The following is not an end product of carbon dioxide fixation in Plasmodium
Complete
Marked out of 1 Select one:
The end products have been identified as alanine, aspartate,
a. Aspartate glutamate, and citrate with -ketoglutarate and oxaloacetate
as intermediate products
b. Alanine

c. glutamate

d. citrate

e. Proline

Question 15 Consider this pathway: epinephrine — G protein-coupled receptor — G protein >


Complete adenylyl cyclase — cAMP. Identify the second messenger.
Marked out of1
Select one:
a. adenylyl cyclase
b. cAMP

c. G protein-coupled receptor

d. G protein

e. GTP

Question 16 Protein phosphorylation is commonly involved with all of the following except
Complete
Marked out of 1 Select one:
a. activation of G protein-coupled receptors.

b. regulation of transcription by extracellular signaling molecules.

c. enzyme activation.

d. activation of receptor tyrosine kinases.

e. activation of protein kinase molecules.

Question 17 In the muscle, the purine nucleotide cycle (PNC) promotes the following except;
Complete
Marked out of 1 Select one:
a. Excretion of excess amino groups through the gut and kidney
b. Synthesis of fumarate
c. Synthesis of glutamine
d. Catabolism of branched chain amino acids (BCAAs)

e, Synthesis of X-ketoglutarate
Question 18 Cholera toxin produces diarrhoea due to:
Complete
Marked out of 1 Select one:
a. Stimulate cholinergic receptors

b. Inhibits the Na+ - K+ stimulated ATPase

c. By decreasing C-AMP level

d. By increasing synthesis of C-AMP


e. Prevents axonal ion transport

Question 19 The normal function of restriction endonucleases is to:


Complete
Marked out of 1 Select one:
a. Repair damaged DNA
b. Excise introns from hn-RNA

c. Protect bacteria from foreign DNA

d. Polymerise nucleotides to form RNA

e. Remove primer from Okazaki fragments

Question 20 A receptor for the SARS-CoV-2 Virus is


Complete
Marked out of 1 Select one:
a. CPG
b. Erb-B
c. ACE-2

d. TLR-9
e. CXCR4

Question 21 A bacterium is infected with an experimentally constructed bacteriophage composed


Complete of the T2 phage protein coat and T4 phage DNA. The new phages produced would
Marked out of 1
have

Select one:
a. T2 protein and T4 DNA.

b. T4 protein and T4 DNA.


c. a mixture of the DNA and proteins of both phages.
d. T2 protein and T2 DNA.
e. T4 protein and T2 DNA.
Question 22 Which of the following stimulates the production of progesterone by the corpus
Complete luteum?
Marked out of 1
Select one:
a. PRL

b. FSH
c. GH

d. TSH

e. LH

Question 23 The following statement is NOT true about hydroxyproline


Complete
Marked out of 1 Select one:
a. Its formation requires Fe2+ and molecular O2
b. It is an important constituent of mature collagen

c. Formation of hydroxyproline from proline is dependent on Vitamin C


d. It causes skin lesions and blood-vessel fragility

e. Lower amounts of hydroxyproline causes scurvy

Question 24 Many nucleotide analogues can be utilized in all the following EXCEPT;
Complete
Marked out of 1 Select one:
a. As antitumor agents

b. Psychoactive drug

c. All the other choices


d. As enzyme inhibitor

e. Inhibitors of replication

Question 25 Degeneracy of the genetic code denotes the existence of:


Complete
Marked out of 1 Select one:
a. Codons consisting of only two bases

b. Different protein synthesising systems in which a given triplet codes for


different amino acids
c. Codons that include one or more unusual bases

d. Multiple codons for a single amino acid


e. Base triplets that do not code for any amino acids
Question 26 The best prospects for treatment and cure of microbial diseases are always those
Complete unique factors of a pathogen's life cycle that can be altered without affecting the
Marked out of 1
survival of the host's own cells. In HIV, one such therapeutic target would be the
products of the pol gene, which codes for the reverse transcriptase unique to the
retroviral life cycle. If it were possible to ablate expression of the HIV pol gene, what
other aspect of the virus's life cycle would be directly altered?
The pol gene encodes several proteins,
Select one: including the virion “reverse transcriptase,”
a. Production of viral mRNA which synthesizes DNA by using the genome
b. Viral maturation RNA as a template, an integrase that
integrates the viral DNA into the cellular
c. Transcription from proviral DNA
DNA, and a protease that cleaves the various
d. Integration of proviral DNA viral precursor proteins
e. Nucleocapsid

Question 27 A second-year medical student is working in a laboratory studying gene regulation in


Complete a mouse model of hepatocellular carcinoma. He isolates nucleic acids from the cells
Marked out of 1
after exposure to a known carcinogen and has the sequences analyzed. He is
surprised to find that some of the nucleotides are pseudouridine and ribothymidine.
Which type of nucleic acid has the student likely isolated?
tRNAs often contain post-transcriptionally modified bases, including the
Select one: conversion of uridine to pseudouridine and ribothymidine. rRNAs are found
a. snRNP within ribosomes anddo not contain these modified bases. snRNPs (small
nuclear RNAs complexed with protein) are involved in splicing and also lack
b. hnRNA
ribothymidine and pseudouridine. hnRNA is processed to produce
c. rRNA mRNA, which also lacks these modified bases.
d. mRNA

e. tRNA

Question 28 What is NOT TRUE ofa steroid hormone?


Complete
Marked out of 1 Select one:
a. Steroid receptor complex enters nucleus

b. Binds to HRE of the gene and increases gene transcription.


c. Binds to specific receptor protein in cytosol

d. Stimulates adenyl cyclase activity

e. Synthesised from cholesterol


Question 29 NO, a neurotransmitter is made from the amino acid
Complete
Marked out of 1 Select one:
a. Proline

b. Serine
c. Arginine

d. Glycine
e. Threonine

Question 30 In de novo purine synthesis, it takes __ reaction steps for production of Inosine 5’
Complete Monophosphate
Marked out of 1
Select one:
ar7

b. 12

c.6

d. 11

e.B.9

Question 31 Which of the following is not true about energy systems in a skeletal muscle?
Complete
Marked out of 1 Select one:
a, The aerobic system is more reliable in endurance activities
b. The phosphagen system breaks down phosphocreatine to rebuild ATP

c. The aerobics system may break a range of fuels such as fats, carbohydrates
and proteins to rebuild ATP

d. The aerobic system is slower in ATP production but has lower capacity to
produce ATP

e. The anaerobic glycolysis system only breaks down carbohydrates to rebuild


ATP

Question 32 Steroid and peptide hormones typically have in common


Complete
Marked out of 1 Select one:
a. the building blocks from which they are synthesized.
b. their requirement for travel through the bloodstream.

c. their reliance on signal transduction in the cell.

d. the location of their receptors.

e. their solubility in cell membranes.


Question 33 What is an Interferon?
Complete
Marked out of 1 Select one:
a. ls a synthetic antiviral agent

b. Is an adaptive immune response

c. Is a bacterial product
d. Requires expression of cellular genes

e. Is virus specific

Question 34 What is the term applied to a segment of a bacterial chromosome where genes for
Complete the enzymes of a particular metabolic pathway are clustered and subject to
Marked out of 1
coordinate control?

Select one:
a. Operon

b. Origin
c. Operator

d. Promoter

e. Terminal controller

Question 35 Following specialized transduction, if any of the bacterial genes transferred in are to
Complete be stabilized, what process must occur?
Marked out of 1
Select one:
a. Transduction

b. Specialized transduction

c. Conjugation

d. Site-specific recombination

e. Homologous recombination

Question 36 The nonsteroidal anti-inflammatory drugs, as a class, are anti-inflammatory primarily


Complete through their ability to inhibit
Marked out of1
Select one:
a. thromboxane synthesis.

b. cyclooxygenase (COX) 1.

c. COX-3.
d. leukotriene synthesis.

e. COX-2.
Question 37 A segmented, ambisense genome is found in
Complete
Marked out of 1 Select one:
a. Reovirus

b. Parainfluenza virus

c. Arenavirus

d. Bunyavirus
e. Parvovirus

Question 38 A bacterial structure involved in adherence is


Complete
Marked out of 1 Select one:
a. Lipopolysaccharide

b. O-specific side-chain

c. Pili
d. Capsule

e. Teichoic acid

Question 39 A 43-year-old female has developed breast cancer. Her mother and one maternal
Complete aunt had breast cancer and her maternal grandmother had ovarian cancer. Which of
Marked Gcator4 the following best describes the mechanism behind this inherited problem?

Select one:
a. Atumor suppressor leading to loss of apoptosis

b. An oncogene leading to a constitutively active MAP kinase pathway

c. An oncogene leading to loss of apoptosis

d. Atumor suppressor leading to an inability to repair DNA


e. All the responses are correct

Question 40 Lesch-Nyhan syndrome is due to deficiency in the enzyme


Complete
Marked out of1 Select one:
a. Adenine phosphoribosyl transferase

b. Hypoxanthine-Guanine phosphoribosyl transferase

c. Adenosine deaminase
d. 5’ Nucleotidase
e. Purine nucleoside phosphorylase

39. certain inherited gene changes can greatly increase the risk for developing certain
cancers and are linked to many of the cancers that run in some families. For instance, the
BRCA genes (BRCA1 and BRCA2) are tumor suppressor genes. When one of these genes
changes, it no longer suppresses abnormal cell growth, and cancer is more likely to develop
Question 41 Which pyrimidine nucleotide acts as the high energy intermediate?
Complete
Marked out of 1 Select one:
a. UTP

b. ATP

c. UDPG

d. CMP

e. Cyclic AMP

Question 42 HTLVs
Human T-lymphotropic virus type 1 (HTLV-1) is associated with adult
Complete T-cell leukemia; HTLV-2 is implicated in human hairy cell leukemia.
Marked out of 1 Select one:
a. Are DNA viruses

b. Are defective RNA tumor viruses


c. Carry tyrosine protein kinase oncogenes

d. Synthesize early proteins that interact with p53Rb


e. Are associated with leukemias

Question 43 Of the receptors listed below, which can directly conduct a flow of ions across the
Complete plasma membrane when bound to their cognate ligand?
Marked out of1
Select one:
a. Steroid hormone receptors

b. Ligand-gated channels
c. G-protein-coupled receptors (GPCRs)

d. G-protein gamma a subunit.

e. Receptor tyrosine kinases (RTKs)

Question 44 In purine biosynthesis, which compound donates an N and 2C atoms?


Complete
Marked out of 1 Select one:
a. Aspartate

b. Glycine

c. Arginine

d. Ornithine

e. Glutamine
Question 45 Which of the following statements correctly describes insulin?
Complete
Marked out of 1 Select one:
a. Its action is antagonistic to that of glucagon

b. It is converted from proinsulin to insulin primarily following secretion from B-


cells
c. It does not have a prohormone form

d. It is a small polypeptide composed of a single chain bridged by disulfide groups

e. It is an anabolic signal to cells that glucose is scarce

Question 46 A functional virogene is missing in * has high oncogenic potential


Complete * is a defective virus: because it lacks functional
Marked out of 1 Select one:
virogene.
a. Harvey sarcoma virus *viruses with high oncogenic potential lack
b. Hepatitis B virus functional virogene.
Harvey sarcoma virus, like most viruses with high
c, Rous sarcoma virus
oncogenic potential, is a defective virus that lacks
d. Human T lymphotrophic virus at least one functional virogene
e. Mouse mammary tumor virus

Question 47 In the hydrogenosome, under anaerobic conditions serves as the terminal


Complete electron acceptor
Marked out of1
Select one:
a. H+

b. CO2

c. O2

d. Hydrogen
e. Acetate

Question 48 Co-translational targeting occurs in proteins destined to


Complete
Marked out of 1 Select one:
a. Endoplasmicreticulum (ER)

b. Nucleus

c. Mitochondria

d. Peroxisomes

e. Chloroplasts
Question 49 What is the term applied to the complete collection of proteins present in a particular
Complete cell type?
Marked out of 1
Select one:
a. Proteome

b. Translatome

c. Genome

d. Transcriptome
e. Peptide collection

Question 50 Linear, single-stranded DNA is the genetic material of


Complete
Marked out of 1 Select one:
a. Calciviruses

b. Retroviruses

c. Flaviviruses

d. Papillomaviruses

e. Parvoviruses

Question 51 Which of the following compounds is not a precursor of de novo pyrimidine synthesis
Complete in parasitic protozoa?
Marked out of 1
Select one:
a. Aspartate

b. lactate

c. Carbon dioxide

d. Glutamine

e. Formate

Question 52 Which of the following enzymes of the glycolytic pathway in Fasciola hepatica is
Complete inhibited by clorsulon (MK- 401)?
Marked out of 1
Select one:
a. Hexokinase

b. Glyceraldehyde-3-phosphate dehydrogenase

c. Phosphofructokinase 1

d. Pyruvate kinase
e. Phosphoglycerate mutase
Question 53 Nucleotides perform the following actions
Complete
Marked out of 1 Select one:
a. All the other choices are true

b. Electron acceptors in cellular respiration

c. Metabolic regulators
d. Co-enzymes

e. Enzyme inhibitors

Question 54 A 23-year-old woman, who has had numerous episodes of unprotected intercourse
Complete since the age of 12 years, visits a gynecologist for her first Pap smear. The results
ariedlonnit’d return positive for atypical cells, indicative of human papilloma virus (HPV) infection.
Which of the following correctly describes the effects of HPV on cell growth?

Select one:
a. The viral E6 protein degrades cellular p53.

b. The virus causes insertional inactivation of critical genes.


c. The viral E7 protein degrades cellular p53.
d. The viral LMP-1 protein prevents the expression of cellular bcl-2.

e. The viral E6 protein perturbs the normal function of cellular Rb.

Question 55 Common DNA damage caused by UV light is


Complete
Marked out of 1 Select one:
a. Winding of the DNA duplex

b. A-C mismatch

c. Formation of thymine-thymine dimers

d. Deamination of cytosine to uracil

e. Nicking of the DNA strands

Question 56 The following are haemoglobin catabolism products in humans EXCEPT;


Complete
Marked out of1 Select one:
a. Biliverdin

b. B-hematin

c. haem

d. Bilirubin

e. Bilirubin diglucuronide
Question 57 RNA is converted to complementary DNA using the enzyme
Complete
Marked out of 1 Select one:
a. Primase

b. DNA gyrase

c. DNA polymerase

d. RNA polymerase
e. Reverse transcriptase

Question 58 What happens to protein kinase A (PKA) following the binding of cAMP?
Complete
Marked out of 1 Select one:
a. The regulatory subunits of PKA dissociate, thereby activating the catalytic
subunits.

b. Phosphodiesterase binds to the catalytic subunits, which results in enzyme


inactivation.

c. The inhibitory regulatory subunits dissociate from the catalytic subunits,


completely inactivating the enzyme.

d. The stimulatory regulatory subunits dissociate from the catalytic subunits,


inhibiting the enzyme.

e. PKA catalytic subunits then bind to two regulatory subunits, thereby activating
the catalytic subunits.

Question 59 The expression of the lac operon


Complete
Marked out of 1 Select one:
a. Necessitates the finding of RNA polymerase followed by transcription

b. Occurs in Yeast

c. Must be initiated by the binding of an inducer protein

d. Does not involve the expression of structural genes

e. Involves the release of allolactose from a repressor protein

Question 60 An example of antagonistic hormones controlling homeostasis is


Complete
Marked out of 1 Select one:
a. progestins and estrogens in sexual differentiation.

b. insulin and glucagon in glucose metabolism.

c. epinephrine and norepinephrine in fight-or-flight responses.

d. oxytocin and prolactin in milk production.

e. thyroxine and parathyroid hormone in calcium balance.


Question 61 The melting temperature of DNA (Tm) is:
Complete he higher the G-C content, the stronger the
Select one: hydrogen bonds between the base pairs, and
Marked out of 1
the higher the melting temperature. Conversely,
a. Directly proportional to G-C content
the lower the G-C content and the higher the
b. None of the above A-T content, the weaker the hydrogen bonds
between the base pairs, and the lower the
c. Not related to base composition at all melting temperature
d. Directly proportional to the length of DNA

e. Directly proportional to A-T content

Question 62 All of the following statements regarding hormones are TRUE except:
Complete
Marked out of 1 Select one:
a. May affect the rate of enzymatic catalysis

b. May act by regulating pre-existing processes

c. May alter the permeability of plasma membranes

d. May act as enzymes or coenzymes


e. May influence the synthesis of enzymes

Question 63 Tetrahydrobiopterin (GH4) is a reductant in the metabolism of the following except;


Complete
Marked out of 1 Select one:
a. Phenylalanine catabolism

b. Nitric oxide (NO) synthesis

c. Melanin synthesis
d. Catecholamine synthesis

e, Serotonin synthesis

Question 64 Within a cell, the amount of protein made using a given MRNA molecule depends
Complete partly on
Marked out of 1
Select one:
a, the presence of certain transcription factors.
b. the degree of DNA methylation.
c. the types of ribosomes present in the cytoplasm.

d. the rate at which the mRNA is degraded.

e. the number of introns present in the mRNA.


Question 65 If the percentage concentration of thymine in DNA is 40%, the concentration of
Complete cytosine will be:
Marked out of 1
Select one:
a. 40%

b. 10%

c. 20%

d. 50%

e. 30%

Question 66 The end of the rising phase of an action potential occurs when the;
Complete
Marked out of 1 Select one:
a. Sodium channels close

b. Chloride channels open

c. Potassium channels open

d. Potassium channels close and Sodium channels close

e. Potassium channels close

Question 67 Erythromycin is the antibiotic of choice when treating respiratory tract infections in
Complete whooping cough, and mycoplasma based pneumonia because of its ability to inhibit
Marked out of 1
protein synthesis in certain bacteria by doing which of the following?

Select one:
a. Acting as an analogue of mRNA

b. Mimicking mRNA binding


c. Inhibiting translocation by binding to 50S ribosomal subunits
d. Causing premature chain termination
e. Inhibiting initiation

Question 68 Proto-oncogenes can change into oncogenes that cause cancer. Which of the
Complete following best explains the presence of these potential time bombs in eukaryotic
Marked out of1
cells?

Select one:
a. Cells produce proto-oncogenes as they age.

b. Proto-oncogenes are mutant versions of normal genes.

c. Proto-oncogenes first arose from viral infections.

d. Proto-oncogenes are genetic “junk.”

e. Proto-oncogenes normally help regulate cell division.


Question 69 The subunits of the heterotrimeric G-proteins are called the __, _,and__ subunits.
Complete
Marked out of 1 Select one:
a.a,y, and 6

b. y, 6, and n

c. a, B, and x
d. a, B, andy

e.a, B, and &

Question 70 A 37-year-old immigrant from Tanzania develops fevers, night sweats, weight loss,
Complete and a blood-tinged cough. He present to the emergency room, where an infectious
ariedlonnit’d disease doctor is consulted and immediately prescribes a multidrug regimen that
includes rifampin. Rifampin inhibits which one of the following types of enzymes?

Select one:
a. None of the above

b. Reverse transcriptase
c. RNA-dependent DNA polymerase

d. DNA-dependent RNA polymerase

e. DNA-dependent DNA polymerase

Question 71 Red muscle fibers are characterized by the following except;


Complete
Marked out of 1 Select one:
a. Contains more mitochondria

b. Glycogen stores are low

c. Contains much blood capillaries

d. Contains high concentrations of myoglobin

e. Depend on anaerobic oxidation

Question 72 Which bacterial gene transfer process would be inhibited by free extracellular
Complete exonucleases?
Marked out of1 In transformation, the DNA is extracellular before it is
Select one: picked up by the competent cells; during this period, the
a. Transformation DNA is subject to the extracellular exonucleases.
Because the DNA in generalized and specialized
b. Transduction
transductions ist protected extracellularly by the virus
c. Transposition capsid, it is not subject to extracellular exonucleases, In
d. Conjugation
conjunction, the DNA is never outside of a cell.
Transposition is a mechanism of inserting a transposon
e. Specialized transduction into another molecule of DNA and has no extracellular
transport mechanism associated with it.
Question 73 A key difference between purine and pyrimidine synthesis de novo is all the following
Complete EXCEPT;
Aspartate is required for both purine and pyrimidine
Marked out of 1
Select one:
synthesis de novo. It is not a key difference between the
two pathways.
a. Amino acid aspartate is required

b. Energy requirement in total number of ATPs

c. The amino acids involved

d. All reactions for pyrimidine biosynthesis are cytosolic


e. The number of reaction steps involved

Question 74 Which of the following is a not protozoan parasite


Complete
Marked out of 1 Select one:
a. Fasciola hepatica

b. Trichomonas vaginalis

c. Leishmania donovani

d. Trypanosoma brucei

e. Plasmodium falciparum

Question 75 Which of the following statements is NOT true about transcription in prokaryotes.
Complete
Marked out of 1 Select one:
a, primer is not required
b. Requires single strand of DNA as template
c. Adenine base pairs with uracil

d. produces a double stranded DNA

e. Utilizes RNA polymerase

Question 76 In Trichomonas vaginalis oxidative decarboxylation of Malate to Pyruvate is catalyzed


Complete by
Marked out of 1
Select one:
a. Alcohol dehydrogenase

b. Pyruvate dehydrogenase

c. Pyruvate decarboxylase

d. Pyruvate phosphate dikinase

e. Malic enzyme
Question 77 Cytochrome P450 (CYP) enzymes are except;
Complete
Marked out of 1 Select one:
a. They comprise of both oxidase and reductase functions

b. Carry out hydroxylation reactions

c. Complex with carbon monoxide and absorb light maximally at 450mm


d. Complex with carbon dioxide and absorb light maximally at 450mm
e. Metalloproteins

Question 78 The following is FALSE about mitochondria! proteins


Complete
Marked out of 1 Select one:
a. Mitochondrial Proteins are synthesized in the cytosol

b. The targeting sequence is removed in the mitochondrial matrix

c. Their targeting sequence is located at the N-terminus

d. Most mitochondrial proteins are encoded by nuclear DNA


e. Most mitochondrial proteins are encoded by mitochondral DNA

Question 79 A 23-year-old man presents to his family physician with a painless swelling of his
Complete testicles. An ultrasound is suspicious for a neoplasm, and a biopsy confirms the
Marked oukur4 presence of cancer. He is referred to an oncologist, who begins treatment with the
topoisomerase inhibitor etoposide. The normal function of this enzyme is to do
which of the following? Topoisomerase creates
double-stranded breaks ahead
of the replication fork to relieve
Select one:
the supercoiling induced by the
a. Break and rejoin the DNA helix during replication action of helicase, which
unwinds the DNA helix during
b. Prevent the single strands of DNA from reannealing during replication
replication. DNA polymerase
c. Synthesize RNA primers for DNA polymerase and ligase, along with specific
endonucleases, are important in
d. Unwind the DNA helix during replication
repairing DNA damage.
e. Repair nuclear DNA in the event of DNA damage Single-stranded binding
proteins prevent the separated
strands from reannealing during
replication. Primase is required
Question 80 Sugar transport into bacteria is frequently facilitated by to synthesize primers for DNA
Complete
replication by DNA polymerase.
Marked out of1 Select one:
a. Penicillin-binding proteins

b. Phosphotransferases

c, Oxidases

d. Neuraminidases
e. Acetyltransferase
Question 81 Which of the following is noted in Cushing syndrome, a disease of the adrenal cortex?
Complete
Marked out of 1 Select one:
a. Excessive production of epinephrine

b. Decreased production of cortisol

c. Decreased production of epinephrine


d. Excessive production of vasopressin
e. Excessive production of cortisol

Question 82 Which of the following is not a Phase 1 reaction?


Complete
Marked out of 1 Select one:
a. Hydroxylation

b. Hydrolysis
c. Oxidation

d. Acetylation

e. Deamination

Question 83 Posttranscriptional modification includes the following EXCEPT


Complete
Marked out of 1 Select one:
a. RNA editing

b. mRNA splicing
c. Protein phosphorylation

d. Capping at the 5’ end


e. Polyadenylation

Question 84 The region of the Lac operon which must be free (unbound) for structural gene
Complete transcription to occur is:
Marked out of 1
Select one:
a. The ‘a’ gene

b. The ‘i’ gene

c. The ‘z' gene

d. The promoter site

e. The operator locus


Question 85 The removal of introns and subsequent self-splicing of adjacent exons occurs in
Complete some portions of primary ribosomal RNA transcripts. The splicing of introns in
Marked out of 1
messenger RNA precursors is which of the following?

Select one:
a. RNA catalyzed in the absence of protein
b. Self-splicing

c. Regulated by RNA helicase

d. Controlled by RNA polymerase

e. Carried out by spliceosomes

Question 86 COSMIC database can be used to access information on;


Complete

Marked out of 1 Select one:


a. Metabolic pathways of parasites

b. Genomic data in humans


c. Drugs used in cancer treatment

d. Nucleotide sequences of microorganisms

e. Somatic mutations and cancer type in humans

Question 87 Zellweger syndrome


Complete
Marked out of 1 Select one:
a. Is due to loss of mitochondrial functions

b. Doesn't cause any developmental defects


c. Is not a genetic disorder

d. Is not associated with protein targeting


e. ls due to mutation in any of the PEX genes

Question 88 What virus is noted for such a high incidence of genetic drift that more than one
Complete antigenic variant can be isolated from most infected individuals who have high viral
Marked out of1
titers?

Select one:
a. Herpes virus

b. Influenza virus

c. Human immunodeficiency virus (HIV)

d. Poliovirus

e. Adenovirus
Question 89 All the enzyme listed below are regulated in de novo purine synthesis EXCEPT;
Complete
Marked out of 1 Select one:
a. Adenylsuccinate synthetase

b. PRPP synthetase

c. PRPP amidotransferase
d. Glycinamide Synthetase
e. IMP dehydrogenase

Question 90 UniProtKB/TrEMBL;
Complete
Marked out of 1 Select one:
a. ls part of the Universal Protein Resource

b. Is a protein sequence database


c. Contains translations of coding sequences (CDS) from EMBL(ENA)

d. ls a computationally-annotated database
e. All the other choices are correct

Question 91 Influenza virus results in more than 500,000 deaths worldwide annually. Influenza A
Complete contains an eight-piece segmented negative-sense RNA genome. Two important
Marked out of 1 proteins encoded by this genome are HA (hemagglutinin) and NA (neuraminidase).
The HA protein directly binds to which host cell epithelial component?

Select one:
Sialic acid is a type of sugar molecule that
is found on the surface of many host cell
a. Sialic acid
types, including epithelial cells in the
b. Serine-threonine kinase receptors respiratory tract. Influenza virus uses the
c. Uronic acid subgroups
HA protein to bind to sialic acid on the host
cell surface, which facilitates viral entry
d. Chemokine receptors into the host cell and infection.
e. Cytokine receptors

Question 92 Peroxisome targeting sequence 1 (PTS1)


Complete
Marked out of1 Select one:
a. Occurs only on unfolded proteins

b. Is important in the sorting of proteins to peroxisomes

c. Is the only targeting sequence for proteins destined to peroxisomes

d. Doesn't enter the peroxisome matrix

e. Doesn't bind specific receptors


Question 93 Which of the following agents, if introduced into a growing culture of bacteria, would
Complete halt growth but, if then removed, would allow growth to resume?
Marked out of 1
Select one: A bacteriostatic agent inhibits the growth and reproduction of
a. Sterilizing Agent bacteria but does not kill them. If the agent is removed, the bacteria
can resume growth
b. Disinfectant

c. Bacteriostat

d. Antiseptic
e. Bacteriocide

r
Question 94 Gene transcription rates and MRNA levels were determined for an enzyme that is
Complete induced by glucocorticoids. Compared with untreated levels, glucocorticoid
Marked out of 1
treatment caused a 10-fold increase in the gene transcription rate and a 20-fold
increase in both mRNA levels and enzyme activity. These data indicate that a primary
effect of glucocorticoid in this assay is to decrease which of the following?

Select one:
a. The activity of RNA polymerase III

b. The rate of binding of ribosomes to mRNA

c. The activity of RNA polymerase II

d. The ability of nucleases to act on MRNA

e. The rate of mRNA translation

Question 95 A 13-year-old boy has developed polydipsia, polyuria, polyphagia, and weight loss
Complete over the past few weeks. He was brought to the emergency room by his parents
Marked out of 1
because he woke up this morning very lethargic. His blood glucose was found to be
600 mg/dL, and he was immediately placed on an insulin drip. Insulin works primarily
by which one of the following mechanisms?

Select one:
a. Causing phosphorylation of tyrosine residues

b. Activating caspases

c. Producing cGMP
d. Activating adenylate cyclase

e. Binding to an intracellular receptor

Insulin works primarily by binding to its receptor on the cell surface, which activates the
receptor’s intrinsic tyrosine kinase activity. This leads to the phosphorylation of tyrosine
residues on the receptor and on intracellular signaling proteins, initiating a cascade of events
that results in the cellular response to insulin.
Question 96 N-acetylmuramic acid is located in
Complete
Marked out of 1 Select one:
a. Teichoic acid

b. Peptidoglycan

c. Lipoprotein

d. Outer membrane
e. Lipopolysaccharide

Question 97 A viral disease in which antiviral antibodies are thought to have an “enhancing” effect
Complete so that reinfection with the virus causes a more serious disease is
Marked out of1
Select one: ADE has been observed with dengue virus 2, Zika virus 3,
a. Rubella virus Ebola virus 4 and, importantly in the context of COVID
b. Hanta virus

c. Dengue virus

d. Coxsackie A virus

e. West Nile virus

Question 98 Growth factors are local regulators that


Complete Growth factors are local regulators that bind to specific cell-surface receptors
Select one:
and stimulate growth and development of target cells. They play important roles
Marked out of 1
in processes such as wound healing, tissue repair, and embryonic development
a. are found on the surface of cancer cells and stimulate abnormal cell division.

b. bind to cell-surface receptors and stimulate growth and development of target


cells.

c. are modified fatty acids that stimulate bone and cartilage growth.
d. are produced by the anterior pituitary.
e. convey messages between nerve cells.

Question 99 Which of the following pathway would not occur in the in the liver mitochondria?
Complete
Marked out of 1 Select one:
a. FA metabolism

b. FA synthesis
c. FA oxidation

d. Urea synthesis

e, Ketone body (KB) synthesis


Leukotrienes are a family of lipid mediators derived from arachidonic acid through the action of the lipoxygenase
enzyme system. They are involved in various physiological and pathological processes such as inflammation and
allergic reactions. SRS-A (slow-reacting substance of anaphylaxis) is a mixture of leukotrienes LTC4, LTD4, and
LTE4. Leukotrienes do not contain a cyclopentane ring; instead, they have a conjugated triene structure. Some
leukotrienes can stimulate chemotaxis and chemokinesis of neutrophils.
Question 100 The leucotrienes are characterised by all of the following except:
Complete
Marked out of 1 Select one:
a. Enzyme responsible for conversion of arachidonate to leucotrienes is
lipoxygenase system
b. SRS-A is a mixture of LT-C4, LT-D4 and LT-E4

c. All of them contain cyclopentane ring

d. Are formed from arachidonate

e. One of LTs stimulates chemotaxis and chemokinesis of neutrophils

Question 101 Binding of a signaling molecule to which type of receptor leads directly to a change in
Complete the distribution of ions on opposite sides of the membrane?
Marked out of 1
Select one:
a. ligand-gated ion channel

b. receptor tyrosine kinase

c. phosphorylated receptor tyrosine kinase dimer

d. intracellular receptor
e. G protein-coupled receptor

Question 102 If the cytosine content of double stranded DNA is 20% of the total bases, the adenine
Complete content will be:
Marked out of1
Select one:
a. 30%

b. 50%

c. 10%

d. 40%

e, 20%

Question 103 The first viral-induced defense mechanism in a non-immune individual is the:
Complete
Marked out of 1 Select one:
a. Production of complement system
b. Synthesis of lymphokines

c, Production of interferon
d. Generation of cytotoxic T lymphocytes

e. Synthesis of neutralizing antibodies


Question 104 Basic Local Alignment Search Tool (BLAST)
Complete
Marked out of 1 Select one:
a. ls a rarely used bioinformatics protocol

b. Is the only homology search algorithm

c. Is a family of programs that carry out different classes of homology search


d. Uses only DNA sequence
e. Uses only protein sequence

Question 105 What bacterial gene transfer process is most sensitive to extracellular nucleases?
Complete No answer In transformation, the DNA is extracellular before it
Marked out of 1 Select one: is picked up by the competent cells; during this
a. Transduction period, the DNA is subject to the extracellular
b. Homologous recombination exonucleases. Because the DNA in generalized
and specialized transductions ist protected
c. Site-specific recombination
extracellularly by the virus capsid, it is not subject to
d. Conjugation extracellular exonucleases, In conjunction, the DNA
is never outside of a cell. Transposition is a
e. Specialized transduction
mechanism of inserting a transposon into another
molecule of DNA and has no extracellular transport
mechanism associated with it.
Question 106 What features of the nuclear receptor superfamily suggest that these proteins have
Complete evolved from a common ancestor?
Marked out of1
Select one:
a. They all bind DNA.

b. They all contain regions of high amino acid sequence similarity/identity.

c. They all function within the nucleus.

d. They are all subject to regulatory phosphorylation.

e. They all bind the same ligand with high affinity.

-
Question 107 Which of the following is the most likely explanation for hypothyroidism in a patient
Complete whose iodine level is normal?
Marked out of 1
Select one:
a. hyposecretion of TSH

b. greater production of T3 than of T4


c. hypersecretion of TSH
d. hypersecretion of MSH
e. a decrease in the thyroid secretion of calcitonin
Question 108 RNA polymerase || of eukaryotes
Complete
Marked out of 1 Select one:
a. Has high sensitivity to a-amanitin

b. Synthesizes all types of RNA

c. Synthesizes transfer RNA


d. Does not require transcription factors
e. Produces 45S rRNA

Question 109 Which of the following defects in adrenal cortex is responsible for lack of
Complete glucocorticoids and mineralocorticoids?
Marked out of1
Select one:
a. Estrone deficiency

b. C2 hydroxylase deficiency

c. Testosterone deficiency

d. 17-a-OH progesterone deficiency

e. Androstenedione deficiency

Question 110 The following statement is true about Sulfadoxine


Complete
Marked out of 1 Select one:
a. Sulfadoxine inhibits the formation of toxic reactive oxygen species

b. Sulfadoxine inhibits the formation of B-hematin

c. Resistance to Sulfadoxine is mediated by removal or inactivation of the drug


d. Sulfadoxine inhibits dihydropteroate synthase in de novo synthesis of
tetrahydrofolate

e. Mechanism of resistance involves ABC type transporters


Question 111 Your patient, who has been hospitalized in a coma for 2 weeks, has developed
Complete erythema around his catheter. You order the catheter be pulled and the tip cultured
Marked out of 1
and that a new line be started on the opposite side of the body. The report comes
back that it is Staphylococcus epidermidis, which does not surprise you, as it is noted
for the production of a loose network of polysaccharide sticking to catheters. What is
this layer on the cell called?

Select one:
a. Glycocalyx

b. Teichoic acids

c. Peptidoglycan

d. Biofilm

e. Pili

Question 112 Literature on genetic disorders can be searched in;


Complete
Marked out of 1 Select one:
a. European Nucleotide Archive (ENA)

b. UniprotKB
OMIM (Online Mendelian Inheritance in Man) is a comprehensive
database of human genes and genetic disorders. It contains
c. PROSITE information on all known Mendelian disorders and over 15,000 genes
d. GenBank
e. OMIM

Question 113 Nonsense codons bring about:


Complete
Marked out of 1 Select one:
a. Elongation of polypeptide chains
b. Initiation of protein synthesis
c. Amino acid activation

d. Termination of protein synthesis


e. Post-translational modification of proteins

Question 114 A-B subunit structure as it relates to bacterial pathogenesis refers to the structure of
Complete
Marked out of 1 Select one: Many bacterial exotoxins have an A-B
a. Penicillin-binding proteins subunit structure in which the B subunit is
b. Nucleic acid inhibitor antibiotics
involved in binding and the A subunit
possesses biological activity inside the
c. Gram-negative bacteria endotoxin affected cell.
d. Bacterial exotoxins

e. Resistance transfer factors


Question 115 Translocase is an enzyme required in the process of:
Complete
Marked out of 1 Select one:
a. DNA replication

b. RNA synthesis

c. Termination of protein synthesis


d. Elongation of peptides

e. Initiation of protein synthesis

Question 116 Synthesis of RNA molecule is terminated by a signal which is recognised by:
Complete
Marked out of 1 Select one:
a. None of the above

b. a-factor

c. 6-factor

d. p-(rho)-factor

e. B-factor

Question 117 A nasopharyngeal swab obtained from a 4-month-old infant with rhinitis and
Complete paroxysmal coughing tested positive upon culture for Bordetella pertussis. He was
Marked out of1 admitted to the hospital for therapy with an antibiotic that inhibits the translocation
of the 70S ribosomes on the mRNA. This patient was most likely treated with

Select one:
a. levofloxacin

b. tetracycline

c. chloramphenicol

d. rifampicin
e. erythromycin

Question 118 Acetylcholinesterase has an essential residue in its active site


Complete
Marked out of1 Select one:
a. Carboxyl

b. Methy!
c. Hydroxyl

d. Acetyl
e. Sulfhydryl
Question 119 Enteroviruses differ from rhinoviruses mainly in their:
Complete
Marked out of 1 Select one:
a. Capsid shape

b. Strandedness
c. Tropism

d. Ability to survive acidic conditions


e. Type of nucleic acid

Question 120 T-G mismatch in humans is mostly caused by


Complete
Marked out of 1 Select one:
a. A large proportion of T and G bases in the cells

b. Lack of repair mechanism in the cells

c. The need to have more mutants

d. Lack of enzymes which recognize the mismatch


e. Deamination of cytosine to Uracil or 5-methyl cytosine to thymine

Question 121 A 73-year-old woman is admitted to the intensive care unit for septic shock from a
Complete urinary tract infection. The critical care fellow is concerned she may not have an
Marked Gcator4 appropriate stress response and orders a cosyntropin test. Which hormone does this
test evaluate? The cosyntropin test evaluates the function
of the adrenal cortex by measuring the level
Select one: of cortisol in the blood. Cosyntropin is a
a. Corticotropin-releasing hormone (CRH)synthetic form of adrenocorticotropic
hormone (ACTH) that stimulates the
b. Vasopressin adrenal glands to produce cortisol. During
c. Cortisol the test, a baseline blood sample is taken to
measure the initial cortisol level. Then, an
d. Adrenocorticotropic hormone (ACTH)
injection of cosyntropin is given, and
e. Oxytocin additional blood samples are taken at
specific intervals to measure the cortisol
response
Question 122 Indicate the term generally applied to the extracellular messenger molecules that
Complete bind to transmembrane receptor proteins.
Marked out of1
Select one:
a. Scatchard curve

b. Competitive inhibitor

c. Substrate
d. Ligand
e. Key
Question 123 Which of the following compounds is not a DNA mutagen
Complete
Marked out of 1 Select one:
a. methyl methanesulfonate

b. Ethidium bromide

c. proflavin

d. Thymine
e. N-methyl-N'-nitro-N-nitrosoguanidine

Question 124 Thiocyanates and perchlorates interfere with:


Complete
Marked out of 1 Select one:
a. Trapping of I2 by thyroid

b. lodination of tyrosine
c. All of the above

d. Release of T3

e. Release of T4

Question 125 At-RNA molecule has the following:


Complete
Marked out of 1 Select one:
a. None of the above
b. An anticodon arm

c. A clover-leaf structure

d. All of the above

e. Aloop containing minor base dihydrouracil

Question 126 t-RNA specifically reacts with:


Complete
Marked out of 1 Select one:
a. Ribosomal DNA

b. None of the above


c. Specific aminoacyl adenylates

d. Ribosomal RNA

e. Nuclear DNA
Question 127 Which of the following is required for certain types of eukaryotic protein synthesis
Complete but NOT for prokaryotic protein synthesis?
Marked out of 1
Select one:
a. GTP

b. Peptidyl transferase

c. Signal recognition particle

d. Messenger RNA
e. Ribosomal RNA

Question 128 The ability ofa cell to bind DNA to its surface and import it is required for which
Complete genetic process?
Marked out of 1
Select one:
a. Conjugation

b. Specialized transduction

c. Transformation

d. Site-specific recombination

e. Homologous recombination

Question 129 A severe birth defect syndrome called retinoic acid embryopathy is caused by
Complete medications such as Accutane when taken by pregnant women for treatment of
Marked out of 1
acne. Retinoic acid and steroid hormones are group | signals that cross the cell
membrane to interact with cytosolic or nuclear receptors. Which of the structural
domains of such receptors would interact with retinoic acid?

Select one:
a, Antirepressor domains

b. DNA-binding domains

c. Transcription-activating domains
d. Ligand-binding domains
e. Response elements
Question 130 All of the following hormones use cyclic-AMP as a second messenger except:
Complete
Marked out of 1 Select one:
a. LH

b. Estrogen

c. Glucagon

d. FSH
e. Epinephrine

Question 131 Which of the following is not a manifestation of insulin deficiency?


Complete
Marked out of 1 Select one:
a. Polydipsia

b. Keto acidosis
c. Glycosuria

d. Metabolic acidosis
e. Polyuria

Question 132 The following apply to the compound 6-mercaptopurine except;


Complete
Marked out of 1 Select one:
a. Can easily pass through plasma membrane

b. Is like to interfere with cellular DNA replication

c. It mimics Adenine
d. Is naturally occurring

e. Is a powerful anticancer agent

Question 133 O-antigens are found in:


Complete
Marked out of 1 Select one:
a. Peptidoglycan

b. Capsule

c. Lipopolysaccharide

d. Teichoic acid

e. Lipoprotein
Question 134 Polysomes do NOT contain:
Complete
Marked out of 1 Select one:
a.m-RNA

b. r-RNA
c. Protein

d. t-RNA
e.DNA

Question 135 Viruses whose genomes have a messenger (positive-sense) polarity are
Complete
Marked out of 1 Select one:
a. Polioviruses

b. Adenoviruses
c. Ebola Viruses

d. Papoviruses

e. Paramyxoviruses

Question 136 Most eukaryotic transcription factors contain at least two domains, each of which
Complete mediate different aspects of transcription factor function. Which of the following are
Marked Gutor4 these domains? The DNA-binding domain is
responsible for binding to specific
Select one:
DNA sequences and recognizing the
a. DNA-binding domain and activation domain target gene. The activation domain
b. Activation domain and repression domain is responsible for interacting with
other proteins, such as the
c. RNA-binding domain and repression domain
transcriptional machinery, to activate
d. RNA-binding domain and the activation domain or enhance transcription.
e. DNA-binding domain and ligand-binding domain

Question 137 m-RNA is a complementary copy of:


Complete
Marked out of1 Select one:
a. Antisense strand of DNA

b, 5’ - 3'-strand of DNA
c. t-RNA
d. Ribosomal RNA
e, 3'- 5’-strand of DNA
Question 138 You would like to establish the correlation between gut bacteria and obesity. Which
Complete of these databases will give you relevant information
Marked out of 1
Select one:
a. GiardiaDB

b. ToxoDB

c. Vectorbase

d. PlasmoDB

e. MicrobiomeDB

Question 139 A double-stranded RNA genome isolated from a virus in the stool ofa child with
Complete gastroenteritis was found to contain 15% uracil. What is the percent- age of guanine
Marked out of 1
in this genome?

Select one:
a. 85

b. 20
e.15

d. 25
e. 35

Question 140 In the 20th century, three influenza pandemics occurred: the Spanish influenza in
Complete 1918, the Asian influenza in 1957, and the Hong Kong influenza in 1968. How many
Marked out of 1
million deaths were registered in those pandemics respectively?

Select one:
a. 50 to 200
/ 20/50

b.17
to 100/2/1

c.1to2/0.1/0.3

d. 100 to 200 / 40 / 500

e.0.2 t00.8/1/0.2

Question 141 Which of the following is not correct regarding 2, 3-Biphosphoglycerate?


Complete
Marked out of 1 Select one:
a. Its concentration increases during adaptation to high altitude (acclimatization)

b. All the above are correct

c. It has a less effect on HbF than on HbA

d. It is a positive allosteric effector of the oxygen affinity of hemoglobin

e. It decreases the oxygen affinity of deoxy hemoglobin


Question 142 The activation of receptor tyrosine kinases is characterized by:
Complete
Marked out of 1 Select one:
a. GTP hydrolysis.

b. a phosphorylation cascade.

c. channel protein shape change.


d. dimerization and IP3 binding.
e. dimerization and phosphorylation.

Question 143 Which heterotrimeric G-proteins couple receptors to adenylyl cyclase via the
Complete activation of GTP-bound Ga subunits?
Marked out of1
Select one:
a. Gx family

b. G12/13 family

c. Gs family
d. Gq family

e. Gi family

Question 144 The only correct statement about oncoviruses is


Complete
Marked out of 1 Select one:
a. Both DNA and RNA viruses can be oncoviruses

b. Reverse transcriptase is present in all oncoviruses

c. All the oncoviruses are DNA viruses

d. Viral oncogenes are identical to human protooncogenes

e. All the oncoviruses are RNA viruses

Question 145 The following compound inhibits protein synthesis in humans


Complete
Marked out of 1 Select one:
a, Tetracycline

b. Fusidic acid

c. Erythromycin

d. Diphtheria toxin

e. puromycin
Question 146 Polymers of N-acetylglucosamine and N-acetylmuramic acid are found in which of
Complete the following structures?
Marked out of 1
Select one:
a. Teichoic acid

b. Mitochondrion
c. Cell wall

d. Glycocalyx
e. Lipopolysaccharide

Question 147 Which of the following is the recommended first-line treatment for gout?
Complete
Marked out of 1 Select one:
a. Azaserine

b. None of the other choices

c. Allantoin

d. Allopurinol

e. Methotrexate

Question 148 What must happen in order to prevent overstimulation by a hormone?


Complete
Marked out of 1 Select one:
a, Receptors must be blocked from continuing to activate G-proteins.
b. None of the above
c. G-proteins must be recycled and then degraded.

d. Hormones must be degraded.

e. Receptors must dimerize.


Question 149 After a meal and clearance of carbohydrates/chyomicrons from the blood stream,
Complete blood glucose is initially maintained by liver glycogenolysis then sustained by fat
Marked out of 1
mobilization from adipose tissue. Children with glycogen storage disorders have
immediate problems with glucose maintenance while those with fatty acid oxidation
disorders have later problems after glycogen is depleted. Which of the following
mechanisms best explains the coordinate stimulation of glycogenolysis and lipolysis?

Select one:
a. Adenosine 3’,5’-cyclic monophosphate (cyclic AMP) is the second messenger in
adipocytes, but not in the liver

b. The type II hormone cortisol, a glucocorticoid, binds to liver and adipose


plasma membrane receptors to increase CAMP

c. The type II hormone epinephrine diffuses into the cytosol of liver and adipose
cells, activating phosphorylase kinase and triglyceride lipase by allosteric
mechanisms

d. The type | hormone cortisol, a glucocorticoid, alters transcription in liver and


adipocytes

e. The type II hormone epinephrine acts through the second messenger cAMP
and protein kinases to stimulate glycogen phosphorylase and triglyceride lipase
activities

Question 150 The cardioprotective effects of aspirin occur due to the inhibition of the synthesis of
Complete which one of the following?
Marked out of 1
Select one:
The cardiovascular benefits of aspirin result from its
a. PGA2
irreversible acetylation of serine in cyclooxygenase
(COX)-1 within platelets. This effect abolishes the
b. PGF2a
production of thromboxane A2 (TXA2), a potent
c. PGI2 vasoconstrictor and promoter of platelet
d. TXA2 aggregation for the life of the platelet
e. PGE2
(approximately 7 to 10 days).

149.Epinephrine binds to beta-adrenergic receptors on the cell membrane of liver


and adipose cells, activating adenylate cyclase, which increases the production of
cAMP. cAMP activates protein kinase A (PKA), which in turn activates glycogen
phosphorylase in liver cells to stimulate glycogenolysis and hormone-sensitive
lipase (HSL) in adipose tissue to stimulate lipolysis.
MID SEMESTER 2
2021

"| will not wait for the last moment to read"


is now about to cram past papers @ &

Compiled by Jitu, 2022

Answered by Haron Sami


Biochemistry cat 3
Homologous recombination Homologous recombination is a type of genetic
Select one: recombination in which nucleotide sequences
are exchanged between two similar or identical
a. Involves the sharing of very short regions of DNA sequence
>

molecules of DNA. It is a common mechanism


b. Doesn’t occur in bacteria
Foo Oo

for repairing double-strand breaks in DNA and


c. Doesn’t require the formation of holliday junction for generating genetic diversity during meiosis.
The process involves the formation of a Holliday
d. Reduces genetic diversity junction intermediate and the exchange of
e. Involves the sharing of extensive regions of similar DNA sequence genetic information between homologous DNA
sequences.
Question 2
Complete
Marked out of |

Flag question

Question text

An interferon
Select one:
a. Is an adaptive immune response
>

b. Is virus specific
>

c. Requires expression of cellular genes


5
oOo

d. Is a synthetic antiviral agent


aH

e. Is a bacterial product
Question 3
Complete
Marked out of |

Flag question

Question text

One of the following viral protein is thought to induce tumours by binding to a cellular tumour suppressor
protein
Select one:
a. Adenovirus ELA
>
>

b. Epstein-Barr nuclear antigen proteins


c. Human immunodeficiency virus gag protein
&

d. Human immunodeficiency virus Env protein


Oo
SY

e. Hepatitis B virus e protein

Question 4
Complete
Marked out of |

Flag question

Question text

Inclusion Cell Disease is due to


Select one:
a. Elevated levels of mannose-6-phosphate in the cells
>

b. A deficiency of GlcNAc phosphotransferase (GPT)


>

c. Accumulation of enzymes in the lysosomes


&

d. Lack of mannose in the cells


&
So

e. Lack of proper functioning golgi apparatus


Question 5
Complete
Marked out of 1

Flag question

Question text

One of the following is a viral disease in which antiviral antibodies are thought to have an ‘“enhancing”’
effect so that reinfection with the virus causes a more serious disease
Select one:
a. Hanta virus ADE has been observed with dengue virus 2, Zika virus 3, Ebola
>

virus 4 and, importantly in the context of COVID


>

b. Dengue virus
c. West Nile virus
o

d. Adenovirus E1A
So

e. Coxsackie A virus

Question 6
Complete
Marked out of |

Flag question

Question text

Enteroviruses differ from rhinoviruses mainly in their:


Select one:
a. Ability to survive acidic conditions
>

b. Tropism
>

c. Capsid shape
&

d. Type of nucleic acid


&
So

e. Strandedness
Question 8
Complete
Marked out of |

Flag question

Question text

One of the following is a mechanism of action of AIDS drug azidothymidine (AZT), the first drug to be
effective against AIDS, including the reduction of maternal-to-child AIDS transmission by 30%.

Select one:
®

a. It inhibits viral reverse transcriptase


b. It inhibits RNA synthesis
©@

c. It inhibits viral DNA polymerase


&@

d. It blocks viral budding


So

e. It stimulates DNA provirus production


Question 9
Complete
Marked out of 1

Flag question

Question text

Noncyclic photophosphorylation produces


Select one:
a. NADPH only,
aoOo ©

b. ATP only.

c. Oxygen.

d. Water and ATP.


®

e. ATP and NADPH.

Question 10
Complete
Marked out of 1

Flag question

Question text

One of the following diagnostic tests has been more commonly used in the diagnosis of COVID-19
Select one:
>

a. Virus isolation in cell culture

b. Real-time reverse transcription polymerase chain reaction (rRT-PCR) assay


>

c. Viral antigen detection test


Oa do

d. Immunofluorescent assay (IFA)


e. Urinalysis
Question 12
Complete
Marked out of |

Flag question

Question text
What is the percentage of guanine in a double-stranded RNA genome isolated from a virus in the stool of
a child with gastroenteritis whose uracil content is 15%?

Select one:
a. 25
Oo
56

b. 85
oe

c. 35
d. 15
o
oO

e. 20
Question 14
Complete
Marked out of |

Flag question

Question text

Bacterium can acquire antibiotic resistance by all of the following mechanisms, EXCEPT

Select one:
5

a. Exchanging DNA with another bacterium


Oo

b. On its own through evolution


c. Acquiring resistance gene from its host’s cells


d. From its parent cell
©
oo

e. Scavenging resistance genes from the environment


Question 15
Complete
Marked out of |

Flag question

Question text

Influenza virus which results in more than 500,000 worldwide annual deaths contains an eight-piece
segmented negative-sense RNA genome that encodes the HA (hemagglutinin) protein that directly binds
to one of the following host cell epithelial component
Select one:
a. Serine-threonine kinase receptors Sialic acid is a type of sugar molecule that is found on the surface of
»oaasds

many host cell types, including epithelial cells in the respiratory tract.
b. Uronic acid subgroups Influenza virus uses the HA protein to bind to sialic acid on the host cell
c. Chemokine receptors surface, which facilitates viral entry into the host cell and infection.
d. Sialic acid
e. Cytokine receptors
+

Question 16
Complete
Marked out of |

Remove flag

Question text

Resistance of Staphylococcus aureus to methicillin is most often caused by


Select one:
a. Alternation of the major target for the drug
>

b. Cell membrane permeability


c. Synthesis of a potent Beta Lactamase
aansa

d. Inactivation of autolysins
e. Decreased uptake of the antibiotic

Methicillin-resistant Staphylococcus aureus (MRSA) strains have acquired a gene called mecA,
which encodes a modified penicillin-binding protein (PBP2a) that has a low affinity for methicillin
and other beta-lactam antibiotics. This allows the bacteria to continue synthesizing their cell walls
even in the presence of these antibiotics
Question 17
Complete
Marked out of 1

Flag question

Question text

An example of a killed virus vaccine is


Select one:
The Salk poliovirus vaccine, also known as the
a. Salk poliovirus vaccine
>

inactivated poliovirus vaccine (IPV), is made from


d

b. Enders’ measles vaccine inactivated (killed) poliovirus and is used to prevent


c. Jeryl Lynn mumps vaccine
o

poliomyelitis (polio).
d. Oka varicella-zoster vaccine
oo

e. COVID-19 Pfizer vaccine

Question 18
Complete
Marked out of 1

Flag question

Question text

An example of an RNA virus that has a nuclear phase to its replication process is
Select one:
>

a. Retrovirus

b. Togavirus
Yoo 5

c, Coronavirus
d. Smallpox virus
e. Rhabdovirus
Question 19
Complete
Marked out of |

Remove flag

Question text

Passive immunization is available for protection from one of the following viruses
Select one:
a. Influenza A virus
@

b. Yellow Virus
&

c. Rubella virus
&
aS

d. Parainfluenza type 2 virus


e. Hepatitis A virus
>
Question 21
Complete
Marked out of 1

Flag question

Question text

Viral oncogenes are present in


Select one:
a. JC virus
eo35 5

b. Simian virus 40
c. Moloney murine leukaemia viruses
d. Human T-cell Lymphotropic virus type I
oO

e. Rous sarcoma virus

Question 22
Complete
Marked out of 1

Flag question

Question text

A mutation in DNA gyrase is likely to result in resistance to which one of the following antibiotics
Select one:
a. Streptomycin
eoao 5

b. Penicillin
c. Rifampin
d. Ciprofloxacin
a.

e. Amphotericin B
Question 23
Complete
Marked out of |

Remove flag

Question text

One of the following best describes the mechanism of an inherited problem in a 32-year-old female breast
cancer whose mother and one maternal aunt had breast cancer while her maternal grandmother had
ovarian cancer. Inherited mutations in tumor suppressor
genes such as BRCA1 and BRCA2 can
Select one:
increase the risk of developing breast and
9

a. An oncogene leading to loss of apoptosis ovarian cancer. These genes are involved
b. An oncogene leading to a constitutively active MAP kinase pathway in DNA repair and when they are mutated,
>_>

the cell’s ability to repair DNA damage is


c. A tumor suppressor leading to an inability to repair DNA compromised, which can lead to the
d. A tumor suppressor leading to loss of apoptosis accumulation of mutations and the
3

development of cancer. It is important to


mo

e. All the responses are correct


note that not all cases of breast and
Question 24 ovarian cancer are caused by inherited
Complete mutations in BRCA1 or BRCA2 and other
Marked out of | factors can also increase the risk of
developing these cancers.

Flag question

Question text

The differential aspect of MacConkey's agar (MAC) reveals one of the following about the metabolic
capabilities of the organism that grows within
Select one:
a. If the organism produces hemolytic enzymes that can lyse erythrocytes
oO

b. Whether the bacteria are Gram + or Gram -


9Oo

c. If the organism ferments lactose


d. If the organism ferments mannitol
oO da

e. If the organism is acid-fast

The differential aspect of MacConkey’s agar (MAC) reveals if the organism ferments lactose.
Question 26
Complete
Marked out of |

Flag question

Question text

The following statements are true regarding the bacterial M protein, EXCEPT
Select one:
a. It is found on Streptococcus pyogenes.
o Oo

The bacterial M protein is a virulence factor found on the surface of Streptococcus


b. It is found on fimbriae. pyogenes (group A Streptococcus) and is known to confer resistance to
phagocytosis.
oe

c. It is readily digested by phagocytes.


d. It is a protein.
o
oO

e. It is heat- and acid-resistant.


One of the following is an example of direct damage due to bacterial infection
Select one:
a. The excessive secretion of fluids in a Vibrio cholera infection
»oO 9

b. The uncontrolled muscle contractions in Clostridium tetani infection


c. The invasion and lysis of intestinal cells by E. coli
6

d. The hemolysis of red blood cells in a staphylococcal infection


mo

e. The fever, nausea, and low blood pressure in a Salmonella infection


Question 29
Complete
Marked out of |
Flag question

Question text

Frameshift mutation can be due to


Select one:
a. Deletions
6
2Oo

b. Indels
c. All the other choices are correct
d. Duplications
©
oO

e. Insertions
Question 30
Complete
Marked out of 1

Flag question

Question text

Fermentative metabolic pathways


Select one:
a. Produce substrates for the Krebs cycle.
aoOo ©

b. Occur only when oxygen is readily available.


c. Are alternatives for the pentose phosphate pathway.
d. Produce substrates for glycolysis.
e. Use an organic molecule as a final electron acceptor.
®

Question 3 1
Complete
Marked out of 1

Flag question

Question text

Common DNA damage caused by UV light is


Select one:
a. A-C mismatch
eoao 5

b. Nicking of the DNA strands


c. Winding of the DNA duplex
d. Formation of thymine-thymine dimers
@.

e. Deamination of cytosine to uracil


Question 32
Complete
Marked out of |

Flag question

Question text

Zellweger syndrome
Select one:
a. Is due to loss of mitochondrial functions
>

b. Is due to mutation in any of the PEX genes

c. Is not a genetic disorder


OOo5

d. Is not associated with protein targeting


e. Doesn’t cause any developmental defects

Question 33
Complete
Marked out of 1

Flag question

Question text

A well-organized layer of material lying outside the bacterial cell wall and which is not easily washed off
is known as.
Select one:
a. Slime layer
>

b. Capsule
>

c. Glycocalyx
&
S: &

d. Envelope

e. Spore
Question 34
Complete
Marked out of |

Remove flag

Question text

One of the following rankings is true of some viruses which are highly contagious and do not spread as
easily as of July 2021.
Select one:
&

a. COVID-19 &lt; influenza &lt; SARS &lt; Ebola


b. Influenza &lt; SARS &lt; COVID-19 &lt; Ebola
&

c. Influenza &lt; COVID-19 &lt; Ebola &lt; SARS


oo

d. Ebola &lt; SARS &lt; Influenza &lt:;COVID-19


*

e. SARS &lt; influenza &lt; Ebola &lt; COVID-19


Question 36
Complete
Marked out of 1

Flag question

Question text

The structure of teichoic acids consists of


Select one:
a. Glycerol residues
©

b. Glucose residues
o

c. Sorbitol or glycerol residues


&

d. Ribitol residues
a

e. Ribitol or glycerol residues


®

Question 37
Complete
Marked out of 1

Flag question

Question text

Which of the following compounds is not a DNA mutagen?


Select one:
a. Proflavin
>

b. Thymine
>

c. Methyl methanesulfonate
5

d. Ethidium bromide
Oa

e. N-methyl-N'-nitro-N-nitrosoguanidine
Question 38
Complete
Marked out of |

Flag question

Question text

The fimbriae of Neisseria gonorrhea and enteropathogenic E. coli functions as


Select one:
a. Adhesins and ligands.
5
5

b. Ligands.

c. Receptors.
Ho

d. Adhesins, ligands, and receptors.


»

e. Adhesins.

Question 39
Complete
Marked out of |

Flag question

Question text

Gram (-) and Gram (+) bacteria can be distinguished by one of the following characteristics

Select one:
a. One has 70S ribosomes and the other has 80S ribosomes
oooO 5

b. The cell membrane is covalently connected to peptidoglycan in one but not in the other
c. The pilus composition and structure differ in the two
d. One has permeases and the other has group translocation proteins
ry)

e. One produces endotoxin, but the other does not


Question 40
Complete
Marked out of |

Flag question

Question text

Which of the following choices lists the steps of pathogenesis in the correct order?
Select one:
a. Invasion, infection, adhesion, exposure
aods 6

b. Disease, infection, exposure, invasion

c. Infection, invasion, exposure, adhesion

d. Adhesion, exposure, infection, invasion


®

e. Exposure, adhesion, invasion, infection

Question 4 1
Complete
Marked out of 1

Flag question

Question text

The role of bacterial capsules as virulence factors is usually related to their ability to interfere with
Select one:
o

a. Antibacterial penetration of bacterial cells

b. B lymphocyte activation
eo

c. Phagocytosis
d. The release of interferon-gamma and other macrophage activating cytokines
YY

e. Antibody binding
Question 43
Complete
Marked out of |

Flag question

Question text
The first viral-induced defense mechanism in a nonimmune individual is the:
Select one:
a. Synthesis of lymphokines
Oo

b. Production of interferon
©

c. Production of complement system


d. Synthesis of neutralizing antibodies
&
a

e. Generation of cytotoxic T lymphocytes


Question 45
Complete
Marked out of |

Flag question

Question text
A change in a single base pair ina DNA sequence that leads to a change in amino acid of the encoded
protein is called

Select one:
a. Missense mutation
»

b. Nonsense mutation
Oooo DO

c. Frameshift mutation
d. Silent mutation
e. Point mutation
Question 46
Complete
Marked out of |

Flag question

Question text

N-glycosylation in most proteins is through which amino acid


Select one:
N-glycosylation is a type of covalent modification where a carbohydrate
a. Tyrosine
5

molecule is attached to a nitrogen atom in the side chain of an asparagine


b. Proline
5

residue in a protein. The process occurs in the endoplasmic reticulum and


c. Glycine Golgi apparatus of eukaryotic cells and is important for protein folding, stability,
Ho

d. Serine
and function.
e. Asparagine
®

Question 47
Complete
Marked out of |

Flag question

Question text

The bacterial cell wall has all of the following properties, EXCEPT: It
Select one:
a. It consists of a glycan backbone, a tetrapeptide with a peptide cross-linkage
eo ds

b. Is the structure principally responsible for the reaction of gram staining


c. Is a unique flexible plastic structure
d. It contains D-isomers of amino acids
YY

e. Consists of a mixed polymer called peptidoglycan


Question 49
Complete
Marked out of |

Flag question

Question text
DNA alkylating agents that are mutagenic include

Select one: Nitrosoguanidine(N-methyl-N'-nitro-N-nitrosoguanidine,


a. Bromouracil MNNG)
Oo

• DNA alkylating agent that is a potent mutagenic substance


eoOo

b. Aminopurine • Experimentally used to induce mutagenesis


c. Nitrous acid
d. Acridine orange
e. Nitrosoguanidine
a
Question 50
Complete
Marked out of |

Flag question

Question text

Linear, single-stranded DNA is the genetic material of


Select one:
a. Flaviviruses
08 3 5

b. Calciviruses
ao

c. Parvoviruses
d. Retroviruses
e. Papillomaviruses
Trp operon
Select one:
a. Regulates expression of genes responsible for arabinose metabolism
3
oO

b. Inhibits expression of genes required for tryptophan catabolism


oe

c. Regulates the expression of genes responsible for tryptophan metabolism


d. Controls the breakdown of lactose
6

e. Activates utilization of tryptophan in prokaryotic cells


a

Question §2
Complete
Marked out of |
Flag question

Question text

One of the following compounds on the cell wall of gram-positive bacteria is involved in the induction of
septic shock
Select one:
a. Peptidoglycan
>

b. Phospholipid
oo4 5

c. Capsular protein

d. Teichoic acid
e. Endotoxin
Question 53
Complete
Marked out of 1

Flag question

Question text

HTLVs
Select one:
a. Are associated with leukemias
>

Human T-lymphotropic virus type 1 (HTLV-1) is


b. Carry tyrosine protein kinase oncogenes
58 5

associated with adult T-cell leukemia; HTLV-2 is


Ho

c. Are defective RNA tumor viruses implicated in human hairy cell leukemia.
d. Are DNA viruses
e. Synthesize early proteins that interact with pS3Rb

Question 54
Complete
Marked out of 1

Flag question

Question text

One of the following is a receptor for the SARS-CoV-2 Virus


Select one:
a. ACE-2
Oo
oO

b. CPG
oe

c. TLR-9

d. Erb-B
oO
oO

e. CXCR4
Question 55
Complete
Marked out of |

Flag question

Question text

A 37-year-old immigrant from Tanzania develops fevers, night sweats, weight loss, and a blood-tinged
cough and present himself to the emergency room, where an infectious disease doctor is consulted and
immediately prescribes a multidrug regimen that includes Rifampin which is known to inhibit one of the
following types of enzymes
Select one:
a. DNA-dependent DNA polymerase
>

b. DNA-dependent RNA polymerase


>

c. Reverse transcriptase
6
oO

d. None of the above


oO

e. RNA-dependent DNA polymerase

Question 56
Complete
Marked out of 1

Remove flag

Question text
The following are all functions of bacterial capsule, EXCEPT:

Select one:
a. Aid in attachment to surfaces.
>
»>

b. Exchange genetic information by conjugation.


c. Block attachment of bacteriophage.
5

d. Protect against temporary drying.


Ho

e. Prevent engulfment by white blood cells.


Question 57
Complete
Marked out of 1

Flag question

Question text

Which of the following characteristics do not occur in prokaryotic cells?


Select one:
a. Oxygenic photosynthesis
aoOo ©

b. Anaerobic respiration
c. Thylakoid membranes within chloroplasts
d. Several circular chromosomes
*)

e. Cellular organization

Question 58
Complete
Marked out of 1

Remove flag

Question text

One of the following databases will provide relevant information in the establishment of Of a correlation
between gut bacteria and obesity.
Select one:
a. MicrobiomeDB
5
Oo

b. ToxoDB
Oo

c. Vectorbase
AO

d. PlasmoDB
e. GiardiaDB
>
Question 59
Complete
Marked out of |

Flag question

Question text

One of following correctly describes the effects of HPV on cell growth for a 50-year-old woman who has
had numerous episodes of unprotected intercourse since the age of 12 years and tests positive for Pap
smear test indicative of human papilloma virus (HPV) infection after visiting a gynaecologist.
Select one:
a. It synthesizes a transforming large T antigen.
One me)

b. The viral E6 protein perturbs the normal function of cellular Rb.


c. The viral E6 protein degrades cellular p53.
d. The viral E7 protein degrades cellular p53.
3
Oo

e. The viral LMP-1 protein prevents the expression of cellular bel-2.

Question 60
Complete
Marked out of |

Flag question

Question text

The following statements are true regarding bacteria, EXCEPT:


Select one:
a. The flagella allow substances in and out of the bacteria
*

b. Peptidoglycan is the main constituent of a Gram-positive cell wall


aod sd

c. A plasmid is a piece of extra-chromosomal bacterial DNA


d. Bacterial cells have a large surface to volume ratio
e. Bacterial cell walls are a potential target for antibiotics
Question 61
Complete
Marked out of |

Flag question

Question text

One of the following is a virus that infects and lyses progenitor erythroid cells causing aplastic crises in
patients with hemolytic anemia
Select one:
>

a. Hepatitis B virus
>

b. Parvovirus B19
c. Yellow fever virus
Yo 5

d. Epstein-Barr virus
e. California encephalitis virus

Question 62
Complete
Marked out of |

Flag question

Question text

The following statements are true regarding reduction reactions, EXCEPT


Select one:
a. They frequently involve electron carrier molecules.
>

b. An electron acceptor becomes more positively charged.


>

c. An electron acceptor gains an electron.


3
Oo

d. They are coupled with oxidation reactions.


aH

e. A molecule gains a hydrogen atom.

In a reduction reaction, an electron acceptor gains an electron and becomes more negatively charged
Question 63
Complete
Marked out of 1

Flag question

Question text

One of the following metabolic processes is commonly used to identify bacteria


Select one:
a. Krebs cycle products
>

b. Carbohydrate fermentation
c. Lipid catabolism
OOo5

d. Lipid synthesis
e. Pentose phosphate pathway products

Question 64
Complete
Marked out of 1

Remove flag

Question text

The likely pathogenesis factor of patient with bacterial contaminated & blocked catheter is
Select one:
a. Possession of pili
>

b. Biofilm production
Yoo 5

c. Ergosterol containing membrane


d. Possession of IgA protease
e. Peptidoglycan layer

Biofilms are communities of microorganisms that attach to surfaces and produce a slimy film that can protect
them from antibiotics and the immune system. Biofilms can form on medical devices such as catheters and
can lead to infections
Question 65
Complete
Marked out of |

Flag question

Question text

Strategies to reduce bacterial resistance to antibiotics may involve:


Select one:
a. Genetic typing of bacteria to determine transmission route.
>
*>

b. De-escalation and restrictive antibiotic policies.


c. Greater use of broad-spectrum agents.
aoOo 5

d. Promotion of handwashing and basic hygiene measures.


e. Contact isolation and screening.

Question 66
Complete
Marked out of |

Flag question

Question text

One of the following bacterial antibiotic resistance mechanisms is effective against a wide range of
antimicrobials
Select one:
a. Drug inactivation
5

b. Target mimicry
o

c. Target modification
oo

d. Efflux pump
oO

e. Target overproduction
Question 67
Complete
Marked out of 1

Flag question

Question text
The following statements are true regarding endotoxins, EXCEPT
Select one:
a. They often exhibit enzymatic action
>

b. Is produced mainly by gram-negative bacteria


Ho85 5

c. Toxoids cannot be made from it


d. They are heat stable
e. Protein-polysaccharide-lipid complex in nature

Question 68
Complete
Marked out of 1

Flag question
Question text

Breast cancer can be caused by a mutation in the following gene


Select one:
a. XP-G
eooOo 5

b. PMS2
c. MSH2
d. BRCAI
@.

e. MLHI
Question 69
Complete
Marked out of 1

Flag question

Question text

What is the most likely viral infection in a 23-year-old female who in the past had mononucleosis
currently presents with chronic fatigue for the past 9 months with blood tests indicating chronic viral
infection while liver biopsy in conditions that initiates apoptosis indicates continued cellular growth
Select one:
a. HIV-1 virus
5

b. Influenza virus
o

c. Herpes simplex virus


o
9

d. Epstein-Barr virus
&

e. Polio virus
Question 72
Complete
Marked out of |

Remove flag

Question text
A clinical viral disease

Select one:
*

a. Is associated with target organs in most disseminated viral infections


b. Is most frequently due to toxin production
& @
ao

c. Is a major feature of congenital viral infections


d. Usually follows virus infection
e. Can result without infection of host cells

Many viral infections are asymptomatic or subclinical. Clinical disease, however, is often associated with viral replication in
target organs during disseminated viral infections
Question 73
Complete
Marked out of |

Flag question

Question text

T-G mismatch in humans is mostly caused by


Select one:
a. Deamination of cytosine to Uracil or 5-methyl cytosine to thymine
*

b. Lack of repair mechanism in the cells


&

c. Lack of enzymes which recognize the mismatch


&

d. The need to have more mutants


ao

e. A large proportion of T and G bases in the cells

Question 74
Complete
Marked out of 1

Flag question

Question text

Which of the following utilizes recombination to repair double strand breaks?


Select one:
o

a. Homologous recombination Homologous recombination is a mechanism that repairs


b. Nucleotide excision repair double-strand breaks in DNA by using an undamaged,
oo

homologous DNA molecule as a template.


c. None of the other choices is correct
d. Base excision repair
YY

e. Mismatch excision repair


Question 75
Complete
Marked out of |

Flag question

Question text

The exchange of homologous segments of RNA between two different influenza type A viruses is called
Select one:
a. Super infection
aods 6

b. Phenotypic masking

c. Phenotypic mixing
d. Complementation
e. Genetic reassortment
®

The following statements are true regarding mitochondrial proteins, EXCEPT


Select one:
a. The targeting sequence is removed in the mitochondrial matrix
o
oe

b. Most mitochondrial proteins are encoded by mitochondral DNA


c. Mitochondrial Proteins are synthesized in the cytosol
ao so

d. Most mitochondrial proteins are encoded by nuclear DNA


e. Their targeting sequence is located at the N-terminus

Question TS
Complete
Marked out of |
Flag question

Question text

One of the following molecules typically serves as the final electron acceptor during bacterial
fermentative processes
Select one:
a. NAD+
©

b. Pyruvate
c. Nitrate
3

d. Oxygen
oo

e. CO2
Question 78
Complete
Marked out of |

Flag question

Question text

Mycobacterium tuberculosis cell walls mainly consists of


Select one:
5

a. Acetone

b. Cellulose
oo

c. Mycolic acid
d. Myristic acid
6
ao

e. Nitric acid

Question 79
Complete
Marked out of 1

Flag question

Question text

Individuals taking antibiotics often develop Candida albicans infection mainly because:
Select one:
o

a. Antibiotics stimulate the synthesis of ergosterol of C. albicans.

b. Antibiotics support the growth of C. albicans.


eo

c. The antibiotics damage the host mucous membranes.


d. The normal bacterial flora is greatly affected by the antibiotic
Oo
YY

e. Albicans degrades the antibiotics.


Question 80
Complete
Marked out of |

Flag question

Question text

Which of the following statements is not true about DNA polymerase?


Select one:
a. It is important in DNA replication
5

b. It has proofreading activity


3

c. DNA polymerase can introduce and also correct errors


eo

d. It is synthesized in the nucleus


oO

e. It is required in nucleotide excision repair

Question 8 1
Complete
Marked out of 1

Flag question

Question text

Which of the following is not a structural variation?


Select one:
a. Microsatellites
o

b. Insertions
eo

c. Single nucleotide polymorphisms (SNPs)


d. Copy number variations (CNVs)
oO
oY

e. Inversions
Question 83
Complete
Marked out of 1

Flag question

Question text
Host cell tRNAs are involved in the genome replication of
Select one:
a. Influenza A virus
OD

b. Rhinovirus
Ga

c. Respiratory syncytial virus


Oo
oa

d. Retroviruses
a

e. Adenoviruses
Question 84
Complete
Marked out of |

Flag question

Question text

Most of the ATP generated in prokaryotic cells occurs in the


Select one:
a. Cytosol
5

b. Thylakoids
oo

c. Cytoplasmic membrane
d. Inner mitochondrial membrane
6
ao

e. On ribosomes

Question 85
Complete
Marked out of 1

Remove flag

Question text

What is the most likely causative factor of a malignancy in a 62-year-old male with an initially reddish,
rough patch with white scales on the top of his ear which does not get treated and 3 years later become
enlarged, raised lesion with a central ulcerated area that does not heal.
Select one:
a. Oncogenic RNA virus
aooO 5

b. Double-strand breaks in the DNA


c. Creation of pyrimidine dimers
d. Oncogenic DNA virus
e. Creation of hydroxy] radicals
>

Pyrimidine dimers are formed when UV radiation causes adjacent pyrimidines on the same DNA strand to
bond together. This can result in DNA damage and mutations that can lead to skin cancer.
Question 86
Complete
Marked out of 1

Flag question

Question text

DNA mutations can be caused by


Select one:
a. X- and gamma-rays

b. chemical compounds
ao8 5

c. DNA recombination
d. DNA replication
e. Visible light

Question 87
Complete
Marked out of 1

Flag question

Question text

The following statements are true regarding bacterial virulence factors, EXCEPT
Select one:
a. Hyaluronidase breaks down substances between cells
Hoa ds

b. Leukocidins destroy neutrophils


c. Kinase destroys fibrin clots
d. Hemolysins lyse red blood cells
e. Coagulase destroys blood clots
>
Question 88
Complete
Marked out of |

Flag question

Question text

Viruses whose genomes have a messenger (positive-sense) polarity are


Select one:
a. Papoviruses
»

b. Paramyxoviruses
aaod d

c. Polioviruses
d. Adenoviruses
e. Ebola Viruses

Question 89
Complete
Marked out of |

Flag question

Question text

The following are some of the usual features in the replication cycle of coronaviruses, except
Select one:
>

a. Replication in hepatocytes

b. They take advantage of recombination with the long RNA genome


Yoo o

c. They use capped cellular mRNA's


d. They are not highly mutable
e. The RNAs all terminate in a common 3’ and produce nested set transcripts
Question 90
Complete
Marked out of |

Flag question

Question text

The following are some biochemical mechanisms occurring in bacterial resistance, EXCEPT:
Select one:
a. The development of alternative biochemical pathways.
5

b. Active efflux of antibiotics.


35
oO

c. Modifications to cell wall permeability.


d. Reverse chemotaxis away from areas of high antibiotic concentration.
*
Oo

e. Producing enzymes that break down antibiotic.

Question 91
Complete
Marked out of 1

Flag question

Question text

Regarding oncoviruses
Select one:
o

a. All the oncoviruses are DNA viruses

b. Reverse transcriptase is present in all oncoviruses


eo

c. Both DNA and RNA viruses can be oncoviruses


d. Viral oncogenes are identical to human protooncogenes
YY

e. All the oncoviruses are RNA viruses


Question 92
Complete
Marked out of |

Flag question

Question text

The following proteins/enzymes are encoded for by lac operon genes EXCEPT;
Select one:
a. galactoside transacetylase
>

b. Hexokinase
>

c. Lac repressor
aoOo 5

d. B-galactosidase
e. galactoside permease

Question 93
Complete
Marked out of 1

Flag question

Question text

What is the common feature between N-acetylglucosamine and N-acetylmuramic acid of the bacterial
peptidoglycan?
Select one:
a. They are amino acids.
O

b. They have peptides attached to them.


Oo
Oo

c. Neither of them is found in the peptidoglycan chains.


oe

d. They are carbohydrates.


oO

e. Neither of them is an organic compound


Question 94
Complete
Marked out of 1

Flag question

Question text

All of the following contribute to a pathogen's invasiveness, EXCEPT


Select one:
a. Coagulases
So& ©

b. Cell wall components

c. Toxins
d. Hyaluronidase
e. Capsules
®

Question 95
Complete
Marked out of 1

Flag question

Question text

The following statements are true regarding exotoxins, EXCEPT


Select one:
a. They have specific methods of action
>

Exotoxins are susceptible to heat and can be inactivated by heat


b. They are resistant to heat
>

treatment.
c. They are composed of proteins
5

d. They are produced by gram-positive bacteria


Oa

e. They are more potent than endotoxins


Question 96
Complete
Marked out of 1

Flag question

Question text

How many million deaths were registered in three 20th influenza pandemics (the Spanish influenza, 1918,
the Asian influenza, 1957 and the Hong Kong influenza, 1968) respectively
Select one:
a. 0.2 to 0.8/1/0.2
oo Oo

b. 100 to 200
/ 40 / 500
c. 50 to 200
/ 20/50
d.17to 100/2/1
©
SS

e. 1 to2/0.1/0.3

Question 97
Complete
Marked out of |

Flag question

Question text

The inducer molecule in lac operon is


Select one:
a. Lactose
&Oo &

b. Glucose
c. Galactose
Oo

d. Mannose
>

e. Allolactose
Question 98
Complete
Marked out of |

Flag question

Question text

Which of the following is a component of Translocase of the inner mitochondrial membrane?


Select one: TIM23 (Translocase of Inner Membrane 23) is a protein complex that is located in the
a. TOM70
5

inner mitochondrial membrane and is responsible for translocating proteins from the
cytosol into the mitochondrial matrix. TOM70, TOM20, TOM22, and TOM40 are
b. TOM20
35

components of the TOM (Translocase of the Outer Membrane) complex, which is


oO

c. TOM22 involved in the initial recognition and binding of precursor proteins to the mitochondrial
d. TIM23 outer membrane.
*
Oo

e. TOM40

Question 99
Complete
Marked out of 1

Flag question

Question text

What is the mode of action of the aminoglycoside’s antibacterial agents particularly on against Gram-
negative bacilli?
Select one:
a. Inhibition of protein synthesis
*

b. Disruption of cytoplasmic membrane function


0
Oo

c. Inhibition of bacterial DNA gyrase


d. Inhibition of bacterial cell wall synthesis
o
aH

e. Interference with bacterial folic acid metabolism


MID SEMESTER 2
2021 MAKE UP
CAT

Karibu nikukumbushe Dr. Mulinge alisema hatarudia


maswali kutoka kwa past papers€><>

Answered by Haron Sami

Compiled by Jitu, 2022


Search Courses Q (Search Courses)

S> My courses > Biochemistry > TopicS >» MBCHB BPHARM


& BDS LEVEL
II SPECIAL CAT III

Startedon Friday, 27 August 2021, 2:31 PM


State Finished
Completed on Friday, 27 August 2021, 4:27 PM
Time taken 1 hour 55 mins

Question 1 The following proteins/enzymes are encoded for by lac operon genes EXCEPT;
Complete
Marked out of 1 Select one:
a. B-galactosidase The lac operon contains three genes: lacZ,
b, galactoside permease
lacY, and lacA. These genes are transcribed as
a single mRNA, under control of one promoter.
c. Lac repressor
Genes in the lac operon specify proteins that
d. galactoside transacetylase help the cell utilize lactose
e. Hexokinase

Question 2 An interferon
Complete
Marked out of 1 Select one:
a. Is a synthetic antiviral agent
b. Is an adaptive immune response

c. Requires expression of cellular genes


d. Is virus specific

e. Is a bacterial product
Question 3 Bacterial plasmids
Complete
Marked out of 1 Select one:
a. Are often the site of pathogenic genes

b. Are essential for survival

c. Are found in all bacteria


d. Cannot be passed on to progeny
e. Cannot be passed between organisms

Question 4 The exchange of homologous segments of RNA between two different influenza type
Complete A viruses is called
Marked out of 1
Select one:
a. Phenotypic masking
b. Genetic reassortment
c. Complementation

d. Phenotypic mixing
e. Super infection
Question 6 Each of the following statements concerning bacterial spores is correct. EXCEPT:
Complete
Marked out of 1 Select one:
a. Are more likely to survive treatment with disinfectants
b. They are formed by gram-positive rods
c. Their survival ability is based on their enhanced metabolic activity
d. They can be killed by being heated to 1210C for 15 minutes
e. They contain much less water than bacterial cells
Question 7 Gram-positive bacteria
Complete
Marked out of1 Select one:
a. Use teichoic acid to move ions across the cell wall.
b. Can release porins to poison the host.
c. Are freely permeable to disinfectants.
d. Have a double membrane.
e. Have a thin peptidoglycan layer.
Question 11 Which of the following does NOT contribute to a pathogen's invasiveness?
Complete
Marked out of 1 Select one:
a. Ligands
b. Cell wall

c. Capsule

d. Hyaluronidase
e. TOxINs
Question 14 Endotoxins:
Complete
Marked out of 1 Select one:
a. Are lipopolysaccharide present on bacterial cell wall
b. May be converted to toxoids.
c. Are only produced by Gram-negative bacteria.
d. Can act as antibiotics
e. Are tissue-destroying enzymes.
Question 1 5 Common DNA damage caused by UV light is
Complete
Marked out of 1 Select one:
a. Formation of thymine-thymine dimers

b. A-C mismatch

c. Winding of the DNA duplex


d. Nicking of the DNA strands
e. Deamination of cytosine to uracil

Question 16 The inducer molecule in lac operon is


Complete
Marked out of 1 Select one:
a. Glucose

b. Lactose

c. Galactose

d. Allolactose

e. Mannose

Question 17 Polymers of N-acetylglucosamine and N-acetylmuramic acid are major components


Complete of
Marked out of1
Select one:
a, Teichoic acid
b. Lipopolysaccharide

c. Glycocalyx
d. Capsule
e. Cell wall

Question 18 One of the following is a receptor for the SARS-CoV-2 Virus


Complete
Marked out of 1 Select one:
a, Erb-B

b. CXCR4

c. CPG
d. TLR-9

e. ACE-2
Question 19 Which of the following is not a structural variation?
Complete
Marked out of 1 Select one:
a. Single nucleotide polymorphisms (SNPs)
b. Microsatellites

c. Inversions

d. Copy number variations (CNVs)


e, Insertions

Question 20 The drug azidothymidine (AZT) for AIDS management , works by


Complete
Marked out of 1 Select one:
a. Blocking the viral budding

b. Inhibiting the viral DNA polymerase


c. Stimulating DNA provirus production
d. Inhibiting the viral reverse transcriptase
e. Inhibiting the RNA synthesis
Question 22 The cell wall complex of Gram positive bacteria can be best described by one of the
Complete following statement?
Marked out of 1
Select one:
a. Astructure composed of lipoteichoic, teichoic acids and peptidoglycan.
b. Acell surrounded by an outer membrane, but lacking a peptidoglycan layer
c. Astructure that contains lipopolysaccharide, lipoteichoic acids and
peptidoglycan
d. Athin peptidoglycan layer surrounded by an external outer membrane
e. A thick, homogeneous peptidoglycan layer surrounded by an outer membrane
Question 23 Zellweger syndrome
Complete
Marked out of 1 Select one:
a. Is due to loss of mitochondrial functions
b. Doesn't cause any developmental defects
c. Is not a genetic disorder

d. Ils due to mutation in any of the PEX genes


e. Is not associated with protein targeting

Question 24 Passive immunization is available for protection from one of the following viruses
Complete
Marked out of 1 Select one:
a. Hepatitis A virus

b. Parainfluenza type 2 virus


c. Yellow Virus

d. Rubella virus

e. Influenza A virus
Question 28 All bacterial cells have
Complete
Marked out of1 Select one:
a. Capsules

b. One or more chromosomes

c. The ability to produce endospores

d. One or more fimbriae

e. Flagella
Question 32 DNA alkylating agents that are mutagenic include
Complete
Marked out of 1 Select one:
Nitrosoguanidine(N-methyl-N'-nitro-N-nitrosoguanidine,
a. Acridine orange MNNG)
• DNA alkylating agent that is a potent mutagenic substance
b. Aminopurine
• Experimentally used to induce mutagenesi
c. Bromouracil
d. Nitrous acid

e. Nitrosoguanidine

Question 33 Each of the following statement is true concerning penicillin, EXCEPT:


Complete
Marked out of 1 Select one:
a. Penicillin inhibits trans-peptidases, which are important for cross-linking
peptidoglycan.
b. Penicillin is a bacteriostatic drug because autolytic enzymes are not activated.

c. Resistance to penicillin occurs due to reduced affinity of the transpeptidases.


d. Penicillin structure is analogous to a dipeptide of alanine, a component of
peptidoglycan.

e. An intact B-lactam ring of penicillin is required for its activity.


Question 35 Gram negative organisms Gram-negative bacteria have an outer membrane that protects
Complete them from certain antibiotics, making them generally less
Marked out of 1 Select one: susceptible to antibiotics than gram-positive bacteria
a. None of the choices are correct

b. Stain purple in the gram stain


c. Are more susceptible to antibiotics than gram positive organisms
d. Are less susceptible to antibiotics than gram positive organisms
e. Encompass all pathogens

Question 36 Homologous recombination


Homologous recombination is a type of
genetic recombination in which nucleotide
Complete
sequences are exchanged between two
Marked out of 1 Select one:
similar or identical molecules of DNA. It is a
a. Doesn't require the formation of holliday junction
common mechanism for repairing
b. Involves the sharing of extensive regions of similar DNA sequence double-strand breaks in DNA and for
generating genetic diversity during meiosis.
c. Doesn't occur in bacteria
The process involves the formation of a
d. Involves the sharing of very short regions of DNA sequence Holliday junction intermediate and the
e. Reduces genetic diversity
exchange of genetic information between
homologous DNA sequences.

Question 37 N-glycosylation in most proteins is through which amino acid


Complete
Marked out of 1 Select one: N-glycosylation is a type of covalent modification where
a. Tyrosine a carbohydrate molecule is attached to a nitrogen atom
b. Proline in the side chain of an asparagine residue in a protein.
The process occurs in the endoplasmic reticulum and
c. Glycine Golgi apparatus of eukaryotic cells and is important for
d. Asparagine protein folding, stability, and function.
e. Serine
r

Question 41 One of the following statements regarding endospore formation is correct


Complete
Marked out of 1 Select one: Endospores are a type of dormant cell produced
a. Endospores are easily stained in a Gram stain. by certain bacteria as a survival mechanism in
response to harsh environmental conditions. They
b. Endospores are for reproduction. are highly resistant to heat, desiccation, radiation,
and chemicals. When conditions become
c. Acell can produce many endospores. favorable again, the endospore can germinate
and return to its normal vegetative state.
d. Acell produces one endospore and keeps growing.
e. Endospores allow a cell to survive environmental changes.

Question 42 A clinical viral disease


Complete
Marked out of 1 Select one:
a. Is a major feature of congenital viral infections
b. Is associated with target organs in most disseminated viral infections
c. Usually follows virus infection

d. Is most frequently due to toxin production


e. Can result without infection of host cells

Many viral infections are asymptomatic or subclinical. Clinical disease, however, is often associated with viral
replication in target organs during disseminated viral infections
Question 43 One of the following resistance mechanisms describes the function of B-lactamase
Complete
Marked out of 1 Select one:
a. Target overproduction

b. Target mimicry
c. All the above
d. Drug inactivation

e. Efflux pump

Question 44 The two functions of bacterial appendages are


Complete
Marked out of 1 Select one:
a. Attachment and motility
b. Protection and motility
c. Energy reactions and synthesis
d. Attachment and protection
e. Motility and slime production

Question 45 An eight-piece segmented negative-sense RNA genome of influenza virus that


Complete encodes the HA (hemagglutinin) protein binds directly to one of the following host
Marked out of 1 cell epithelial component
Sialic acid is a type of sugar molecule that is found on
the surface of many host cell types, including epithelial
Select one:
cells in the respiratory tract. Influenza virus uses the HA
a. Uronic acid subgroups
protein to bind to sialic acid on the host cell surface,
b. Sialic acid which facilitates viral entry into the host cell and infection.
c. Chemokine receptors
d. Serine-threonine kinase receptors
e. Cytokine receptors
Macrolides are a class of antibiotic, characterized by a large macrocyclic lactone ring that can be
inactivated by macrolide phosphotransferase enzymes

Question 47 Regarding resistance to antibiotics


Complete
Marked out of 1 Select one:
a. Penicillinases cannot inactivate cephalosporins

b. Mutation of aminoglycoside binding site is its main mechanism of resistance

c. Tetracycline resistance is a marker for multidrug resistance


d. Macrolides can be inactivated by transferases
e. Resistance to antibiotics is rarely plasmid encoded

Question 48 Enteroviruses differ from rhinoviruses mainly in their:


Complete
Marked out of 1 Select one:
a. Capsid shape
b. Strandedness

c. Tropism

d. Type of nucleic acid

e. Ability to survive acidic conditions

Question 49 The following statements are correct, except


Complete
Marked out of 1 Select one:
a, Leukocidins destroy neutrophils.
b. Hyaluronidase breaks down substances between cells.

c. Kinase destroys fibrin clots.

d. Hemolysins lyse red blood cells.


e. Coagulase destroys blood clots

Question 50 Septic shock in gram + bacteria can be caused by one of the following cell wall
Complete structures
Marked out of 1
Select one:
a. Endotoxin

b. Phospholipid
c. Peptidoglycan
d. Teichoic acid

e. Capsular protein
Question 51 Lipopolysaccharide is an important cell wall component of
Complete
Marked out of 1 Select one:
a. Protoplasts

b. Mycoplasmas
c. Gram negative bacteria

d. Gram positive bacteria

e. Acid fast bacteria

Question 52 Which of the following utilizes recombination to repair double strand breaks?
Complete
Marked out of1 Select one:
a. None of the other choices is correct
b. Homologous recombination Homologous recombination is a mechanism
that repairs double-strand breaks in DNA by
c. Base excision repair using an undamaged, homologous DNA
molecule as a template.
d. Nucleotide excision repair
e. Mismatch excision repair

Question 53 The third amino acid in the tetra-peptide of peptidoglycans is always either m-
Complete diaminopimelic acid (m-DAP) or L-lysine because
Marked out of 1
Select one:
a. Have large R-side chains needed to fill space in the cell wall.
b. Do not occur naturally and are only specific to bacteria

c. Are positively charged and form salt bridge that crosslink peptidoglycan
strands.
d. Have a free amino group that allows peptide linkages between strands of
peptidoglycan.
e. Are hydrophilic.
Question 55 The ability of some microbes, such as Trypanosoma or Giardia to alter their surface
Complete molecules and evade destruction by the host's antibodies is called
Marked out of 1
Select one:
a. Lysogenic conversion.
b. Virulence.
c. Cytocidal effect.
d. Antigenic variation.
e. Cytopathic effect.

Question 56 The presence of thylakoids, phycocyanin, gas inclusions and cysts would be
Complete associated with
Marked out of 1
Select one:
a. Cyanobacteria
b. Green sulfur bacteria

c. Purple sulfur bacteria


d. Spirochetes
e. Archaea

Question 57 Resistance to Penicillin and other R-lactams is due to the following, EXCEPT
Complete
Marked out of 1 Select one:
a. Presence of an efflux pump
b. Formation of biofilms
c. Inactivation of antibiotics by I-lactamase
d. Impaired penetration of drug to target PBPs
e. Modification of target PBPs
Question 59 Each of the following antibiotics bind to the 50S subunit of the ribosome and inhibit
Complete protein synthesis, EXCEPT
Marked out of 1
Select one:
a. Clindamycin
b. Doxycycline
c. Chloramphenicol
d. Erythromycin
e. Linezolid
Question 61 A bacterial cell wall does all of the following except
Complete
Marked out of 1 Select one:
a. Itis important in bacterial identification
b. Is associated with some symptoms of disease
c. Is the site of action for some antibiotics
d. Protects the cell from phagocytosis
e. Gives shape and rigidity to the cell
Question 64 The sequence of cell envelope components in Gram-negative bacteria from cell's
Complete inside are
Marked out of 1
Select one:
a. Cell membrane, peptidoglycan layers periplasmic space
b. Cell membrane, periplasmic space, outer membrane
c. Phospholipid bilayer, porins, LPS, Techoic acids
d. Outer membrane, peptidoglycan, cell membrane
e. Cell membrane, periplasmic space, peptidoglycan outer membrane

Question 65 An example of a killed virus vaccine is


Complete The Salk poliovirus vaccine, also known as
Marked out of 1 Select one: the inactivated poliovirus vaccine (IPV), is
a. COVID-19 Pfizer vaccine made from inactivated (killed) poliovirus and
b. Enders’ measles vaccine is used to prevent poliomyelitis (polio).
c. Oka varicella-zoster vaccine

d. Salk poliovirus vaccine


e. Jeryl Lynn mumps vaccine

Question 66 A change in a single base pair in a DNA sequence that leads to a change in amino
Complete acid of the encoded protein is called
Marked out of 1
Select one:
a. Nonsense mutation

b. Point mutation

c. Missense mutation

d. Silent mutation

e. Frameshift mutation
Question 68 One of the following attributes is most likely to be a factor in the pathogenesis for a
Complete cancer chemotherapy patient whose intravenous port had to be revised following
Marked out of 1
blockage with the catheter found to contain bacterial contaminants.

Select one:
a. Possession of pili
b. Possession of IgA protease
c. Biofilm production
d. Ergosterol containing membrane
e. Peptidoglycan layer
Question 70 Each of the following structures is used for bacterial adherence, except
Complete
Marked out of1 Select one:
a. Lipoproteins
b. Fimbriae
c. Cell membrane mannose
d. Glycoproteins
e. Capsules
Question 71 Which of the following statements is not true about DNA polymerase?
Complete
Marked out of 1 Select one:
a. It is synthesized in the nucleus

b. It has proofreading activity

c. It is important in DNA replication

d. DNA polymerase can introduce and also correct errors

e. Itis required in nucleotide excision repair

Question 72 The following statements are true regarding mitochondrial proteins, EXCEPT
Complete
Marked out of 1 Select one:
a. Their targeting sequence is located at the N-terminus
b. Mitochondrial Proteins are synthesized in the cytosol
c. Most mitochondrial proteins are encoded by nuclear DNA
d. Most mitochondrial proteins are encoded by mitochondral DNA
e. The targeting sequence is removed in the mitochondrial matrix

Question 73 The antibiotic trimethoprim antibiotics works by inhibiting the synthesis of


Complete
Marked out of 1 Select one:
a. RNA
b. Protein synthesis
c. DNA

d. Peptidoglycan
e. Folic acid

Question 74 Host cell tRNAs are involved in the genome replication of


Complete
Marked out of 1 Select one:
a. Rhinovirus

b. Retroviruses

c. Influenza A virus

d. Adenoviruses

e. Respiratory syncytial virus


Question 76 One of the following is a virus that infects and lyses progenitor erythroid cells causing
Complete aplastic crises in patients with hemolytic anemia
Marked out of 1
Select one:
a. Hepatitis B virus

b. Epstein-Barr virus

c. Parvovirus B19
d. California encephalitis virus

e. Yellow fever virus

Question 77 COVID-19 is commonly diagnosed by use of one of the following diagnostic tests
Complete
Marked out of 1 Select one:
a. Real-time reverse transcription polymerase chain reaction (rRT-PCR) assay

b. Urinalysis
c. Immunofluorescent assay (IFA)

d. Viral antigen detection test


e. Virus isolation in cell culture

Question 78 One of the following viral protein is thought to induce tumours by binding to a
Complete cellular tumour suppressor protein
Marked out of 1
Select one:
a. Hepatitis B virus e protein

b. Epstein-Barr nuclear antigen proteins

c. Human immunodeficiency virus Env protein


d. Adenovirus E1A
e. Human immunodeficiency virus gag protein
Question 79 The first viral-induced defense mechanism in a nonimmune individual is the:
Complete
Marked out of 1 Select one:
a. Production of complement system

b. Synthesis of lymphokines
c. Synthesis of neutralizing antibodies
d. Generation of cytotoxic T lymphocytes
e. Production of interferon
Question 82 The following are correct statements concerning peptidoglycan, EXCEPT:
Complete
Marked out of 1 Select one:
a. It can be degraded by lysozyme
b. It is generally a site of many antibiotics
c. It is thinner in gram-positive than in gram negative cells
d. Cross-links between the tetrapeptide involve D-alanine
e. Ithas a backbone composed of alternating units of muramic acid and
acetylgiucosamine
Question 84 T-G mismatch in humans is mostly caused by
Complete
Marked out of 1 Select one:
a. Deamination of cytosine to Uracil or 5-methyl cytosine to thymine
b. Lack of repair mechanism in the cells
c. The need to have more mutants
d. Lack of enzymes which recognize the mismatch
e. A large proportion ofT and G bases in the cells

Question 85 Which of the following, pertaining to prokaryotic cell membranes is mismatched?


Complete
Marked out of 1 Select one:
a. Form a bilayer - phospholipids
b. Sterols present - mycoplasmas
c. Contain photosynthetic pigments - cyanobacteria
d. Function - regulates transport of nutrients and wastes
e. Unique hydrocarbon present - cyano bacteria

Question 86 An antimicrobial drug ability to harm the target microbe and not the host is termed
Complete as:
Marked out of 1
Select one:
a. Selective toxicity
b. Spectrum of activity
c. Mode of action

d. Therapeutic level
e. Minimum Inhibitory concentration
Question 88 Methanogens - convert CO2 and H2 gases into methane
Complete
Marked out of1 Select one:
a. Psychrophiles - adapted to very low temperatures
b. Extreme halophiles - adapted to salty habitats
c. Thermo plasmas - adapted to frozen environments
d. Hyperthermophiles - adapted to high temperatures

Question 89 An organism that respires in the presence of O2 & ferments in the absence of O2 is
Complete known as
Marked out of 1
Select one:
a. Facultative anaerobe

b. Anaerobe

c. Aerobe
d. Microaerophile
e. Facultative aerobe

Question 90 Which of the following is mismatched?


Complete
Marked out of 1 Select one:
a, Ribosomes - protein synthesis

b. Nucleoid - hereditary material


c. Cytoplasm - dense, gelatinous solution
d. Inclusions - excess cell nutrients and materials
e. Plasmids - genes essential for growth and metabolism
Question 91 The term that refers to the presence of flagella all over the cell surface is
Complete
Marked out of 1 Select one:
a. Lophotrichous

b. Amphitrichous
c. Monotrichous
d. Peritrichous
e. Atrichous

Question 92 Microbial virulence may occur due to the following, except;


Complete
Marked out of 1 Select one:
a. Production of surface adhesins and invasins

b. Production of various enzymatic activities such as hemolysins and leucocidins.


c. Lack of compartmentalized organelles
d. Presence of anatomical structures such as pili.
e. Production of exotoxins or endotoxins.

Question 93 The replication cycle of coronaviruses involves the following features, EXCEPT
Complete
Marked out of 1 Select one:
a. Replication in hepatocytes
b. The RNAs all terminate in a common 3” and produce nested set transcripts
c. They use capped cellular mRNA's
d. Takes advantage of recombination with the long RNA genome
e. They are not highly mutable
Question 95 All of the following structures contribute to the ability of pathogenic bacteria to cause
Complete disease except
Marked out of 1
Select one:
a. Outer membrane of gram negative cell walls
b. Slime layer

c. Capsule
d. Fimbriae
e. Inclusions

Question 96 Fluoroquinolone antibiotics, act by inhibiting


Complete
Marked out of 1 Select one:
a. Folic acid synthesis
b. Prostaglandin synthesis
c. Protein synthesis
d. DNA synthesis
e. RNA synthesis

Question 97 One of the following is a target site of action of the antibiotic vancomycin
Complete The bactericidal action of vancomycin results primarily from
Marked out of 1 Select one: inhibition of cell-wall biosynthesis. Specifically, vancomycin
prevents incorporation of N-acetylmuramic acid (NAM)- and
a. Cell membrane integrity
N-acetylglucosamine (NAG)-peptide subunits from being
b. DNA synthesis incorporated into the peptidoglycan matrix, which forms the
major structural component of Gram-positive cell walls.
c. RNA synthesis

d. Cross-linking of peptidoglycan
e. Protein synthesis via the 50S ribosomal subunit

Question 98 What is the percentage of guanine in a double-stranded RNA genome isolated from a
Complete virus in the stool of a child with gastroenteritis whose uracil content is 15%?
Marked out of 1
Select one:
a. 25

b. 20
c. 85

d. 35
e.15
Question 99 The following statements are true regarding the mode of action of antibacterial
Complete drugs, except:
Marked out of 1
Select one:
a. Tetracyclines are bacteriostatic drugs that inhibit protein synthesis by blocking
tRNA binding.

b. Quinolones, such as ciprofloxacin, act by inhibiting the DNA gyrase of bacteria.

c. Erythromycin is a bactericidal drug that disrupts cell membranes by a


detergent-like action.

d. Vancomycin acts by inhibiting peptidoglycan synthesis.

e. Cephalosporins are bactericidal drugs that inhibit the transpeptidase reaction


and prevent cell wall synthesis

Question 100 Trp operon


Complete
Marked out of 1 Select one:
a, Regulates expression of genes responsible for arabinose metabolism
b. Regulates the expression of genes responsible for tryptophan metabolism
c. Activates utilization of tryptophan in prokaryotic cells
d. Inhibits expression of genes required for tryptophan catabolism

e. Controls the breakdown of lactose


END OF YEAR 2021
EXTRACTED
QUESTIONS

Only a foolish man can go after a leopard with his bare hands.
Revise questions and know how they're set, don't just read slides€>

Compiled by Jitu, 2022

Answered by Haron Sami


i & Courses & Health Sciences & Pharmacy # Bachelor of Pharmacy (B.Pharm) { Biochemistry Topic5 &

MBCHB BPHARM &BDS LEVEL |! 2020 /2021 FINAL EXAMS SEPTEMBER 2021

Started on Wednesday, 8 September 2021, 9:00 AM


State Finished
Completed on Wednesday, 8 September 2021, 11:44 AM
Time taken 2 hours 44 mins

Question 1 The following proteins/enzymes are encoded for by the lac operon genes, EXCEPT;
Complete
Marked out of 1 Select one: The lac operon contains three genes: lacZ, lacY, and lacA.
e a.Hexokinase These genes are transcribed as a single mRNA, under control
of one promoter. Genes in the lac operon specify proteins that
b. Lac repressor help the cell utilize lactose
c. B-galactosidase

d. Galactoside permease

e. Galactoside transacetylase
Question 4 Microbial virulence may occur due to the following, EXCEPT
Complete
Marked out of 1
Select one:
a. Production of surface adhesins and invasins

b. Production of exotoxins or endotoxins.

c. Presence of anatomical structures such as pili.


d. Production of various enzymatic activities such as hemolysins and leucocidins.

* e.Lack of compartmentalized organelles

Question 5 The following are correct statements concerning peptidoglycan, EXCEPT:


Complete
Marked out of 1
Select one:
a. Cross-links between the tetrapeptide involve D-alanine

b. It can be degraded by lysozyme

c. It has a backbone composed of alternating units of muramic acid and


acetylglucosamine

d. It is generally a site of many antibiotics

* — e.ltis thinner in gram-positive than in gram negative cells


Question 9 The following compounds are all DNA mutagens, EXCEPT
Complete
Marked out of 1 Select one:
e a. Thymine

b. Ethidium bromide

c. Methyl methanesulfonate
d. Proflavin

e. N-methyl-N'-nitro-N-nitrosoguanidine

Question 10 One of the following is a characteristic of bacterial endotoxins:


Complete
Marked out of 1 Select one:
a. May be converted to toxoids.
b. Can act as antibiotics

* c. Are lipopolysaccharide present on bacterial cell walls

d. Are tissue-destroying enzymes.


e. Are only produced by Gram-negative bacteria.
Question 12 An example of genetic disorder associated with mutation in mitochondria is:
Complete
Marked out of 1 Select one:
a. Sickle cell disease

b. Leber’s hereditary optic neuropathy

© c. Parkinson's disease

d. Albinism

e. Achondroplasia
Question WZ In a eukaryotic cell, DNA is found in:
Complete
Marked out of 1
Select one:
a. Nucleus only

e b. Nucleus and Mitochondria only

c. Mitochondria only
d. Chloroplast only
e. Nucleus, mitochondria and chloroplast
Question 19 The first viral-induced defense mechanism in a nonimmune individual is the:
Complete
Marked out of 1
Select one:
® a. Production of interferon

b. Synthesis of lymphokines
c. Synthesis of neutralizing antibodies

d. Production of complement system

e. Generation of cytotoxic T lymphocytes


Question Zt The replication cycle of coronaviruses involves the following features, EXCEPT
Complete
Marked out of 1 Select one:
a. They use capped cellular mRNAs

b. The RNAs all terminate in a common 3’ and produce nested set transcripts

c. They are not highly mutable

d. Takes advantage of recombination with the long RNA genome

e — e. Replication in hepatocytes

Question 22 Each of the following antibiotics bind to the 50S subunit of the ribosome and inhibit protein
Complete synthesis, EXCEPT
Marked out of 1
Select one:
a. Linezolid

b. Clindamycin

c. Chloramphenicol

d. Erythromycin
e e. Doxycycline
Question 28 One of the following cell wall components is most associated with the induction of septic
Complete shock in gram-positive bacteria
Marked out of 1
Select one:
a. Phospholipid

b. Peptidoglycan

c. Capsular protein
e d. Teichoic acid

e. Endotoxin
Question 34 Bacteriophages are readily counted by the process of:
Complete
Marked out of 1
Select one:
a. ELISA

b. Tissue cell culture

c. Plaque assays

e d. Electron Microscopy
e. Immunoassays

Question 37 Resistance to Penicillin and other R-lactams is due to the following, EXCEPT
Complete
Marked out of 1 Select one:
a. Modification of target PBPs
b. Impaired penetration of drug to target PBPs

c. Inactivation of antibiotics by |-lactamase

d. Presence of an efflux pump

* — e. Formation of biofilms

Question 38 Regarding resistance to antibiotics


Complete
Marked out of 1 Select one:
a. Penicillinases cannot inactivate cephalosporins
b. Resistance to antibiotics is rarely plasmid encoded
c. Tetracycline resistance is a marker for multidrug resistance

e d. Mutation of aminoglycoside binding site is its main mechanism of resistance


e. Macrolides can be inactivated by transferases

The macrolides are a class of antibiotic, characterized by a large macrocyclic lactone ring that can be inactivated by macrolide
phosphotransferase enzymes
Question 40 Passive immunization is available for protection from one of the following viruses
Complete
Marked out of 1 Select one:
a. Influenza A virus

b. Parainfluenza type 2 virus

c. Rubella virus

d. Hepatitis A virus

e — e. Yellow Virus

Question 41 Zellweger syndrome


Complete
Marked out of 1 Select one:
a. Is due to loss of mitochondrial functions

b. Doesn't cause any developmental defects


c. Is not a genetic disorder
d. Is not associated with protein targeting

* e. ls due to mutation in any of the PEX genes


cr

Question 44 The approximate size of human genome in base-pairs is


Complete
Marked out of 1
Select one:
a. 2.5 billion

b. 4 billion

® c.3billion

d. None of the other choices

e. 3.5 billion
Question 49 One of the following viral proteins is thought to induce tumors by binding to a cellular tumor
Complete suppressor protein
Marked out of 1
Select one:
a. Human immunodeficiency virus Env protein

b. Human immunodeficiency virus gag protein

c. Epstein-Barr nuclear antigen proteins

ed. Adenovirus E1A

e. Hepatitis B virus e protein


Question 52 An example of a killed virus vaccine is
Complete
Marked out of 1
Select one:
The Salk poliovirus vaccine, also known as the inactivated poliovirus
a. COVID-19 Pfizer vaccine vaccine (IPV), is made from inactivated (killed) poliovirus and is used
to prevent poliomyelitis (polio).
b. Enders’ measles vaccine

* — c. Salk poliovirus vaccine


d. Oka varicella-zoster vaccine

e. Jeryl Lynn mumps vaccine


Question 54 One of the following attributes is most likely to be a factor in the pathogenesis for a cancer
Complete
chemotherapy patient whose intravenous port had to be revised following blockage of a
catheter found to contain bacterial contaminants.
Marked out of 1

Select one:
a. Peptidoglycan layer
e _ b. Biofilm production

c. Possession of pili

d. Ergosterol containing membrane


e. Possession of IgA protease
Question 55 A large majority of the sequences in human genome comprises of,
Complete
Marked out of 1 Select one: https://www.nature.com/articles/s41576-021-00354-8
* a. Regulatory sequences

b. Transposable elements
c. None of the other choices is correct

d. Repetitive sequences

e. Unique coding sequences


Question 61 A clinically important viral disease Many viral infections are asymptomatic or subclinical. Clinical
disease, however, is often associated with viral replication in target
Complete organs during disseminated viral infections
Marked out of 1 Select one:
a. Is most frequently due to toxin production

b. Can result without infection of host cells

c. ls a major feature of congenital viral infections


e d.ls associated with target organs in most disseminated viral infections

e. Usually follows virus infection

Question 62 One of the following resistance mechanisms describes the mode of action conferred by
Complete bacterial B-lactamase
Marked out of 1
Select one:
e — a. Drug inactivation
b. Target overproduction

c. Biofilm formation

d. Target mimicry
e. Efflux pump
Question 66 In cell cycle, DNA synthesis occurs in:
Complete
Marked out of 1
Select one:
a. G1 and G2 phases
b. G1 -phase

c. G2 phase

d. Cytokinesis phase
e e.Sphase
Question 69 Each of the following statement is true concerning the antibiotic penicillin, EXCEPT:
Complete
Marked out of 1
Select one:
a. An intact B-lactam ring of penicillin is required for its activity.

e _ b. Penicillin is a bacteriostatic drug because autolytic enzymes are not activated.

c. Penicillin structure is analogous to a dipeptide of alanine, a component of


peptidoglycan.
d. Penicillin inhibits trans-peptidases, which are important for cross-linking
peptidoglycan.

e. Resistance to penicillin occurs due to reduced affinity of the transpeptidases.


r-

Question 71 N-acetylglucosamine and N-acetylmuramic acid polymers are major components of bacterial
Complete
Marked out of 1 Select one:
a. Capsule

b. Glycocalyx
c. Teichoic acids

d. Lipopolysaccharide

° e. Cell walls
Question 75 The antibiotic vancomycin targets one of the following processes in the course of its action
Complete

Marked out of 1 Select one:


* a. Cross-linking of peptidoglycan

b. Protein synthesis via the 50S ribosomal subunit


c. DNA synthesis

d. Cell membrane integrity

e. RNA synthesis
Question 78 Breast cancer can be caused by a mutation in one of the following genes
Complete
Marked out of 1
Select one:
a. PMS2

b. XP-G

e c.BRCA1

d. MLH1
e. MSH2
Question 80 The following statements are true regarding the mode of action of antibacterial drugs,
Complete
EXCEPT:
Marked out of 1
Select one:
a. Cephalosporins are bactericidal drugs that inhibit the transpeptidase reaction and
prevent cell wall synthesis

b. Vancomycin acts by inhibiting peptidoglycan synthesis.

c. Quinolones, such as ciprofloxacin, act by inhibiting the DNA gyrase of bacteria.


d. Tetracyclines are bacteriostatic drugs that inhibit protein synthesis by blocking tRNA
binding.

ee. Erythromycin is a bactericidal drug that disrupts cell membranes by a detergent-like


action.

Question 81 The following statements are true concerning bacterial spores, EXCEPT:
Complete
Marked out of 1 Select one:
a. They can be killed by being heated to 1210C for 15 minutes

b. Are more likely to survive treatment with disinfectants

c. They are formed by gram-positive rods


e d. Their survival ability is based on their enhanced metabolic activity

e. They contain much less water than bacterial cells


co

Question 94 The following are reverse transcriptase inhibitors, EXCEPT


Complete
Marked out of 1 Select one:
S a. Ritonavir

b. Azidothymidine

c. Tenofovir

d. Amethopterin

e. Zidovudine
Question 97 An organism that respires in the presence of 02 & ferments in the absence of 02 is known as
Complete
Marked out of 1 Select one:
a. Facultative anaerobe

e _ b. Facultative aerobe

c. Microaerophile

d. Anaerobe

e. Aerobe

Question 98 The aminoglycosides which are very active group of antibacterial agents, particularly against
Coles Gram-negative bacilli acts through
Marked out of 1
Select one:
a. Interference with bacterial folic acid metabolism

b. Inhibition of bacterial DNA gyrase

c. Inhibition of bacterial cell wall synthesis


e d. Inhibition of protein synthesis

e. Disruption of cytoplasmic membrane function


Question 1 02 The third amino acid in the tetra-peptide of peptidoglycans is always either m-diaminopimelic
Complete
acid (m-DAP) or L-lysine because the two amino acids
Marked out of 1
Select one:
a. Do not occur naturally and are only specific to bacteria

b. Are positively charged and form salt bridge that crosslink peptidoglycan strands.

c. Have large R-side chains needed to fill space in the cell wall.
d. Are hydrophilic.

e e. Have a free amino group that allows peptide linkages between strands of
peptidoglycan.
"4

Question 104 The following statements are correct concerning some bacterial virulent factors, EXCEPT
Complete
Marked out of 1
Select one:
e a. Coagulase destroys blood clots
b. Hyaluronidase breaks down substances between cells

c. Kinase destroys fibrin clots

d. Leukocidins destroy neutrophils


e. Hemolysins lyse red blood cells
cr

Question 106 Frameshift mutation can be due to


Complete
Marked out of 1 Select one:
a. Duplications

b. Deletions

c. Indels
d. Insertions

* — e.All the given choices are correct

,
Question 107 The following statements are true regarding chain termination method of DNA sequencing,
Complete EXCEPT
Marked out of 1
Select one:
a. It allows the amplified DNA to be sequenced using either forward or reverse primer

e _ b.Itrequires a pair of primers

c. It requires a single primer

d. It requires chain terminators

e. It requires dideoxynucleoside triphosphates that are labeled with fluorescent tags


Question 11 3 | One of the following immunizations is administered immediately after birth
Complete
Marked out of 1 Select one:
a. Haemophilus influenzae type b vaccine

e _ b. Diphtheria-pertusis-tetanus (DPT) vaccine


c. HIV Vaccine

d. Hepatitis B vaccine

e. Oral Poliovirus
Question 115 An antimicrobial drug ability to harm the target microbe and not the host is termed as:
Complete
Marked out of 1
Select one:
a. Spectrum of activity

* _ b. Selective toxicity

c. Mode of action
d. Minimum Inhibitory Concentration (MIC)

e. Therapeutic level
Question 1 17 A common structural component found in all viruses is:
Complete
Marked out of 1 Select one:
a. DNA

b. The envelope
e c.Capsid

d. Spikes

e. Tail fibers
Question 128 The following statements are true about RNA polymerase in Mycobacterium tuberculosis
Complete EXCEPT
Marked out of 1
Select one:
a. It recognizes and binds the promoter for transcription to start
b. It is a drug target in the treatment of tuberculosis

c. Its | subunit catalyzes the synthesis of RNA

d. It is a multiple-subunit protein
e e.ltis activated by Rifampicin in the synthesis of RNA
Question 134 An important defense function of cytotoxic T lymphocytes in viral infection is to:
Complete
Marked out of 1
Select one:
a. Neutralize free virus particles

b. Block cell respiration

c. Lyse viral capsid


e d.Lyse virus infected cells

e. Fragment viral nucleic acid by nucleases


Question 138 The following statements are true about the drug chloramphenicol, EXCEPT
Complete
Marked out of 1
Select one:
* a.ltactivates the process of polypeptide synthesis in bacteria

b. It does not affect protein synthesis in humans

c. Itis an antibiotic
d. It inhibits protein synthesis in prokaryotes

e. It inhibits protein synthesis in bacteria by blocking peptidyl transferase activity


BACTERIA
BIOCHEMISTRY
MOCK CAT 2020

with answers

Compiled by Jitu, 2022


Search Courses

My courses Biochemistry Topic1

MBCHB BPHARM & BDS LEVEL Il AUGUST 2020 ONLINE MOCK CAT

Startedon Tuesday, 11 August 2020, 6:18 PM


State Finished
Completedon Tuesday, 11 August 2020, 7:40 PM
Time taken 1 hour 22 mins
Marks 28.00/40.00
Grade 7,00 out of 10.00 (70%)

Question 1 39. Flagella and pili are made of


Correct
Mark 1.00 out of Select one:
ee a. protein
b. carbohydrates
c. lipids
d. nucleic acids

The correct answer is: protein

Question 2 9. Which resistance mechanisms is the most nonspecific to a particular class of


Correct antimicrobials?
Mark 1.00 out of
1.00 Select one:
a. Target modification
b. Drug modification
c. Efflux pump
d. Target mimicry

The correct answer is: Efflux pump

Question 3 ! 27. Regarding the Gram stain, which one of the following is the MOST accurate.
Correct
Mark 1.00 out of Select one:
100 a. If you forget to stain with the red dye (safranin or basic fuchsin), both
gram-positive bacteria and gram-negative bacteria will appear blue.
b. If you forget to heat-fix, both gram-positive bacteria and gram-negative
bacteria will appear blue.
c. One reason why bacteria have a different color in this stain is because the
gram-positive bacteria have lipid in their membrane, whereas gram-negative
bacteria do not.

d. After adding crystal violet and Gram’s iodine, both gram-positive bacteria
and gram-negative bacteria will appear blue. ./

The correct answer is: After adding crystal violet and Gram’s iodine, both gram-
positive bacteria and gram-negative bacteria will appear blue.
Question 4 32. Which of the following resistance mechanisms is the most nonspecific to a
pacticulanclass
of antimicrobials?
Mark 1.00 out of
1,00 Select one:
a, target modification

b. efflux pump

c. target mimicry
d. drug modification

The correct answer is: efflux pump

Question 5 25. All the statements on the structure and chemical composition of bacteria are
Correct correct, except :
Mark 1.00 out of
1.00 Select one:
a. Some gram-positive cocci contain a layer of teichoic acid external to the
peptidoglycan
b. Some mycoplasmas contain pentaglycine in their peptidoglycan

c. Some gram-negative rods contain lipid A in their cell wall


d. Some gram-positive rods contain dipicolinic acid in their spores

The correct answer is: Some mycoplasmas contain pentaglycine in their


peptidoglycan

Question 6 38. A shiny, sticky colony of Streptococcus pneumoniae is likely to be


Correct

Mark 1.00 out of Select one:


1,00 a. nonencapsulated and nonpathogenic

b. encapsulated and nonpathogenic

¢c. nonencapsulated and pathogenic

d. encapsulated and pathogenic

The correct answer is: encapsulated and pathogenic

Question 7 30. Each one of the following is a characteristic of a plasmid, except


Incorrect

Mark 0.00 out of Select one:


1.00 a. The plasmid genes are not essential for the survival of bacteria.
b. They are composed of RNA.
c. They are circular. &
d. They are double-stranded.

The correct answer is: They are composed of RNA.

Question 8 3. Endotoxins have all but one of the properties given below:
Correct

Mark 1,00 out of Select one:


1,00 a. Cannot be toxoided
b. Produced by gram positive bacteria /
c. Not secreted outside the cell
d. Produced by gram negative bacteria

The correct answer is: Produced by gram positive bacteria


Question 9 37. Aslippery outer covering in some bacteria that protects them from host cell
2
Mark 1.00 out of
1,00 Select one:
a. capsule ./
b. cell wall
c. peptidoglycan

d. flagellum

The correct answer is: capsule

Question 10 33. Which of these antibiotics inhibit cell wall synthesis?


Correct

Mark 1.00 out of Select one:


1.00 a. Tetracycline
b. Trimethoprim

c. Penicillin

d. Fluoroquinolones

The correct answer is: Penicillin

Question 11 22. Plaques are:


Correct
Mark 1.00 out of Select one:
1,00 a. Bacterial colonies on agar /
b. Virus colonies on agar.
c. Stained areas in a cell culture indicating cells infected by a virus.

d. Clear areas in a lawn of cultured cells caused by virus infection.

The correct answer is: Bacterial colonies on agar

Question 12 16. Protective mechanisms used by bacteria to survive in the host may include
Incorrect

Mark 0.00 out of Select one:


1.00 a. M protein

b. all of the above


c. Capsules &
d. Various enzymes

The correct answer is: all of the above

Question 13 13. Bacterial fimbriae present on the outer cell surface are used for:
Incorrect

Mark 0.00 out of Select one:


1.00 a. Adherence to surfaces *
b. Sexual reproduction

c. Cellular motility
d. Adherence and exchange of genetic information

The correct answer is: Sexual reproduction


Question 14 29. Each of the following statements concerning Mycobacterium tuberculosis is
correct EXCEPT
Mark 0.00 out of
1,00 Select one:
a. M. tuberculosis appears as a red rod in Gram-stained specimens.

b, M. tuberculosis resists decolorization with acid alcohol after staining with


carbolfuchsin.
c. M. tuberculosis has a large amount of mycolic acid in its cell wall. *
d. M. tuberculosis appears as a red rod in acid-fast stained specimens.

The correct answer is: M. tuberculosis appears as a red rod in Gram-stained


specimens.

Question 15 17. Which of the following statement regarding endotoxin is correct?


Correct

Mark 1.00 out of Select one:


1.00 a. Are found in both gram positive and gram negative bacterial cell wall

b, All have the same effect regardless of bacterial source

c. None of the above statement is correct

d. Are part of the cell wall of gram positive bacteria only

The correct answer is: None of the above statement is correct

Question 16 6. Each of the following statements concerning endotoxins is correct EXCEPT:


Correct

Mark 1.00 out of Select one:


1.00 a. Endotoxins are located in the cell wall
b. The antigenicity of somatic (O) antigen is due to repeating
oligosaccharides
c. Endotoxins are found in most gram-positive bacteria /
d. The toxicity of endotoxins is due to the lipid portion of the molecule

The correct answer is: Endotoxins are found in most gram-positive bacteria

Question 17 35. Which of the following applies to hyaluronidase?


Incorrect

Mark 0.00 out of Select one:


1.00 a. Itis an example of an adhesin.

b. It is produced by immune cells to target pathogens. *


c. It acts as a spreading factor.
d, It promotes blood clotting.

The correct answer is: It acts as a spreading factor.

Question 18 19. The following are intrinsic bacterial antibiotic resistance mechanisms, except
Incorrect :
Mark 0.00 out of
1,00 Select one:
a. Low permeability envelope *
b, Plasmid-mediated resistance
c. Inherent Efflux Pump

d. Mutations

The correct answer is: Mutations


Question 19 14. Exotoxins are typically

Mark 1.00 out of Select one:


1,00 a. Carbohydrates

b. Lipopolysaccharides
c. Proteins /

d, Lipids

The correct answer is: Proteins

Question 20 23. The identification of bacteria by serologic tests is based on the presence of
Correct specific antigens. Which one of the following bacterial components is LEAST
Mark 1.00 out of likely to contain useful antigens?
1.00
Select one:
a. Flagella
b. Cell wall
c. Capsule

d, Ribosomes

The correct answer is: Ribosomes

Question 21 36. Phospholipases are enzymes that do which of the following?


Correct

Mark 1.00 out of Select one:


1.00 a. degrade nucleic acid to promote spread of pathogen
b. promote pathogen spread through connective tissue.

c. degrade cell membranes to allow pathogens to escape phagosomes ./


d, degrade antibodies

The correct answer is: degrade cell membranes to allow pathogens to escape
phagosomes

Question 22 4. Which of the following combinations would most likely contribute to the
Correct development of a superinfection?
Mark 1.00 out of
1.00 Select one:
a. Short-term use of broad-spectrum antimicrobials
b. Short-term use of narrow-spectrum antimicrobials
c. Long-term use of broad-spectrum antimicrobials

d. Long-term use of narrow-spectrum antimicrobials

The correct answer is: Long-term use of broad-spectrum antimicrobials


Question 23 24. Each of the following statements concerning the resistance of bacteria to
anlimiccobialdeesis
coccoct EXCEPT.
Mark 0.00 out of
1,00 Select one:
a, Resistance to tetracycline is known to be due to an enzyme that
hydrolyzes the ester linkage.
b. Resistance to chloramphenicol is known to be due to an enzyme that
acetylates the drug.
c. Resistance to penicillin is known to be due to reduced affinity of
transpeptidases. *
d, Resistance to penicillin is known to be due to cleavage by beta-lactamase.

The correct answer is: Resistance to tetracycline is known to be due to an


enzyme that hydrolyzes the ester linkage.

Question 24 12. Which of the following BEST describes the mode of action of endotoxin?
Correct

Mark 1.00 out of Select one:


1.00 a. Causes the release of tumor necrosis factor /
b. Inactivates elongation factor 2

c. Blocks release of acetylcholine

d. Degrades lecithin in cell membranes

The correct answer is: Causes the release of tumor necrosis factor

Question 25 31. Which of these statements is FALSE?


Correct

Mark 1.00 out of Select one:


1.00 a. Isoniazid is a relatively weak drug that only requires combination with
stronger drugs. /

b. Anti-mycobacterial drugs must be able to penetrate cells and tissues well.


c. Mycobacteria cause tuberculosis and leprosy.
d, Ethambutol is a drug that specifically inhibits the synthesis of
mycobacterial cell walls.

The correct answer is: Isoniazid is a relatively weak drug that only requires
combination with stronger drugs.

Question 26 40. The bacterial envelope includes all of the following structures except
Correct

Mark 1.00 out of Select one:


1.00 a. cell wall

b. capsule

c. endospore ./

d. cell membrane

The correct answer is: endospore


Question 27 20. When flagella are distributed all around a bacterial cell, the arrangement is
called
Mark 1.00 out of
1,00 Select one:
a, Random

b. Encapsulated

c. Peritrichous

d. Polar

The correct answer is: Peritrichous

Question 28 18. Differences between eukaryotic and prokaryotic cells include all of the
Incorrect following except
Mark 0.00 out of
1.00 Select one:
a. Prokaryotic cells have more complex cell walls

b. Prokaryotic cells have no genetic material


c. Eukaryotic cells have mitochondria *
d, Eukaryotic cells have cilia and flagella with complex structure

The correct answer is: Prokaryotic cells have no genetic material

Question 29 10. One of the following statements concerning the surface structures of
Correct bacteria is false
Mark 1,00 out of
1.00 Select one:
a, Pili mediate the interaction of bacteria with mucosal epithelium

b. Both gram-negative rods and cocci have lipopolysaccharide “endotoxin” in


their cell wall
c. Bacterial flagella are nonantigenic in humans because they closely
resemble human flagella in chemical composition /
d. Polysaccharide capsules retard phagocytosis

The correct answer is: Bacterial flagella are nonantigenic in humans because
they closely resemble human flagella in chemical composition

Question 30 1. Which of the following is true about cell wall of gram-positive bacteria?
Correct

Mark 1,00 out of Select one:


1,00 a. It contains teichoic acids

b. It is thicker than that associated with gram-negative bacteria


c. All of these

d. It consists of multiple layers

The correct answer is: All of these

Question 31 8. Which of the following antimicrobials inhibits the activity of DNA gyrase?
Correct

Mark 1.00 out of Select one:


1.00 a. Nalidixic acid y

b, Polymyxin B
c. Rifampin

d. Clindamycin

The correct answer is: Nalidixic acid


Question 32 7. Which of the following does not bind to the 50S ribosomal subunit?

Mark 1.00 out of Select one:


1,00 a. Macrolides

b. Tetracyclines ./

c. Chloramphenicol

d, Lincosamides

The correct answer is: Tetracyclines

Question 33 5. Which of the following is NOT one of the groups of beta-lactam


Correct antimicrobials?
Mark 1.00 out of
1.00 Select one:
a. Glycopeptides

b. Penicillins
c. Monobactams
d. Cephalosporins

The correct answer is: Glycopeptides

Question 34 21. Endospore are all of the following except as compared to


Incorrect vegetative cells.
Mark 0.00 out of
1,00 Select one:
a. More resistant to staining

b. More likely to survive treatment with disinfectants *


c. More resistant to temperature changes

d. More likely to die in nutritionally poor conditions

The correct answer is: More likely to die in nutritionally poor conditions

Question 35 2. The penicillins act by :


Correct

Mark 1.00 out of Select one:


1.00 a. Interfering with bacterial cell wall synthesis. ./

b. Inhibition of protien synthesis


c. Phagocytic action.

d, Inhibition of neuclic acid synthesis

The correct answer is: Interfering with bacterial cell wall synthesis.

Question 36 15. Each of the following statements concerning bacterial and human cells is
Incorrect correct EXCEPT:
Mark 0.00 out of
1.00 Select one:
a. Bacterial cells possess peptidoglycan, whereas human cells do not.

b. Bacteria are prokaryotic (ie, they have one molecule of DNA, are haploid,
and have no nuclear membrane), whereas human cells are eukaryotic (ie, they
have multiple chromosomes, are diploid, and have a nuclear membrane)
c. Bacteria derive their energy by oxidative phosphorylation within
mitochondria in a manner similar to human cells &

d. Bacterial and human ribosomes are of different sizes and chemical


composition

The correct answer is: Bacterial and human ribosomes are of different sizes and
chemical composition
Question 37 11. The following bacterial surface structures exhibit antigenic diversity, EXCEPT:

Mark 0.00 out of Select one:


1.00 a. Pili X
b. Flagella
c. Capsules

d. Peptidoglycan

The correct answer is: Peptidoglycan

Question 38 28. Which of the following statements is most correct about the differential
Correct Gram stain?
Mark 1.00 out of
1.00 Select one:
a. Crystal violet differentially stains Gram positive cells.

b. Safranin differentially stains Gram negative cells.


c. Gram's iodine differentially stains Gram positive cells.
d. Acetone/Alcohol differentially destains Gram negative cells.

The correct answer is: Acetone/Alcohol differentially destains Gram negative


cells.

Question 39 26. Each of the following statements concerning exotoxins is correct EXCEPT:
Incorrect

Mark 0.00 out of Select one:


1.00 a. Exotoxins are less toxic than the same amount of endotoxins

b. Exotoxins are polypeptides

c. Exotoxins can be converted to toxids *


d. Exotoxins are more easily inactivated by heat than are endotoxins

The correct answer is: Exotoxins are less toxic than the same amount of
endotoxins

Question 40 34. Which of the following would be a virulence factor of a pathogen?


Correct

Mark 1.00 out of Select one:


1.00 a. A surface protein allowing the pathogen to bind to host cells «/
b. A secondary host the pathogen can infect

c. the ability to form a provirus


d. Asurface protein the host immune system recognizes

The correct answer is: A surface protein allowing the pathogen to bind to host
cells
MID SEMESTER 2
2020 EXTRACTED
QUESTIONS

Compiled by Jitu, 2022

Answered by Haron Sami


Search Courses

My courses Biochemistry Topic! HBC 200 UPC 202 VBC 200 LEVEL BIOCHEMISTRY

Startedon Thursday, 13 August 2020, 2:31 PM


State Finished
Completed on Thursday, 13 August 2020, 3:31 PM
Time taken 1 hour

Question 1 24, A cell wall can increase a bacterium’'s virulence because cell wall lipid A
Complete
Marked out of 1 Select one:
a, Destroys host tissues.

b. Resists phagocytosis.
c. Is toxic,
d. Helps the bacterium attach.

e. All bacteria have a cell wall and all are not pathogenic; therefore, cell walls
do not contribute to virulence

Question 2 1, The likely pathogenesis factor of patient with bacterial contaminated &
Complete blocked catheter is
Marked out of 1
Select one:
Biofilms are communities of microorganisms that
» a. Biofilm production
attach to surfaces and produce a slimy film that can
b. Possession of pili
protect them from antibiotics and the immune system.
c. Ergosterol containing membrane
Biofilms can form on medical devices such as
d, Peptidoglycan layer
catheters and can lead to infections
e, Possession of IgA protease

Question 3 6. All the following statements are true regarding exotoxins, except
Complete
Marked out of 1 Select one:
a, They are composed of proteins.

b. They have specific methods of action.

c. They are more potent than endotoxins.


d, They are not destroyed by heat.

e, They are produced by gram-positive bacteria.

Question 4 9. One of the following bacterial structures is involved in surface adherence


Complete
Marked out of 1 Select one:
a. Teichoic acid
b. Common pili

c. Capsule
d, O-specific side chain
e. Lipopolysaccharide
Question 6 27. Microorganisms might be virulent due to the following , except;
Complete
Marked cut of 1 Select one:
a. Presence of anatomical structures such as pili.
b. Production of various enzymatic activities such as hemolysfns an
leucocidins. genes
c. Production of surface adhesins and invasins

d. Production of exotoxins or endotoxins.

e. Lack of compartmentalized organelles


Question 8 18. One of the following statements regarding endospore formation is correct
Complete
Marked cut of 1 Select one: Endospores are a type of dormant cell produced by
a. A cell can produce many endospores. certain bacteria
a as a survival mechanism in
response to harsh environmental conditions. They
b. Endospores are easily stained in a Gram stain. are highly resistant to heat, desiccation, radiation,
c. Endospores are for reproduction. and chemicals. When conditions become favorable
again, the endospore can germinate and return to
d. A cell produces one endospore and keeps growing.
its normal vegetative state.
e. Endospores allow a cell to survive environmental changes.

Question 9 16. Prokaryotic cells can be distinguished by the following characteristics, except
Complete
Marked cut of 1 Select ane: a
a. They have cell walls containing peptidoglycan.
b. They lack membrane-enclosed organelles. ge
c. Their DNA is not enclosed within a membrane.
d. Their DNA is not associated with histones.
e. None of the above.
Question 10 22. Fimbriae (or pili):
EAA
Marked cut of 1 Select one:
a. Assist bacteria to attach to surfaces.

b. Are functionally like cilia of protozoans and other single celled eukaryotes.
c. Are used by bacteria to sense changes in nutrient concentration.
d. Enable bacteria to move through fluids.
e. Are used by bacteria as osmoregulatory sensors.
Question 12 20. L-lysine & m-diaminopimelic acid occur in the 3rd position of murein tetra-
Complete peptide because
Marked out of 1
Select one:
a. Are positively charged and form salt bridge that crosslink peptidoglycan We
strands.

b. Have large R-side chains needed to fill space in the cell wall.
c. Do not occur naturally and are only specific to bacteria
d. Have a free amino group that allows peptide linkages between strands of
peptidoglycan.
e. Are hydrophilic.
Question 14 29. Ribosomal resistance occurs with the following groups of antibiotics
Complete
Marked cut of 1 Select one:
a. Penicillin —

b. Sulphonamides ae
c. Fluoroquinolones

d. Trimethoprim
e. Macrolides
Question 15 14. Place the steps of the Gram stain in the correct order of staining: 1-Alcohol-
comprete vio
2.Crystal2Safran
acetone dlodine
ioler
Markec cut of 1
Select one:
a. 2-4-1-3

b. 1-2-3-4

c. 4-3-2-1

d. 2-1-4-3

e, 1-3-2-4

Question 16 23. Endotoxins are


Complete

Markee cut of 1 Select one:


a. Specific in their method of action.
b. Excreted from the cell.
c. Associated with gram-positive bacteria.
d. Part of the gram-negative cell wall.
e. A-B type toxins.
Question 21 11. Which of the following components are found in the cell walls of gram-
Complete positive bacteria but not gram-negative bacteria?
Markec cut of 1
Select one:
a. Outer membrane

b. Peptidoglycan
c. Teichoic acid
d. Cytoplasmic membrane
e, Lipopolysaccharide
3. One of the following antibiotics DOES NOT bind to the SOS subunit of the
Complete ribosome
Marked cut of 1
Select one:
a. Linezolid.
b. Chloramphenicol. wt
c. Erythromycin.
d. Clindamycin.
e. Doxycycline.
Question 25 4. Resistance to Penicillin and other o-lactams is due to
Aenean

Marked out of 1 Select one:


a. All of the above

b. Modification of target PBPs


c. Inactivation of antibiotics by I-lactamase
d. Impaired penetration of drug to target PBPs
e. Presence of an efflux pump
Question 29 25. The following are all pathogen virulence factors, except
Complete
Marked cut of 1 Select one:
a. All of the above contribute to a pathogen's virulence.
b. Enzymes
c. Numbers of microorganisms that gain access to a host
d. Toxins
e. Cell wall
Question 31 2. All of the following are used by bacteria to attach to host cells EXCEPT
Complete
Marked cut of 1 Select one:
a. Ligands
b. M protein
c. A-B toxins

d. Capsules
e. Fimbriae

Question 32 26. Beta-lactams antibiotics work by inhibiting the synthesis of


Complete
Marked cut of 1 Select one:
a. DNA

b. Peptidoglycan

c. RNA

d. Protein

e. Folic acid
Question 37 33. In cell cycle, DNA synthesis occurs in:
Complete
Marked cut of 1 Select one:
a. G1 and G2 phases
b. G2 phase

c. S phase
d. Cytokinesis phase
e. G1 -phase
Question 39 30. Antibiotics that inhibit or inactivate cellular ribosomes will result directly in
Complete the loss of
Marked cut of 1
Select one:
a. DNA replication
b. ATP production

c. Protein synthesis

d. Peptidoglycan synthesis
e. Cell division

Question 40 28. Each of the following statements concerning penicillin is correct EXCEPT:
Complete
Marked aut of 1 Select one:
a. The structure of penicillin resembles that of a dipeptide of alanine, which
is a component of peptidoglycan.

b. An intact B-lactam ring of penicillin is required for its activity.


c. Penicillin inhibits transpeptidases, which are required for cross-linking
peptidoglycan.

d. Resistance to penicillin is known to be due to reduced affinity of


transpeptidases.

e. Penicillin is a bacteriostatic drug because autolytic enzymes are not


activated.
Question 41 5. One of the following bacteria cannot be classified using gram staining
wera ehesa-
Marked out of 1 Select one:
a. Bacillus

b. Escherichia a
c. Streptococcus
ae ae
d. Staphyloccocus
e. Mycoplasma

Question 42 19. One of the following best decribes the cell wall complex of Gram positive
Complete bacteria
Markec out of 1
Select one:
a. A structure composed of lipoteichoic, teichoic acids and peptidoglycan.
b. A thick, homogeneous peptidoglycan layer surrounded by an outer
membrane

c. A thin peptidoglycan layer surrounded by an external outer membrane


d. A structure that contains lipopolysaccharide, lipoteichoic acids and
peptidoglycan

e. A cell surrounded by an outer membrane, but lacking a peptidoglycan


layer

Question 43 15. Which of the following does NOT contribute to a pathogen’s invasiveness?
Complete

Marked cut of 1 Select one:


a. Ligands
b. Cell wall
c. Hyaluronidase
d. Toxins
e. Capsule
Question 45 12. The following statements are correct, except
Complete
Marked cut of 1 Select one:
a. Hyaluronidase breaks down substances between cells.
b. Kinase destroys fibrin clots.
c. Coagulase destroys blood clots
d. Hemolysins lyse red blood cells.
e. Leukacidins destroy neutrophils.
Question 49 13. The antibiotic trimethoprim antibiotics works by inhibiting the synthesis of
Complete

Marked cut of 1 Select one:


a. Peptidoglycan
b. DNA
c. Folic acid

d. Inhibit protein synthesis


e. RNA

Question 50 17. A cluster of polar flagella is called


Complete

Marked aut of 1 Select one:


a. Petritrichous

b. Lophotrichous
c. Monotrichous

d. Amphitrichous

e. Bipolar

Question 51 10. Microbe’s ability to alter their surface molecules and evade the host's
Complete antibodies is called
Marked cut of 1
Select one: een
a. Cytopathic effect.

b. Lysogenic conversion.
c. Virulence. i
d. Cytocidal effect.

e. Antigenic variation.
Question 53 21. Which one of the listed bacterial structures contains teichoic acid as a
Complete structural component?
Marked cut of 1
Select one:
a. Outer membrane of Gram-negative bacteria.
b. Cell wall of Gram-negative bacteria.
c. Cell membrane of either Gram positive or Gram negative bacteria.
d. Cell wall of Gram positive bacteria.
e. Porins of the Gram negative outer membrane.

Question 54 7. Which of the following is NOT a structural component of prokaryotes


Complete
Marked cut of 1 Select one:
a. Peptidoglycan
b. Cell wall 4

c. Cytoplasmic membrane

d. Endoplasmic reticulum
e. Plasma membrane
Question 56 8. Polymers of N-acetyigiucosamine and N-acetylmuramic acid are major
Complete components of
Markec cut of 1
Select one:
a. Glycocalyx
b. Cell wall
c. Capsule

d. Teichoic acid
€. Lipopolysaccharide
END OF YEAR 2020
EXTRACTED
QUESTIONS

Compiled by Jitu, 2022

Answered by Haron Sami


Question 3 Regarding hormones and gene regulation in eukaryotes, which statement is
Complete FALSE?
Marked out of 1
Select one:
a. Hormones act as inducers produced by one cell and cause a physiological
response in another cell.

b. Polypeptide hormones bind at the cell surface and activate


transmembrane enzymes to produce second messengers (such as CAMP) that
activate gene transcription.
» ¢, Hormone response elements (HREs) always occur in single copies in
enhancer sequence regions.
d, Steroid hormones pass through the cell membrane and bind cytoplasmic
receptors, which together bind directly to DNA and regulate gene expression,
e. Hormones act only on target cells with hormone specific receptors, and
levels of hormones are maintained by feedback pathways.
Question 6 The order of reagents used in the Gram staining procedure are:
Complete
Marked cut of 1 Select one:
a. Crystal violet, iodine, alcohol, safranin
b. Alcohol, crystal violet, iodine, safranin
c. lodine, crystal violet, safranin, alcohol
d. Crystal violet, safranin, alcohol, iodine
e. Crystal violet, iodine, safranin, alcohol

Question 7 The following represent different DNA lesions, EXCEPT:


Complete
Marked cut of 1 Select one:
a. Modified bases
b. Nucleotide duplication
c. Intra-strand crosslinks
d. A basic sites
e. Double stranded breaks

Question 8 The following are conjugating agents in drug biotransformation , EXCEPT:


Complete
Marked cut of 1 Select one:
a. Active acetate
b. Active glutamate
c. Active methionine

d. Active glucoronate
e. Active sulfate
Question 12 A progressive disorder that results in loss of multiple peroxisomal metabolic
Complete functions
Marked cut of 1
Select one:
a. Familial Hypercholestrolemia
b. Cystic Fibrosis
c. Inclusion Cell Disease
d. Zellweger syndrome
e€. Primary Hyperoxaluria

Question 13 Anucleoid can be described as the


Complete
Marked cut of 1 Select one:
a. Cytoplasmic location of the bacterial chromosome.
b. Cytoplasmic location of the bacterial nucleus.
c. Membrane bound location of the bacterial chromosome.
d. Cytoplasmic location of the bacterial ribosomes.
e, Membrane bound location of plasmids.
Question 15 An example of a large-scale mutation is:
Complete
Marked cut of 1 Select one:
a. Substitution

b. Nonsense mutation

c. Missense mutation
d. Copy number variation

e. Repeat expansion
Question 16 A defect in the Cernunnos/XRCC4-like factor (XLF) results in
Complete
Marked cut of 1 Select one:
a. Spinocerebellar ataxia with axonal neuropathy-1 (SCAN-1)
b. Radiosensitive severe combined immunodeficiency (RS)-SCID
c. Ataxia-oculomotor apraxia-1 (AOA-1)

d. Hyper-lgM Syndrome and Juvenile Polyposis


e. microcephaly, seizures, and developmental delay (MCSZ) syndrome
Question 21 The following are correct statements concerning peptidoglycan, EXCEPT:
Complete
Marked cut of 1 Select one:
a. Cross-links between the tetrapeptide involve D-alanine

yy\
b. It has a backbone composed of alternating units of muramic acid and
acetylglucosamine

c. It can be degraded by lysozyme


d. It is generally a site of many antibiotics
e. Itis thinner in gram-positive than in gram negative cells

Question 22 Septic shock in gram +ve bacteria can be caused by one of the following cell wall
Complete structures
Marked cut of 1
Select one:
a. Phospholipid
b. Teichoic acid
c. Peptidoglycan
d. Capsular protein
e. Endotoxin
Question 24 One of the following choices lists the steps of microbial pathogenesis in their
Complete correct order
Marked cut of 1
Select one:
a. Exposure, adhesion, invasion, infection

b. Disease, infection, exposure, invasion


c. Adhesion, exposure, infection, invasion

d. Infection, adhesion, invasion, exposure


e. Invasion, infection, adhesion, exposure
Question 27 Endotoxins:
Complete
Marked out of 1 Select one:
a. May be converted to toxoids.
b. Can act as antibiotics
c. Are only produced by Gram-negative bacteria.
d. Are lipopolysaccharide present on bacterial cel! wall

e. Are tissue-destroying enzymes.


Question29 Which of the following IS NOT a stage of virus replication?
Complete
Marked out of 1 Select one:
a. Attachment

b. Penetration
c. Protein expression

d. Uncoating
e. Assembly
Question 31 Gram-positive bacteria
Complete
Markec cut of 1 Select one:
a. Can release porins to poison the host.
b. Have a double membrane.
c. Have a thin peptidoglycan layer.
d. Are freely permeable to disinfectants.
e. Use teichoic acid to move ions across the cell wall.
Which of the following is a repair mechanism that directly ligates broken ends of
Complete DNA DSBs using a heterodimeric enzyme complex consisting of the proteins Ku-
Marked out of 1
70 and Ku-80.

Select one:
a. Reversal of damage
b. Nonhomologous end joining (NHE})
c. Mismatch repair

d. Recombination repair

e. Excision repair
Question 34 Which of the following resistance mechanisms describes the function of B-
Complete lactamase?
Marked cut of 1
Select one:
a. Efflux pump
b. Biofilm formation
c. Drug inactivation

d. Target mimicry
e. Target overproduction

The two major functions of the peptidoglycan structural layer include


Complete
Marked out of 1 Select one:
a, Transport of ions through the wall and production of endotoxins
b. Separate the cell membranes and generate energy for the cell
c. Prevent cell lysis and allow molecules to pass through the wall
d. Transport nutrients into the cell and synthesize porins
e. Prevent cell lysis and prevent diffusion of water from the cell

Question 36 Mismatch repair in E. coli involves one of the following proteins, EXCEPT:
Complete
Marked out of 1 Select one:
a. MutH

b. MutS
c. MuA

d. None of the stated


e. MutL
Question 40 The following are common ways in which bacteria can acquire antibiotic
Complete resistance, except
Marked cut of 1
Select one:
a. On its own through evolution
b. Acquiring resistance gene from its host's cells
c. From its parent cell

d. Scavenging resistance genes from the environment


e. Exchanging DNA with another bacterium
Question 42 The first step of O-glycosylation of target proteins in the Golgi complex is
Complete inhibited by
Marked cut of 1
Select one:
a. Antimycin A
b. Tunicamycin
c. Erythromycin
d. Ampicillin
e, Rotenone
Question 45 All the following interferons antiviral activity, EXCEPT:
Complete
Marked cut of 1 Select one:
a.B
b.y
ct

d.a
ew

Question 46 All the following are regulatory level of gene expression in eukaryotes, EXCEPT:
Complete
Marked out of 1 Select one:
a. RNA processing

b. mRNA degradation
c. Transcription
d. Protein degradation
e. Operon induction
Question 48 An N-linked structural oligosaccharide that acts as a signal that targets proteins
Complete to lysosomes consists of one or more of the following sugar residues.
Marked cut of 1
Select one: ew
a. N-glycoloyineuraminic acid
a
b. Fructose-6-phospate
c. N-acetyl glucosamine
d. N-acetylneuraminic acid
e. Mannose 6-phosphate
Question 49 What is the site of action of the antibiotic vancomycin?
Complete
Markec cut of 1 Select one:
a. RNA synthesis

b. Cross-linking of peptidoglycan
c. Protein synthesis via the SOS ribosomal subunit
d, Cell membrane integrity
e. DNA synthesis

Question 50 The following statements are true regarding the mode of action of antibacterial
Complete drugs, except:
Marked cut of 1

\
Select one:
a. Quinolones, such as ciprofloxacin, act by inhibiting the DNA gyrase of
bacteria.

\
b. Cephalosporins are bactericidal drugs that inhibit the transpeptidase
reaction and prevent cell wall synthesis

|
c. Vancomycin acts by inhibiting peptidoglycan synthesis.
d. Tetracyclines are bacteriostatic drugs that inhibit protein synthesis by
blocking tRNA binding.
e. Erythromycin is a bactericidal drug that disrupts cell membranes by a
detergent-like action.
Question 54 The following are all mechanisms of gene expression in prokaryotes, EXCEPT:
Complete
Marked cut of 1 Select one:
a. Repressible - e.g. the Trp Operon
b. Positive control - e.g. the Ara Operon
c. Negative control - e.g. the Ara Operon
d. Short-term - e.g. mRNA degradation
e. Inducible- e.g. Lac Operon

Question 55 Antibiotics that inhibit or inactivate cellular ribosomes leads to the loss of:
Complete
Marked cut of 1 Select one:
a. DNA replication

b. Peptidoglycan synthesis
c. ATP production
d. Cell division
e. Protein synthesis
Question 62 The periplasmic space is
Complete
Marked out of 1 Select one:
a. Made up of lipopolysaccharides and phospholipids
b. Larger in gram positive bacteria
c. An important reaction site for substances entering and leaving the cell
d. Where peptidoglycan is located
e. Absent in gram negative bacteria
Question 64 The following statements are true regarding short-term transcriptional control
Complete of galactose -utilizing genes in yeast, EXCEPT:
Markec cut of 1
Select one:
a. Additional unlinked gene, GAL4, codes a repressor protein that binds a
promoter element called an upstream activator sequence (UASG).

b. GAL genes are near each other but do not constitute an operon.

c. UASG is located between GAL1 and GAL10.


d. GAL genes are rapidly induced by the presence of galactose and absence
of glucose through GAL Operon.
e. Transcription occurs in both directions from UASG
Question 65 The antibiotic rifampicin
Complete
Markec cut of 1 Select one:
a. Activates the synthesis of RNA in prokaryotes
b. Is not used for treatment of tuberculosis
c. Interferes with tetrahydrofolate metabolism
d, Inhibits protein synthesis
e. Inhibits RNA synthesis in prokaryotes
Question 75 Aviral structural component used for packaging and protecting nucleic acids is
Complete
Marked cut of 1 Select one:
a. Core proteins
b. Capsid
c, Genome
d. Scaffolding proteins
e. Envelope
Question 77 The following statements are true about a gene promoter, EXCEPT:
Complete
Marked cut of 1 Select one:
a. A given gene may have one or many promoters

b. Promoters are activated by specialized transcription factor proteins


c. Occur upstream of the transcription start site.

d. Promoters cannot be positively or negatively regulated.


e. Some determine where and when a transcription begins.
Question 80 Which of the following human diseases is caused by a virusoid?
Complete
Marked cut of 1 Select one:
a. Alper’s Syndrome
b. Fatal Familial Insomnia.
c. Hepatitis B

d. Hepatitis D
e. vCJD
Question 82 The following statements are true about chloroquine, EXCEPT:
Complete
Marked cut of 1 Select one:
a. Chloroquine inhibits the formation of B-hematin
b. Mechanism of resistance involves ABC type transporters
c. Chloroquine is antimalarial drug
d. Resistance to Chloroquine is mediated by removal or inactivation of the
drug
e. Chloroquine inhibits the formation of toxic reactive oxygen species
Question 83 In cell cycle, DNA synthesis occurs in:
Complete
Marked cut of 1 Select one:
a. G2 phase
b. Cytokinesis phase
c. G1 -phase
d. G1 and G2 phases
» e.S phase
Question 91 Which of the following statements about DNA methylation in mammals is
Complete FALSE?
Marked out of 1
Select one:
a. Methylation occurs most often in symmetrical AT sequences.

b. DNA methylation is one of the basis for epigenetic inheritance.

c. Methylation results in a human disease called fragile X syndrome whereby


FMR-1 gene is silenced by methylation.
d. A gene for methylation is essential for development in mice.
e. Transcriptionally active genes possess significantly lower levels of
methylated DNA than inactive genes.
Gene enhancer sequences are found in eukaryotic systems, but not in prokaryotic systems.

Question 94 The he following statements are true about a gene enhancer sequence, EXCEPT:
Compiete
Marked cut of 1 Select one:
a. Loops may form in DNA bound to TFs and contact upstream enhancer
elements.
b. Interactions of regulatory proteins determine if transcription is activated
or repressed

c. Are found in both eukaryotic and prokaryotic systems


d. Regulatory proteins bind specific enhancer sequences; binding is
determined by the DNA sequence.
e. Occur upstream or downstream of the transcription start site.

Question 95 A nuclear localization sequence (NLS) is found in which of the following


Complete proteins?
Marked cut of 1
Histones are proteins that help package
Select one: DNA into a compact structure called
a. Alcohol dehydrogenase chromatin,which is found in the nucleus of
b. Histones eukaryotic cells.
c. NADH dehydrogenase The NLS allows histones to be transported
d, Peroxins into the nucleus where they can carry out
e. LDH
their function.
Question 99 The aminoglycoside antibiotics generally acts through
Complete
Marked cut of 1 Select one:
a. Inhibition of bacterial cell wall synthesis
b. Inhibition of bacterial DNA gyrase
c. Inhibition of protein synthesis by binding to the 30S ribosomal subunit
d. Interference with bacterial folic acid metabolism
e. Disruption of cytoplasmic membrane function
Question 102 Each of the following statements concerning bacterial spores is correct, EXCEPT:
Complete
Marked cut of 1 Select one:
a. Are more likely to survive treatment with disinfectants
b. They are formed by gram-positive rods
c. Their survival ability is based on their enhanced metabolic activity
d. They can be killed by being heated to 1210C for 15 minutes
e. They contain much less water than bacterial cells
An antimicrobial drug ability to harm the target microbe and not the host is
Complete termed as:
Marked cut of 1
Select one:
a. Minimum Inhibitory concentration
b. Therapeutic level
c. Spectrum of activity
d. Mode of action
e. Selective toxicity

Tf
Question 116 The following statements about puromycin are correct, EXCEPT :
Complete
Marked out of 1 Select one:
a. Its structure is analogous to 3’ end of an aminoacyl-tRNA

b. It activates the process of polypeptide synthesis


c. It inhibits protein synthesis
d. Is capable of peptide bond formation to form peptidyl-puromycin
e. Itis an antibiotic
Question 120 Both Viroids and Virusoids
Complete
Marked out of 1 Select one:
a. Have ssRNA Nucleic Acid

b. Need helper viruses


c. Have presence of protein
d. Have a capsid or envelope
e. Are affected by enzymes that digest DNA
Question 122 Which of the following is NOT a membrane-disrupting toxin?
Complete
Marked cut of 1 Select one:
a. Streptolysin S
b. Streptolysin

c. A-B toxin

d. Hemolysin
e. Leukacidin
Question 132 An example of Novel Disorder of Nucleotide Excision Repair (NER) is.
Complete
Marked cut of 1 Select one:
a. Hyper-lgM Syndrome and Juvenile Polyposis
b. Ataxia-oculomotor apraxia-1 (AOA-1)
c. Cockayne syndrome (CS)
d. Radiosensitive severe combined immunodeficiency (RS)-SCID
e. Spinocerebellar ataxia with axonal neuropathy-1 (SCAN-1)
Question 134 Regarding resistance to antibiotics
Complete
Marked cut of 1 Select one:
a. Macrolides can be inactivated by transferases
b. Bacterial resistance to aminoglycoside drugs is due to mutation of the
drug binding site
c. Penicillinases cannot inactivate cephalosporins

d. Tetracycline resistance is a marker for multidrug resistance


e. Resistance to antibiotics is rarely plasmid encoded
Question 140 One of the following human genes is regulated by alternative polyadenylation &
Complete splicing
Markec cut of 1
Select one:
a. Insulin
b. DNase 1
c. Human calcitonin (CACL)

d. Hemoglobin
e. Galactokinase
SPECIAL/
SUPPLEMENTARY
2020 EXTRACTED
QUESTIONS

Answered by Haron Sami

Compiled by Jitu, 2022


One of the following structures is unique to
gram-negative but not in gram-positive
bacteria

Select one:
a. Cytoplasmic membrane
O

©) _ b. Cell wall
() c.Endospore
() d, Outer membrane

* e. Capsule
The periplasmic space is

Select one:
() a. Larger in gram positive bacteria

©) b. Where peptidoglycan is located

©) cc. Absent in gram negative bacteria

() d. Made up of lipopolysaccharides and


phospholipids

() e.Animportant reaction site for


substances entering and leaving the cell
Homologous recombination

Select one:
() a. Doesn't require the formation of
holliday junction

() | b. Involves the sharing of extensive


regions of similar DNA sequence
() c. Reduces genetic diversity

() d. Involves the sharing of very short


regions of DNA sequence

©) e. Doesn't occur in bacteria

Homologous recombination is a type of genetic recombination in which nucleotide sequences are


exchanged between two similar or identical molecules of DNA. It is a common mechanism for repairing
double-strand breaks in DNA and for generating genetic diversity during meiosis. The process involves
the formation of a Holliday junction intermediate and the exchange of genetic information between
homologous DNA sequences.
Antibiotics that inhibit or inactivate cellular
ribosomes leads to the loss of:

Select one:
a. Protein synthesis
O

©) _ b. Peptidoglycan synthesis
©) c. ATP production
©) d.DNA replication

O e. Cell division
One of the following groups of viruses
contain the lysozyme enzyme to aid in their
infection

Select one:
a. Plant Viruses
0 ©

b. Human Viruses
OOo

c. Fungal Viruses

d. Bacteriophage

e. Animal Viruses
COVID-19 is commonly diagnosed by use of
one of the following tests

Select one:
a. Virus isolation in cell culture
Wt) ©

b. Urinalysis

c. Immunofluorescent assay (IFA)


d. Viral antigen detection test

e. Real-time reverse transcription


polymerase chain reaction (rRT-PCR) assay
The antibiotic vancomycin acts by interfering
with

Select one:
©) a. Cell membrane integrity

©) _ b. Protein synthesis via the 50S


ribosomal subunit

©) c. Cross-linking of peptidoglycan

() d.RNAsynthesis
() e.DNAsynthesis
Gram positive bacteria cell wall complex can
be best described by one of the following
statements

Select one:
() a.Athin peptidoglycan layer surrounded
by an external outer membrane

() b.Astructure composed of lipoteichoic,


teichoic acids and peptidoglycan.

©) c.Acell surrounded by an outer


membrane, but lacking a peptidoglycan
layer

() d.Astructure that contains


lipopolysaccharide, lipoteichoic acids and
peptidoglycan

©) e.Athick, homogeneous peptidoglycan


layer surrounded by an outer membrane
The exchange of homologous segments of
RNA between two different influenza type A
viruses is called

Select one:
a. Phenotypic masking
OOO0OD0O0

b. Super infection

c. Genetic reassortment

d. Complementation

e, Phenotypic mixing
Which of the following statements is not true
about protein targeting in bacteria

Select one:
() a. SecA Protein is required

() _ b. SecA Protein releases the translated


polypeptide to periplasmic space

©) cc. SecB Protein is required

C) d.€E. coli lack protein targeting


mechanism

©) e.ATP is the source of energy in


translocation
One of the following is a virus that infects and
lyses progenitor erythroid cells causing
aplastic crises in patients with hemolytic
anemia

Select one:
() a. Parvovirus B19
() _ b. Epstein-Barr virus
©) cc. Hepatitis B virus
() d. California encephalitis virus

©) e. Yellow fever virus


Methanogens - convert CO2 and H2 gases
into methane

Select one:
() a. Thermo plasmas - adapted to frozen
environments

() _ b. Extreme halophiles - adapted to salty


habitats

() c. Hyperthermophiles - adapted to high


temperatures
() d. Psychrophiles - adapted to very low
temperatures
Trp operon

Select one:
() a. Regulates the expression of genes
responsible for tryptophan metabolism

©) _ b. Regulates expression of genes


responsible for arabinose metabolism

©) c. Activates utilization of tryptophan in


prokaryotic cells

©) d. Inhibits expression of genes required


for tryptophan catabolism

©) e. Controls the breakdown of lactose


Which of the following organism's genomes is
likely to have the highest gene density?

Select one:
a. Pig

b. Cat
Ot

c. E. coli

d. Human immunodeficiency virus

e. Cholera bacteria
The following are common ways in which
bacteria can acquire antibiotic resistance,
EXCEPT

Select one:
() a. Exchanging DNA with another
bacterium

() _ b. Scavenging resistance genes from the


environment

() cc. Acquiring resistance gene from its


host's cells

©) d. Onits own through evolution


() e. From its parent cell
a
Common DNA damage caused by UV light is

Select one:
() a. Formation of thymine-thymine dimers

©) b.A-C mismatch
() cc. Deamination of cytosine to uracil

() d.Nicking of the DNA strands


©) e. Winding of the DNA duplex
Host cell tRNAs are involved in the genome
replication of

Select one:
a. Rhinovirus
oO @

b. Respiratory syncytial virus

c. Adenoviruses

d. Retroviruses

e. Influenza A virus
Regarding formation of bacterial endospore

Select one:
() a. Endospores are for reproduction

() _ b. Endospores are easily stained in a


Gram stain

©) c.Acell can produce many endospores

() d. Endospores allow a cell to survive


environmental changes
©) e.Acell produces one endospore and
keeps growing
One of the following stains is used to classify
microbes based on their cell wall content.

Select one:
a. Spore stain
OgOoO©

b. Negative stain

c. Methylene blue

d. Gram stain

e. Capsular stain
The following statements about puromycin
are correct, EXCEPT

Select one:
() a. lt activates the process of polypeptide
synthesis

©) _ b. It is an antibiotic
() _c. Its structure is very similar to 3’ end of
an aminoacyl-tRNA

() d.Is capable of peptide bond formation


to form peptidyl-puromycin

©) e. It inhibits protein synthesis


Which of the following, pertaining to
prokaryotic cell membranes is mismatched?

Select one:
() a. Unique hydrocarbon present - cyano
bacteria

©) b. Sterols present - mycoplasmas

©) cc. Contain photosynthetic pigments -


cyanobacteria

() d. Forma bilayer - phospholipids

©) e. Function - regulates transport of


nutrients and wastes
An eight-piece segmented negative-sense
RNA genome of influenza virus that encodes
the HA (hemagglutinin) protein binds directly
to one of the following host cells epithelial
components

Select one:
a. Chemokine receptors
Ovugeaod

b. Cytokine receptors
c. Uronic acid subgroups

d. Serine-threonine kinase receptors

e. Sialic acid

Sialic acid is a type of sugar molecule that is found on the surface of many host cell types,
including epithelial cells in the respiratory tract. Influenza virus uses the HA protein to bind to sialic
acid on the host cell surface, which facilitates viral entry into the host cell and infection.
Lipopolysaccharide is an important cell wall
component of

Select one:
() a.Gram positive bacteria

©) _ b. Acid fast bacteria


() c.Mycoplasmas
() d. Gram negative bacteria
() e. Protoplasts
One of the following statements about type 1
diabetes mellitus is correct

Select one:
() a. ltis mainly treated with insulin
() b. Can be caused by excessive or
inappropriate glucagon secretion.

©) c. Occurs mainly in adults


() d. It is caused due to the malfunctioning
of the pancreatic alpha cells

C) e.ltis generally treated with metformin


(Glucophage)
Bacterial appendages are involved in

Select one:
a, Attachment and protection
O

©) _ b. Motility and slime production


() cc. Protection and motility
() d. Attachment and motility

O e. Energy reactions and synthesis


Fluoroquinolone antibiotics, act by inhibiting

Select one:
a. Prostaglandin synthesis
Ogoono

b. RNA synthesis

c. Folic acid synthesis

d. DNA synthesis

e. Protein synthesis
A bacterial cell wall performs all of the
following functions, EXCEPT

Select one:
() a.ltis important in bacterial
identification
©) b. Is associated with some symptoms of
disease

©) c. Gives shape and rigidity to the cell

©) d.Is the site of action for some


antibiotics
() e. Protects the cell from phagocytosis
The presence of flagella all over the bacterial
cell surface is often referred to as

Select one:
a. Monotrichous
O

O b. Atrichous

©) c. Amphitrichous

© d. Lophotrichous

© e. Peritrichous
Gram negative organisms

Select one:
() a. Stain purple in the gram stain
() b.Are more susceptible to antibiotics
than gram positive organisms

() c.Are less susceptible to antibiotics than


gram positive organisms

() d. None of the choices are correct


() e. Encompass all pathogens
Which of the following is mismatched?

Select one:
() a. Cytoplasm - dense, gelatinous solution
() _ b. Plasmids - genes essential for growth
and metabolism
() cc. Nucleoid - hereditary material

() d. Inclusions - excess cell nutrients and


materials
() e. Ribosomes - protein synthesis
The sequence of cell envelope components in
Gram-negative bacteria from cell's inside are

Select one:
() a. Outer membrane, peptidoglycan, cell
membrane
() _ b. Phospholipid bilayer, porins, LPS,
Teichoic acids
() c. Cell membrane, periplasmic space,
outer membrane
() d.Cell membrane, peptidoglycan layers
periplasmic space
() e. Cell membrane, periplasmic space,
peptidoglycan outer membrane
Leber’s hereditary optic neuropathy (LHON) is
caused by

Select one:
() a. Inversion of a segment in long arm of
chromosome 21

b. Mutation on chromosome 18
OWOG @

c. Mutation on chromosome 13

d. Mutation in mitochondrial DNA

e. Deletion on p arm of chromosome 5


DNA mutations can be caused by

Select one:
a. DNA recombination
Oo0ocdo

b. X- and gamma-rays
c. chemical compounds

d. DNA replication

e. Visible light
Gram-positive bacteria

Select one:
a. Have a double membrane.
@2OOG

b. Can release porins to poison the host.

c. Have a thin peptidoglycan layer.


d. Use teichoic acid to move ions across
the cell wall.

e. Are freely permeable to disinfectants.


2

Gram-positive bacteria have a thick peptidoglycan layer in their cell wall and do
not have a double membrane. They also contain teichoic acids in their cell
wall, which can help to move ions across the cell wall.
One of the following bacterial resistance
mechanisms is effective against a wide range
of antimicrobials

Select one:
a. Target overproduction
OOO0D0

b. Target modification
c. Drug inactivation

d. Efflux pump
e. Target mimicry
Recombination in meiosis occurs during

Select one:
a. Metaphase II
O

©) b. Telophase
() c. Metaphase!
() d. Prophase |

O e. Prophase |!
One of the following microbial compounds
causes fever and can withstand high
temperatures.

Select one:
a. Lipid A
OOQdO©

b. Endotoxin

c. Lipoprotein

d. Diaminopimelic acid
e. Peptidoglycan
The two major functions of the peptidoglycan
structural layer include

Select one:
() a. Prevent cell lysis and prevent diffusion
of water from the cell
() b. Prevent cell lysis and allow molecules
to pass through the wall
() cc. Transport of ions through the wall and
production of endotoxins

() d. Transport nutrients into the cell and


synthesize porins
() e. Separate the cell membranes and
generate energy for the cell
The following factors contributes to
pathogen's invasiveness of a tissue, EXCEPT

Select one:
a. Cell wall
OOO0OD0

b. Capsule
c. Toxins

d. Ligands
e. Hyaluronidase
A type of cell culture that can reproduce for
an extended number of generations and is
used to support viral replication is a:

Select one:
a. Diploid fibroblast cell
OOOO 0

b. Continuous
cell line
c. Primary cell culture
d. Connective tissue

e. Cell strain
A Gram-negative bacterium does not retain
crystal violet stain because

Select one:
() a. Bacteria have well developed fimbrae
() _ b. Bacteria have thick peptidoglycan
layer

() c. Bacteria have thin peptidoglycan layer


() d. Periplasmic space is absent

C) e. Cell wall include significant amount of


teichoic and lipoteichoic acids
oO

The antibiotic trimethoprim antibiotics works


by inhibiting the synthesis of

Select one:
a. DNA
OOOW0OD0

b. Folic acid

c. Peptidoglycan

d. RNA
e, Inhibit protein synthesis
Which one of the following is the MOST
accurate regarding bacterial spores?

Select one:
() a. They contain endotoxin, which
accounts for their ability to cause disease.
() b. One spore germinates to form one
bacterium.

() c. They are produced primarily within


human red blood cells.
() d. They are killed by boiling at sea level
but not at high altitude.

() e. They are produced by anaerobes only


in the presence of oxygen.
All bacterial cells have

Select one:
a. The ability to produce endospores
b. Capsules
Jt]
O GO

c. One or more chromosomes


d. Flagella
e, One or more fimbriae
Each of the following structures is used for
bacterial adherence, EXCEPT

Select one:
a. Lipoproteins
OOO0OD

b. Glycoproteins

c. Cell membrane mannose

d. Capsules

e. Fimbriae
The following structures contribute to the
ability of pathogenic bacteria to cause
disease, EXCEPT

Select one:
() a. Inclusions
() b. Fimbriae
() c. Outer membrane of Gram-negative
cell walls
() d. Capsule
C) e. Slime layer
Exotoxins

Select one:
() a.Are generally heat resistant.

() b. Enhance the invasive ability of


bacteria.

() c. Are convertible to toxoids.

() d. Can inhibit protein synthesis


() e. Such as those produced by “flesh-
eating bacteria” act to degrade human
tissues.
Bacterial resistance to B-lactam antibiotics

Select one:
() a.Ils most commonly due to modification
of the target PBPs

©) b. Can involve up to 5 different beta-


lactamases

() cc. Does not involve penetration of drug


to target PBPs

() d.Infers resistance only to penicillins


() e. Can be due to an efflux pump
One of the following is a receptor for the
SARS-CoV-2 Virus

Select one:
a. CPG
OOod00

b. Erb-B

c. CXCR4

d. ACE-2

e. TLR-9
The size of viruses varies from

Select one:
() a.400-1000 nm
C) b. 1-100 nm
©) c¢. 10-100 um
©) d. 25-300 nm
©) e.1-10 um
One of the following choices lists the steps of
microbial pathogenesis in their correct order

Select one:
() a. Disease, infection, exposure, invasion
() _ b. Invasion, infection, adhesion,
exposure
©) cc. Infection, adhesion, invasion,
exposure
() d.Adhesion, exposure, infection,
invasion

() e. Exposure, adhesion, invasion,


infection
One of the following viral structural
components is used for packaging and
protecting viral nucleic acids

Select one:
a. Genome
OOO0OD0

b. Envelope
c. Core proteins

d. Capsid
e. Scaffolding proteins
In the adrenal medulla, epinephrine is
synthesized from norepinephrine by

Select one:
a. Methylation
OOOd0O

b. Decarboxylation

c. Dehydrogenation

d. Deamination

e. Hydroxylation
Bacterial plasmids

Select one:
a. Cannot be passed on to progeny
C2 oP &

b. Cannot be passed between organisms


(2)

c. Are often the site of pathogenic genes

d. Are essential for survival

e. Are found in all bacteria

Plasmids are small, circular pieces of DNA that can be transferred between
bacteria and can carry genes that provide a selective advantage to the
bacterium, such as antibiotic resistance or virulence factors.
Individuals taking antibiotics often develop
infections due to Candida albicans because

Select one:
() a. The antibiotics damage the host
mucous membranes.

() b. Antibiotics support the growth of C.


albicans.
() c. The normal bacterial flora is greatly
affected by antibiotics
() d.Albicans degrades the antibiotics.
() e. Antibiotics stimulate the synthesis of
ergosterol of C. albicans.
The bacterial cell wall has all of the following
properties, EXCEPT:

Select one:
() a. Itis a unique flexible plastic structure
©) _ b. It is the structure principally
responsible for the reaction of gram
staining

©) c. It consists of a mixed polymer called


peptidoglycan

() d. Consists of a glycan backbone, a


tetrapeptide with a peptide crosslinkage
() e. It contains D-isomers of amino acids
END COVER

Biochemistry Level 2 Mid Semester 2 Revision MCQs

Compiled by Jitu, Thursday April 7 2022


1st Update, Saturday 30th April 2022
2nd Update, Monday March 13 2023

For if we have seen further, it is by standing


On the shoulders of giants who came before us.

~Sir lsaac Newton in a letter to Robert Hooke, 1675.

END

Regards,
Haron Sami
6/04/2023
Incase of any corrections,reach out to me
0300hrs Tel:+254745776087

You might also like